Sie sind auf Seite 1von 233

Problemas Propuestos y Resueltos

de Electromagnetismo

~ ~D = r n

~ ~B = 0

q0

~ = J~ + D
~
~ H
~ ~E = ~B t
t

Rodrigo Chi Duran


email: rchi@ing.uchile.cl

Version 1.2 - Marzo 2016


ndice general

I Electrosttica 1

1. Ley de Coulomb y Distribuciones Discretas de Cargas 3

I. Problemas Propuestos . . . . . . . . . . . . . . . . . . . . . . . . . . . . . . . . . . 3

II. Soluciones . . . . . . . . . . . . . . . . . . . . . . . . . . . . . . . . . . . . . . . . 5

2. Distribuciones Continuas de Carga 7

I. Problemas Propuestos . . . . . . . . . . . . . . . . . . . . . . . . . . . . . . . . . . 7

II. Soluciones . . . . . . . . . . . . . . . . . . . . . . . . . . . . . . . . . . . . . . . . 12

3. Ley de Gauss 25

I. Problemas Propuestos . . . . . . . . . . . . . . . . . . . . . . . . . . . . . . . . . . 25

II. Soluciones . . . . . . . . . . . . . . . . . . . . . . . . . . . . . . . . . . . . . . . . 29

4. Conductores, Condensadores y Energa 35

I. Problemas Propuestos . . . . . . . . . . . . . . . . . . . . . . . . . . . . . . . . . . 35

II. Soluciones . . . . . . . . . . . . . . . . . . . . . . . . . . . . . . . . . . . . . . . . 41

5. Ecuacin de Laplace/Poisson y Mtodo de las Imgenes 57

I. Problemas Propuestos . . . . . . . . . . . . . . . . . . . . . . . . . . . . . . . . . . 57

II. Soluciones . . . . . . . . . . . . . . . . . . . . . . . . . . . . . . . . . . . . . . . . 62

3
4

6. Dipolo Elctrico 79

I. Problemas Propuestos . . . . . . . . . . . . . . . . . . . . . . . . . . . . . . . . . . 79

II. Soluciones . . . . . . . . . . . . . . . . . . . . . . . . . . . . . . . . . . . . . . . . 81

II Corriente Elctrica 87

7. Medios Conductores y Ecuacin de Continuidad 89

I. Problemas Propuestos . . . . . . . . . . . . . . . . . . . . . . . . . . . . . . . . . . 89

II. Soluciones . . . . . . . . . . . . . . . . . . . . . . . . . . . . . . . . . . . . . . . . 94

8. Circuitos Elctricos 99

I. Problemas Propuestos . . . . . . . . . . . . . . . . . . . . . . . . . . . . . . . . . . 99

II. Soluciones . . . . . . . . . . . . . . . . . . . . . . . . . . . . . . . . . . . . . . . . 102

III Magnetosttica 105

9. Ley de Biot-Savart 107

I. Problemas Propuestos . . . . . . . . . . . . . . . . . . . . . . . . . . . . . . . . . . 107

II. Soluciones . . . . . . . . . . . . . . . . . . . . . . . . . . . . . . . . . . . . . . . . 110

10.Fuerza de Lorentz 113

I. Problemas Propuestos . . . . . . . . . . . . . . . . . . . . . . . . . . . . . . . . . . 113

II. Soluciones . . . . . . . . . . . . . . . . . . . . . . . . . . . . . . . . . . . . . . . . 117

11.Ley de Ampre 119

I. Problemas Propuestos . . . . . . . . . . . . . . . . . . . . . . . . . . . . . . . . . . 119

II. Soluciones . . . . . . . . . . . . . . . . . . . . . . . . . . . . . . . . . . . . . . . . 122


5

12.Potencial Vectorial y Momento Magntico 127

I. Problemas Propuestos . . . . . . . . . . . . . . . . . . . . . . . . . . . . . . . . . . 127

II. Soluciones . . . . . . . . . . . . . . . . . . . . . . . . . . . . . . . . . . . . . . . . 131

IV Campos Electromagnticos Variantes en el Tiempo 137

13.Ley de Faraday-Lenz 139

I. Problemas Propuestos . . . . . . . . . . . . . . . . . . . . . . . . . . . . . . . . . . 139

II. Soluciones . . . . . . . . . . . . . . . . . . . . . . . . . . . . . . . . . . . . . . . . 143

14.Inductancia y Energa Magntica 147

I. Problemas Propuestos . . . . . . . . . . . . . . . . . . . . . . . . . . . . . . . . . . 147

II. Soluciones . . . . . . . . . . . . . . . . . . . . . . . . . . . . . . . . . . . . . . . . 152

15.Corriente Alterna 159

I. Problemas Propuestos . . . . . . . . . . . . . . . . . . . . . . . . . . . . . . . . . . 159

II. Soluciones . . . . . . . . . . . . . . . . . . . . . . . . . . . . . . . . . . . . . . . . 164

16.Leyes de Maxwell y Ondas Electromagnticas 167

I. Problemas Propuestos . . . . . . . . . . . . . . . . . . . . . . . . . . . . . . . . . . 167

II. Soluciones . . . . . . . . . . . . . . . . . . . . . . . . . . . . . . . . . . . . . . . . 171

V Campos Electromagnticos en Medios Materiales 175

17.Campo Elctrico en Medios Materiales 177

I. Problemas Propuestos . . . . . . . . . . . . . . . . . . . . . . . . . . . . . . . . . . 177

II. Soluciones . . . . . . . . . . . . . . . . . . . . . . . . . . . . . . . . . . . . . . . . 182


6

18.Campo Magntico en Medios Materiales 191

I. Problemas Propuestos . . . . . . . . . . . . . . . . . . . . . . . . . . . . . . . . . . 191

II. Soluciones . . . . . . . . . . . . . . . . . . . . . . . . . . . . . . . . . . . . . . . . 195

VI Respuestas 199

19.Respuestas 201
i

Prlogo

El Electromagnetismo es posiblemente una de las ramas ms bonitas de las fsica, la cual tiene muchas
aplicaciones cotidianas de las cuales no nos damos cuenta: prender la luz, llamar por celular o usar
el computador. El estudio de esta rea en el ltimo siglo ha provocado un avance considerable en la
tecnologa y nos entregan un mayor bienestar diariamente.

El apunte aqu presente, nace como una recopilacin de problemas propuestos y resueltos durante el
tiempo que he sido Profesor Auxiliar en la Universidad de Chile y mi breve paso por la Universidad
de los Andes. La mayora de los problemas disponibles han sido extrados de evaluaciones (controles,
ejercicios, tareas, etc) y de guas de problemas propuestos que elabor para que mis alumnos estudia-
ran. Aclaro que la mayora de los problemas presentes no son de mi autora sino de los profesores con
los que he trabajado y de algunos libros de la bibliografa.

Mi principal objetivo con este apunte es entregar un buen material de estudio para las personas que
necesiten estudiar y/o que simplemente quieran aprender. Adems, el hecho de reunir el material
que confeccion durante varios semestres en un solo lugar hace que el trabajo sea mucho ms til y
duradero para las personas que quieran utilizarlo.

Este compilado posee dos tipos de problemas: algunos con su solucin completa y otros que solamente
poseen su respuesta final. Como siempre es recomendado, es importante que al momento de usar este
apunte se den el tiempo de pensar el problema antes de mirar su solucin (si es que la posee). Un rol
activo en la resolucin de problemas les traer muy buenos resultados durante este curso.

He querido ser detallista en la seleccin de problemas, de modo que en la mayora de los captulos he
intentado plasmar un espectro representativo de los de problemas que suelen ser preguntados en la
FCFM (aunque hay profesores que su ingenio siempre puede ms).

Dado lo reciente de esta recopilacin, probablemente existan errores de los cuales han pasado des-
apercibidos. Les pido por favor a los estudiantes que los encuentren que me los notifiquen, as ganarn
buen karma y otros futuros estudiantes se los agradecern .

Finalmente, quiero agradecer a las personas que han hecho posible realizar este proyecto, a los profe-
sores Pablo Zegers, Daniel Escaff, Simn Casassus, Carlos Cartes, Takeshi Asahi, Matas Montesinos
y en particular, a Marcel Clerc y Claudio Romero. Tambin a mis compaeros auxiliares que aportaron
con problemas y soluciones: Susana Mrquez y Luis Mateluna.

Mucho xito!
ii

Notas sobre la Versin 1.2

La presente versin cuenta con 18 captulos, donde existen 201 problemas propuestos de los
cuales 86 tienen solucin.

Los problemas tienen una simbologa de acuerdo a su dificultad:

significa que un problema es sencillo y debera ser resuelto en forma rpida.


significa que el problema intermedio y requiere un mayor anlisis o trabajo algebraico.
significa que es un problema difcil, que requiere un anlisis prolongado.

Todos los problemas del apunte cuentan con su respectiva respuesta al final del documento. Se
puede llegar fcilmente a su respuesta presionando el smbolo X .

En el enunciado de cada problema se detalla si este tiene solucin mediante el smbolo S . Si


se presiona el smbolo se puede llegar rpidamente a su solucin.

He dejado el captulo de Campo Elctrico/Magntico en Medios Materiales al final del apunte,


ya que siempre me ha gustado ms esa forma de ver los contenidos del curso.
Parte I

Electrosttica

1
Ley de Coulomb y Distribuciones Discretas de
1
Cargas

I. Problemas Propuestos

Problema 1.1 X S
Suponga que en lugar de la Ley de Coulomb, uno hubiera
encontrado experimentalmente que la fuerza entre dos
cargas puntales fuera
p q2
q 1 q 2 (1 |~r2 ~r1 |) ~F21
F~12 = (~r2 ~r1 )
40 |~r2 ~r1 |3

donde es una constante.


~r2
a) Escriba el campo elctrico a una carga puntual.
Coloque el origen de coordenadas en la carga pun-
tual. q1
b) Elija una trayectoriacerrada alrededor de la carga ~r1 O
y calcule la integral E ~ Compare el resultado
~ dl. ~F12
obtenido con la Ley de Coulomb.

c) Encuentre en valor de la integral E ~ sobre
~ dS
una superficie esfrica centrada en la carga. Com-
pare el resultado obtenido con la Ley de Coulomb.
e
Problema 1.2 X S
En los vrtices de un tringulo equiltero de lado L se
han situado tres cargas negativas e. Si en el centro de L L
gravedad del tringulo se sita una carga de magnitud
Q, determine el valor que debe poseer esa carga para Q
mantener el sistema en equilibrio.
e L e

3
4 CAPTULO 1. LEY DE COULOMB Y DISTRIBUCIONES DISCRETAS DE CARGAS

Problema 1.3 X
z
Ocho partculas puntuales con carga q estn ubicadas en
los vrtices de un cubo de lado a como se muestra en
la figura. Llamaremos P al punto ubicado en el centro
de la cara del cubo que yace sobre el plano y = a (ver
a P
figura).
a) Determine el campo elctrico producido en el pun-
to P por las cuatro cargas que se ubican en y = a. a y
a
b) Encuentre el campo elctrico en P producido por
la carga ubicada en el origen.
x
c) Calcule el campo elctrico total en el punto P .

+q

Problema 1.4 X +q q

y
Considere seis cargas puntuales ubicadas en los vrtices
de un hexgono regular de lado a. Existen tres cargas a
x
positivas q y tres cargas negativas q distribuidas como
se muestra en la figura. Determine el campo elctrico y +q q
el potencial elctrico en el centro de hexgono.

Problema 1.5 X +q
Dos cargas puntuales positivas +q estn separadas por a
una distancia 2a. Por el punto medio del segmento que
las une se traza un plano perpendicular al mismo. El
Plano Divisor
lugar de los puntos en que la fuerza sobre una carga de a
prueba situada en el plano es mxima es, por razn de
+q
simetra, una circunferencia. Encuentre su radio.

Problema 1.6 X
La cohesin en los cristales inicos se debe a las fuerzas Na
elctricas de atraccin entre los iones positivos y nega-
Cl
tivos. Considere un modelo unidimensional de un cristal
de Cloruro de Sodio (NaCl) que consiste en una lnea a Na
recta infinita en que estn colocados los iones, alternn-
Cl
dose los iones positivos de Na y los iones negativos de
Cl. La distancia entre iones vecinos es a. Los iones posi- Na

tivos tienen carga +e y los iones negativos e. Calcule


Cl
la energa que hay que entregarle a un ion positivo de
Na para sacarlo de su lugar y llevarlo a una distancia
muy grande respecto a a.
II. SOLUCIONES 5

II. Soluciones

Solucin 1.1 P X

a) Colocando la carga q1 en el origen se obtiene ~r1 = 0. Luego llamando q1 = q y recordando que


~ se puede obtener que
F~ = q2 E,
p p
q 2 q (1 |~r2 |) q (1 |~r2 |)
F~ = 3
~ =
r~2 = E ~r2
40 |~r2 | 40 |~r2 |3

Como la direccin de la fuerza y el campo es radial podemos decir que ~r2 = rr. Finalmente

~ q (1 r)
E= r
40 r2

b) Basta tomar una trayectoria cerrada , por ejemplo, una circunferencia de radio R alrededor de
la carga. En ese caso se tiene que la curva queda parametrizada como:

2
~ d~l =
E ~ Rd = 0
|E|r

0

Para el caso de la ley de


Coulomb se tiene que el campo es siempre conservativo, es decir
E ~ = 0, por lo cual ~ d~l = 0 para cualquier camino cerrado . Se concluye que la ley
E

de Coulomb y la ley encontrada experimentalmente entregan el mismo resultado.

c) Para este caso hay que calcular una integral de flujo. Por simplicidad se elige una esfera de radio
R centrada en el origen. Luego el flujo sobre su superficie es

" 2

~ dS
~= q (1 R) 2 q
E rR sin ddr = (1 R)
40 R2 0
0 0

Para el caso de la ley de Coulomb, se sabe que esa integral es conocida ya que coincide con la
Ley de Gauss, es decir "
E ~= q
~ dS
0

Para la nueva ley se tiene que difiere en un factor (1 R) , adems el flujo no es constante
como en la Ley de Gauss y depende de la superficie esfrica que se tome.
6 CAPTULO 1. LEY DE COULOMB Y DISTRIBUCIONES DISCRETAS DE CARGAS

Solucin 1.2 P X

En este problema se aprecia una simetra con respecto a la carga Q, ya que todas las cargas
negativas sienten la misma fuerza de atraccin/repulsin. Por ejemplo, haciendo el DCL a la
carga superior se obtiene lo siguiente

~F1 ~F2
y

x e
~F3

L L

e L e

Figura 1.1: DCL Sobre la Carga Superior

Las fuerzas F~1 y F~2 son repulsivas (generadas por las otras cargas negativas), mientras que la
fuerza F~3 generada por la carga Q positiva es atractiva. Usando el sistema de referencia de la
Figura 1.1 y la Ley de Coulomb, es posible descomponer las fuerzas de la siguiente forma

e2   e2   eQ
F~1 = x cos + y sin , F~2 = x cos + y sin , F~3 = y
40 L2 3 3 40 L2 3 3 40 d2
donde d es la distancia desde cualquier vrtice al centro del tringulo. Usando el Teorema del
Coseno, es posible despejar d como

2 L2
L2 = d2 + d2 2d2 cos = d2 =
3 3
por lo tanto la fuerza total vale
 
e2 3eQ
F~T = sin y
20 L2 3 40 L2

Dado que se desea el sistema en equilibrio se impone F~T = 0, obtenindose que



e2 3eQ e 3 e
sin = 0 = Q = =
20 L2 3 40 L2 3 3
Distribuciones Continuas de Carga
2
I. Problemas Propuestos

Problema 2.1 X S Casquete Semiesfrico

Un disco de radio a completa un casquete semiesfrico


de radio a. Ambas superficies tienen densidad de carga
uniforme . Calcule el campo elctrico en un punto a2 s
sobre el eje Z.
s a
Hint: Y
 
d r z cos(x) (z r cos x) sin x
= 3
dx z 2 r2 2rz cos x + z 2 (r sin2 x + (z r cos x)2 ) 2
2
Disco

l
Problema 2.2 X S
~g
Considere una barra infinita de densidad de carga lineal
. Sobre esta barra se cuelga un pndulo ideal de largo q
l
l y una masa puntual m y carga q, bajo la influencia del
campo gravitatorio como se ilustra en la figura. Encuen-
tre el ngulo de equilibrio del pndulo y determine cul m, q
es ngulo lmite cuando el valor de q crece infinitamen-
te.

X S
Problema 2.3 Q
Un anillo de radio R0 tiene una carga Q positiva, la cual
est distribuida de manera uniforme sobre el anillo, co-
mo se ilustra en la figura. Considere una carga puntual R0
de carga negativa q (q < 0) y masa m, la cual es de-
positada en reposo sobre el eje central del anillo cerca k0
del centro representado por el punto A, adems la car- A q
ga est soldada a un resorte ideal de constante elstica
k0 y largo natural cero con extremo fijo en el punto A.
Calcule la frecuencia de oscilacin partcula puntual. In-
dicacin: Considere que la partcula se mueve sobre el
eje central del anillo.

7
8 CAPTULO 2. DISTRIBUCIONES CONTINUAS DE CARGA

Problema 2.4 X S
Una densidad de carga lineal est repartida de for- l
ma homognea a lo largo de un semicircunferencia BD
centrada en C y de radio R. Una carga puntual q es-
t ubicada en punto A como se indica en la Figura. R
(CA = R).
a) Calcule el potencial elctrico en el punto C, V (C). B C D
b) Por argumentos de simetra, determine la direc- R
~
cin del campo elctrico E(C). ~
Calcule E(C).
~
A q
c) Determine la relacin entre y q tal que E(C) =
0
Problema 2.5 X S
Se tienen dos anillos coaxiales del mismo radio a, con-
tenidos en planos paralelos y separados entre s una dis- l R
0
tancia L. Uno de los anillos tiene densidad de carga O
uniforme + y el otro .
a) Calcule el campo elctrico en el eje comn de los
L
anillos, o sea en el eje O0 O en la figura. l
O
b) Calcule la diferencia de potencial entre los centros
O0 y O de los anillos.

Problema 2.6 X S
y
Un alambre semi-infinito cargado yace sobre el semieje
positivo x. El alambre posee una densidad lineal homo- A
gnea 0 .
a) Determine el valor del campo elctrico en el punto
A de la figura el cual est ubicado sobre el eje y a
a una distancia a del origen.
l0
b) Determine el valor del campo elctrico en el punto
a O x
B de la figura el cual est ubicado sobre el eje x B
a una distancia a del origen.
l

Problema 2.7 X S
R
Considere un alambre muy delgado como el de la figura,
ste esta compuesto por dos rectas infinitas y una arco 135 P
de circulo de 135 . El alambre tiene una densidad lineal
de carga constante. Encuentre el campo producido en
el punto P .
I. PROBLEMAS PROPUESTOS 9

Problema 2.8 X S
Dos barras delgadas e iguales de longitud L y carga total
Q distrbuida uniformemente, estn situadas sobre el eje
x, separadas una distancia d como se indica en la figura. y
a) Calcule el campo elctrico producido por la carga
de la izquierda para un punto situado sobre el eje
~
x, es decir E(x).

b) Calcule la fuerza que ejerce la carga de la izquierda +Q +Q


sobre la carga de la derecha. x
c) Pruebe que si d  L la fuerza entre las barras
L d L
equivale a la de dos cargas puntuales de carga Q.
2
Puede ser til la aproximacin ln(1 + x) x x2
si |x|  1.

Problema 2.9 X
z
Considere un plano infinito con carga superficial > 0.
El plano contiene un orificio circular de radio R en su l a
superficie.
a) Calcule el campo elctrico en cualquier punto del
d
eje z.
R
b) A lo largo del eje del orificio se coloca una lnea s
de carga de largo a, densidad lineal > 0 y cuyo
punto ms prximo se encuentra a una distancia
d del centro del orificio. Calcule la fuerza de re-
pulsin que experimenta la lnea de carga.

s = s (q )

Problema 2.10 X R
q
Calcular el campo electrosttico que genera un casquete
esfrico de centro O y radio R que porta una densidad O x
superficial de carga = 0 cos (en coordenadas esf-
ricas) en su mismo centro.
10 CAPTULO 2. DISTRIBUCIONES CONTINUAS DE CARGA

V?

Problema 2.11 X h

Considere un cono de altura h y radio basal h. La su-


perficie (manto) del cono est cargada uniformemente h
con una densidad 0 . Calcule el potencial en el centro
de la base del cono.

Problema 2.12 X
La contaminacin por compuestos qumicos de un lago
de forma circular ha dejado su densidad superficial de
carga que, expresada en coordenadas polares se puede
escribir como
z
0 a3
(r) = 3
(r2 + a2 ) 2

donde a y 0 son constantes conocidas. Aqu el origen s (r)


de coordenadas es el centro del lago. Se pide:
a) Determine el campo elctrico que afectar la vida
en el lago. Suponga que los peces slo viven en las
cercanas del centro del lago (eje z) y a profundi-
dades mucho menores que la extensin del lago, a
por lo cual ste puede considerarse infinito.

b) Suponga que debido a esta contaminacin, un pez


adquiere una carga Q. Determine el trabajo elec-
trosttico que debe efectuar el pez para llegar al
centro del lago si se encontraba nadando a una
distancia a profundidad sobre el eje z de la Fi-
gura.

Hint:
" #
d (a2 + 2r2 + z 2 ) r
p = 3
dr (a2 z 2 ) (a2 + r2 )(z 2 + r2 ) ((a2 + r2 )(z 2 + r2 )) 2

a
Problema 2.13 X
h
Calcule el campo elctrico creado por un cono macizo de
altura h y semi ngulo , uniformemente cargado con
una densidad volumtrica de carga 0 en su vrtice.
r0
I. PROBLEMAS PROPUESTOS 11

Problema 2.14 X
En una primera aproximacin, una montaa puede ser
modelada como un cono de altura h y semi ngulo de
masa total M distribuida uniformemente. Geofsicos han a a
determinado que la gravedad en su cima tiene un valor
~g1 = g1 z la cual est a una distancia h sobre el nivel z
suelo. Los mismos cientficos saben que si la montaa h
no existiese el campo gravitacional terrestre en el mismo
punto sera ~g0 = g0 z . Determine ~g = ~g1 ~g0 . (Hint:
Puede ser til el Problema 2.13)

P1
Problema 2.15 X S
h
Considere un cilindro macizo de radio R y largo L como R
se muestra en la Figura. El cilindro posee una densidad
de carga volumtrica 0 . Determine
a) El mdulo del campo elctrico en un punto P1 r0
ubicado una distancia h sobre su base superior y L
sobre el eje del cilindro. L h 2
b) El mdulo del campo elctrico en un punto P2
ubicado una distancia h > R del eje cilindro, justo P2
a la mitad de la altura del cilindro.

Problema 2.16 X
Un bloque infinito posee una densidad de carga unifor-
me que ha sido localizado entre x = a y x = a.
Este bloque es infinito en la direccin y y z (saliendo de y
pgina). Dos planos infinitos son localizados en x = b
y x = b, los cuales poseen una densidad de carga 1
y 2 , respectivamente. Al medir el campo elctrico se
observa que:
x
0 x < b b a a b


~ = +E0 x b < x < a
E

E0 x a < x < b

0 x>b s1 r s2

a) Calcule la densidad de carga del bloque infinito.

b) Calcule las densidades de carga superficiales 1


y 2 de los planos a la izquierda y derecha del
bloque respectivamente.
12 CAPTULO 2. DISTRIBUCIONES CONTINUAS DE CARGA

II. Soluciones

Solucin 2.1 P X

Todos los problemas donde se pida determinar el campo elctrico se pueden abordar mediante la defi-
nicin. Sin embargo, no en todos los problemas siempre ser fcil usar esta tcnica (ya que se pueden
llegar a integrales bastante complejas). En general en todos los problemas donde se pida determinar
el campo elctrico en un punto o en un eje (y no en todo el espacio) se pueden abordar por definicin.

Dado que este problema cumple con esa caracterstica, se debe tener claro como parametrizar la
superficie para poder usar la definicin. Sin embargo, no es claro que haya una frmula matemtica
con que se pueda representar de una sola vez a la unin casquete-disco. La parametrizacin anterior
se ve solucionada usando el principio de superposicin; primero se calcula el campo elctrico generado
por el disco y luego el del casquete semiesfrico, para luego sumar ambos resultados y obtener el
campo final en el punto.

Campo generado por el disco.

Usando definicin:

~ = 1 ~r ~r 0
E dq
40 |~r ~r 0 |3

Se debe tener claro que:

~r : Es el vector que seala dnde se quiere conocer el campo elctrico (puede ser un punto
en especfico, un eje, un superficie, etc.).
~r 0 : Es el vector que seala dnde est lo que genera el campo elctrico. Este vector es
variable y no representa un punto por s solo en distribuciones continuas de carga
ya que parametriza una lnea/superficie/volumen.
dq : Es el diferencial de carga, y puede valer dq = dl = dS = dV , eventualmente las
densidades de carga , y no necesariamente son constantes y podran depender del
vector ~r 0 (ie. depende de alguna variable de integracin).
: Es el espacio de integracin de la lnea/rea/volumen de lo que est generando el
campo elctrico. Establece los lmites de todas las integrales.

Luego, aplicado lo anterior al disco de radio a se tiene que (en coordenadas cilndricas):
a
~r = (0, 0, a) = z
2

~r 0 = rr
dq = dS
Donde los lmites de integracin sern r [0, a] , [0, 2] (qu es la parametrizacin del
disco!).
II. SOLUCIONES 13

a Trozo infinitesimal de disco


2 0
~r ~r
s ~r
~r 0
Y

Figura 2.1: Campo Elctrico generado por un disco.

Calculando la integral:


~ = 1 ~r ~r 0
E dS
40 |~r ~r 0 |3

2 a a
z rr
= a2 rdrd
40 z rr 3
0 0 2

2 a a
2 z rr
= 3 rdrd
40 (( a2 )2 + r2 ) 2
0 0
2 a a
2 z r(cos x + sen y)
= 3 rdrd
40 (( a2 )2 + r2 ) 2
0 0

~ de modo que
En este caso se separar por cada componente del vector E,

2 a
~ r cos
Ex = 3 rdrd
40 (( a2 )2 + r2 ) 2
0 0
2 a
r2
= cos d 3 dr
40 (( a2 )2 + r2 ) 2
|0 {z } 0
=0

Dado que la integral del coseno en un perodo es nula, se tiene que el campo elctrico
no tiene
~
componente segn x. Anlogamente, sucede lo mismo al momento de calcular E y , por lo que
14 CAPTULO 2. DISTRIBUCIONES CONTINUAS DE CARGA

tampoco existe una coordenada segn y. Luego

2 a a
~ = 2 z
E 3 rdrd
40 (( a2 )2 + r2 ) 2
0 0
2 a
az r
= d 3 dr
80 (( a2 )2 + r2 ) 2
0 0
a
az r
= 3 dr
40 (( a2 )2 + r2 ) 2
0
" #
d 1 r
Ntese que p a 2 = 3 por lo tanto
dr (2) + r 2 a
(( 2 ) + r2 ) 2
2

r=a
az 1
~ =
E p a 2
40 ( 2 ) + r2
r=0
" #
az 1 1
= p a 2 + a
40 ( 2 ) + a2 2
 
1
= 1 z
20 5

Campo generado por el semicasquete


Al igual que el caso anterior debe procederse a calcular los vectores ~r, ~r 0 y dq, en este caso se
tiene que (en coordenadas esfricas):
a
~r = (0, 0, a) = z
2

~r 0 = ar

dq = dS = a2 sen dd

Donde los lmites de integracin sern [0, 2] , [0, 2 ].



~ = 1 ~r ~r 0
E dS
40 |~r ~r 0 |3


2 2 a
z rr 2
= a2 a sen dd
40 z ar 3
0 0 2

2 2
a
z a(sen cos x + sen sen y + cos z) 2
= a2 a sen dd
40 z a(sen cos x + sen sen y + cos z) 3
0 0 2

2 2
a
2 z a(sen cos x + sen sen y + cos z)
= 3 a2 sen dd
40 (a2 sen2 + ( a2 a cos )2 ) 2
0 0
II. SOLUCIONES 15

En este caso vuelve a ocurrir lo mismo que pas cuando se calcul el campo que genera un
disco, ya que se anulan las componentes en x e y debido a que aparecen un cos y sen
integrados en un perodo.
En general, uno tambin puede argumentar por simetra que algunas componentes del campo
elctrico son nulas, no siempre es necesario el clculo (siempre se llegar a funciones sinuosoidales
integradas en un perodo).
Por lo tanto el valor del campo generado por el semicasquete es:

2 2 a
~ = z a cos
E 2
3 a2 sen dd
40 (a2 sen2 + ( a2 a cos )2 ) 2
0 0

2 2
a2 z ( a2 a cos ) sen
= d 3 d
40 (a2 sen2 + ( a2 a cos )2 ) 2
0 0
=
a2 z a a2 cos 2

= 2 a 2 p a 2
40 ( 2 ) ( 2 ) 2 2 a cos + a
a 2
=0
!
a
2z a
= p a 2 p 2a 2
0 ( 2 ) + a2 (2)
 
2 2
= 1 z
0 5

Dado los resultados anteriores, se procede a sumarlos para encontrar el campo final en el punto
     
~ total = 1 2 2 7
E 1 z + 1 z = 3 z
20 5 0 5 20 5

Solucin 2.2 P X

Para encontrar el ngulo de equilibrio, se debe considerar que la suma de fuerzas que actan sobre la
masa m deben anularse, por consiguiente se hace un DCL para deducir las ecuaciones de movimiento.

l
~g

q
l
~T
~Fe

m~g

Figura 2.2: DCL para masa m.


16 CAPTULO 2. DISTRIBUCIONES CONTINUAS DE CARGA

Con lo anterior las ecuaciones de movimiento son:


mx = Fe T sin = 0
mg
my = T cos mg = 0 T =
cos
Como F~e = q E,
~ se calcular el campo E ~ a una distancia d = l sin producido por la barra infinita
de densidad de carga lineal . Considerando que

~ 1 ~r ~r 0
E= dq
40 |~r ~r 0 |3
l

Donde ~r = dx = l sin x correspondiente a la posicin donde se quiere calcular el campo, ~r 0 = z z


correspondiente a quien genera el campo, y dq = dz dnde z (, +). Adems se considera
que por la simetra del problema, el campo ser el mismo independiente de la posicin en z de la
masa, por lo que E~ slo tiene componente en x. Con lo anterior la ecuacin anterior queda:

+
~ = 1
E
l sin x
3 dz
40 ((l sin )2 + z 2 ) 2


+
l sin dz
= x 3
40 ((l sin )2 + z 2 ) 2

Se hace el cambio de variable z = (l sin ) tan , lo cual implica que dz = (l sin ) sec2 d, y los
lmites de integracin cambian de z (, +) ( 2 , + 2 ). Luego:


+ 2
~ = l sin x (l sin ) sec2 d
E 3
40 ((l sin )2 + (l sin )2 tan2 ) 2
2

+ 2
l sin (l sin ) sec2 d
= x 3
40 (l sin )3 (1 + tan2 ) 2
2

+ 2
sec2 d
= x
40 (l sin ) sec3
2

+ 2

= x cos d
40 (l sin )
2

= x sin |2
40 (l sin ) 2


= x
20 (l sin )

Como ya se conoce el campo, luego F~e = q x. Dicho valor en las ecuaciones de movimiento
20 (l sin )
y despejando T , se llega a:
mg sin2 q
mx = Fe T sin = q sin = 0 = = = qK
20 (l sin ) cos cos 20 lmg
II. SOLUCIONES 17

Se usa la identidad trigonomtrica sin2 = 1 cos2 :


p
1 cos2 2 qK (qK )2 + 4
= qK = cos + qK cos 1 = 0 = cos =
cos 2
Como se espera que (0, /2) (el pndulo no dar la vuelta
p completa), cos > 0, luego se escoge
la solucin positiva. Finalmente si q , se cumple que (qK)2 + 4 qK, entonces
qK + qK
cos = =0
2
El resultado anterior implica que = 2 , esto es esperable ya que al ser una carga positiva se alejar
lo ms posible de quin genere el campo y eso de logra a la mayor distancia, es decir, con el ngulo
encontrado.

Solucin 2.3 P X

La fuerza sobre la carga q es F~q = F~e + F~r donde Fe corresponde a la fuerza que el anillo ejerce sobre
q, y Fr , la fuerza que ejerce el resorte.

La fuerza que ejerce el campo elctrico que genera el anillo sobre la carga q est dada por

F~e = q E
~A

Entonces el campo elctrico que genera el anillo en su eje es:



~ 1 ~r ~r 0
EA = dq
40 |~r r~0 |3

Donde ~r = z z, ~r 0 = R0 r (en coordenadas cilndricas) y dq = R0 d con (0, 2). Adems, dado


que la carga total en e anillo es Q y se distribuye homogneamente, se puede calcular como:
Q Q
= =
L 2R0
con lo anterior:
2
2 2
~A = 1 z z R0 r 1 z z R0 r
E 3 R0 d = 3 R0 d 3 R0 d
40 (z 2 + R2 ) 2 40 (z 2 + R2 ) 2 (z 2 + R2 ) 2
0 0 0 0 0 0

Ntese que por la simetra del anillo la segunda integral se anular. Tambin se puede justificar
notando que r = sin x + cos y, y como la integral de sin y cos se anulan en un intervalo entre
(0, 2), el segundo sumando se anula. Luego:

2
~A = R0 z z R0 z z
E 3 d = 3
40 (z 2 + R02 ) 2 20 (z 2 + R02 ) 2
0

Finalmente, sustituyendo , se concluye el valor de F~e :


Q R0 z z qQz z
F~e = q E~A = q ~
3 = Fe = 3
2R0 20 (z 2 + R2 ) 2 40 (z 2 + R02 ) 2
0
18 CAPTULO 2. DISTRIBUCIONES CONTINUAS DE CARGA

Ntese que como q < 0 esta fuerza apunta en en la direccin z.

Por otro lado, la fuerza elstica para una distancia z (notar que el resorte se mueve slo en el eje),
es la que genera el resorte de constante k0 , la cual est dada por

F~r = k0 z z

Dado los clculos anteriores, la fuerza total sobre q es:


!
qQ
F~q = 3 k0 z z
40 (z 2 + R02 ) 2

Por otra parte, la fuerza en el eje z esta dada por:

F~q = mz z

Si se desea encontrar la frecuencia para pequeas oscilaciones, consideramos que los valores que toma
z  1, con lo que z 2 0. Finalmente:
   
qQ 1 qQ
mz = k 0 z = z + + k0 z =0
40 R03 m 40 R03

y como la ecuacin de la frecuencia en pequeas oscilaciones cumple z + 2 z = 0


s  
1 qQ
= + k0
m 40 R03

Solucin 2.4 P X

a) Es necesario calcular el potencial generado por la semicircunferencia y el creado por la carga


puntual. Para el caso de la semicircunferencia se puede calcular usando la expresin

1 dq
V (~r) =
40 |~r ~r 0 |

en este caso ~r = 0 y ~r 0 = Rr con [0, ] y dq = dl = Rd. Luego


Rd
Valambre (0) = =
40 | Rr| 40
0

Adicionalmente es sabido que el potencial que genera una carga puntual esta dado por
q
Vcarga (0) =
40 R

Por lo tanto el potencial pedido es la superposicin de los resultados anteriores

q
V (C) = +
40 R 40
II. SOLUCIONES 19

b) Por simetra las componentes en horizontales se anulan en el punto C, por ende en ese punto
slo existe la superposicin de de componentes verticales. El campo generado por el alambre
vale
1 (~r ~r 0 )
E(~r) = dq
40 |~r ~r 0 |3
nuevamente se tiene que ~r = 0 y ~r 0 = Rr con [0, ] y dq = dl = Rd, por lo que

~ alambre (0) = (0 Rr)Rd
E 3
= [cos x + sin y] d = y
40 | Rr| 40 R 20 R
0 0

al igual que la parte anterior el campo elctrico generado por la carga est dado por

~ carga (0) = q
E y
40 R2
Finalmente el campo total es
 
~ q
E(C) = 2
y
40 R 20 R

~
c) Imponiendo E(C) = 0, se tiene que

q
2
= = q = 2R
40 R 20 R

Solucin 2.5 P X

a) Inicialmente determinaremos el potencial que genera un anillo por si slo y luego se usar el
hecho que E ~ = V para determinar el campo elctrico. De este modo el potencial de un slo
anillo de radio a y densidad homognea y con su centro en el origen provoca un potencial
dado por:
1 dq
V (~r) =
40 |~r ~r 0 |
donde ~r = z z, ~r 0 = ar y dq = ad con [0, 2]. Por ende

2
1 ad a
V (z) = =
40 2
z +a 2 20 z 2 + a2
0

Extrapolando este resultado al problema se tiene que los anillos estn a distancia L, de modo
que suponiendo que el origen est en el punto de medio de su separacin, el potencial en el eje
es

a a a 1 1
V (z) = q  q  = q  q 
L 2 2 L 2 2 20 L 2 2 L 2 2
20 z 2 + a 20 z+ 2 +a z 2 +a z+ 2 +a

Dado que
~ = V = V z
E
z
20 CAPTULO 2. DISTRIBUCIONES CONTINUAS DE CARGA

Por lo que finalmente


~ = a z L2 z + L2
E  3 z
20   2
 3
2
  2 2
z L2 + a2 z + L2 + a2

b) Considerando lo obtenido en a)
   
L a 1 1
V (O) = V z = =
2 20 a L2 + a2
   
0 L a 1 1
V (O ) = V z = =
2 20 L2 + a 2 a
Por lo que la diferencia vale:
 
a 0 1 1
V (O) V (O ) =
0 a L + a2
2

Solucin 2.6 P X

a) Recordando, que el campo elctrico producido por una distribucin de carga dq(~r 0 ) vale:

~ r) = 1 ~r ~r 0
E(~ dq(~r 0 )
40 |~r ~r 0 |3

Luego se tiene que: ~r = ay; ~r 0 = xx; dq = dx, por lo tanto:




~ y adx xdx
E(ay) = 3 x 3
40 (a2 + x2 ) 2 (a2 + x2 ) 2
0 0

Realizando el cambio de variable:

x = a tan = dx = a sec2 d

Resulta:

2 2
~ a2 sec2 d a2 tan sec2 d
E(ay) = y x
40 a3 sec3 a3 sec3
0 0


2 2
~
E(ay) = y cos d x sin d
40 a
0 0

Obteniendo, finalmente que:


~
E(ay) = (y x)
40 a
II. SOLUCIONES 21

b) Nuevamente usando la definicin se tiene que ~r = ax; ~r 0 = xx; dq = dx




~ x a x
E(ax) = dx
40 | a x|3
0

Manejando algebraicamente la expresin se obtiene que



~ dx 1
E(ax) = = x = x
40 (x + a)2 40 x + a 0 40 a
0

Solucin 2.7 P X

Separando el problema en dos partes:

i) Campo producido por las rectas semi-infinitas: Sean (x0 ; y 0 ) el sistema de referencia para la
recta superior en la figura, y (x00 ; y 00 ) para la recta inferior, orientados de manera que coincidan
con el sistema de referencia de la parte a) entonces:

~1 = ~2 =
E (x0 + y 0 ) E (x00 + y 00 )
40 R 40 R

Ahora descomponiendo en los ejes x e y (x horizontal hacia la derecha, y vertical hacia arriba):

x0 = cos x + sin y
8 8
3 3
y 0 = cos x sin y
8 8

x00 = cos x sin y
8 8
3 3
y 00 = cos x + sin y
8 8
Como cos 3
8 = sin 8 , y superponiendo los campos, entonces:
    
~ 1 +E
~2 = 3  3
E x cos + cos = x cos + sin = x 2 cos
20 R 8 8 20 R 8 8 20 R 8

Otra forma mucho ms rpida de resolver esta parte es la siguiente:



~ 1 | = |E
~ 2| = 2
|E
40 R

Entonces, debido a la simetra con respecto al eje x, las componentes en y de los campos
producidos por las dos rectas se anulan, y se obtiene que:

~1 + E
E ~ 1 | cos 3 = x cos 3
~ 2 = 2|E
8 20 R 8
22 CAPTULO 2. DISTRIBUCIONES CONTINUAS DE CARGA

ii) Campo producido por el sector circular: Calculando por definicin, de manera similar a la parte
anterior:
~ 1 ~r ~r 0
E3 (~r) = dq(~r 0 )
40 |~r ~r 0 |3

Con: ~r = 0 ; ~r 0 = Rr ; dq = Rd. Entonces:


11
8
~ 3 (0) =
E rd
40 R
5
8

Descomponiendo r en los ejes cartesianos, y por la paridad e imparidad del coseno y seno,
la integral en y se anula (tambin se puede comprobar esto ltimo debido a la simetra con
respecto a x) resulta:
11
8 3

~ 3 (0) = x 8
E x cos d = cos d
40 R 20 R 0
5
8

Luego:  
~ 3 (0) = x sin 3 = x cos = x
E sin
5
sin
11
20 R 8 20 R 8 40 R 8 8

Finalmente, sumando todos los campos, se obtiene que el campo elctrico total en el punto P vale:
!
~T = x 1 3 cos 3
x  
E sin 8 = cos 2 sin
0 R 2 8 2 20 R 8 8

Solucin 2.8 P X

a) Primero debe usarse el campo elctrico por definicin para hallar una expresin

1 (~r ~r 0 )dq
E(~r) =
40 |~r ~r 0 |3
Q
En la frmula anterior ~r = xx, ~r 0 = x0 x y dq = dx0 con x0 [0, L] donde = L.
Analizando por intervalos

x>L
L L  
~ x x x0 x dx0 x 1 L 1 1
E(x) = dx0 = = = x
40 0
(x x )3 40 (x x0 )2 40 x x0 0 40 x L x
0 0

x<0
Anlogo a lo anterior:  
~ =
E
1

1
x
40 xL x
II. SOLUCIONES 23

0xL
En este caso el campo elctrico es infinito ya se cae en un punto de carga infinita. Esto
puede verse en la integral, ya que si x = x0 diverge.

b) En este caso un diferencial de fuerza est dado por dF~ = E(x)dq


~ ~
= E(x)dx , por lo tanto

2L+d        
2 x 1 1 2 x L+d 2L + d 2 (L + d)2
F~ = dx = ln ln = ln x
40 xL x 40 d L+d 40 d(2L + d)
L+d

c) Para la ltima parte debe notarse lo siguiente


   2     
(L + d)2 L + 2Ld + d2 L2 L2 L2
ln = ln = ln 1 + ln 1 +
d(2L + d) 2Ld + d2 2Ld + d2 d2 d2
De lo anterior se termina deduciendo que

1 (L)2 1 Q2
F~ x = x
40 d2 40 d2
Que coincide con la fuerza que sienten dos cargas puntuales de carga Q = L.

Solucin 2.15 P X

a) En este caso, se usar el resultado conocido del campo elctrico que genera un disco de radio
R y densidad de carga , el cual corresponde a
 
~ z
E= 1 z
20 z 2 + R2
Ahora, usando este resultado se considerar z = 0 en el punto P1 creciendo hacia arriba. A partir
de esto, el cilindro est compuesto por la superposicin infinitesimal de discos (uno encima del
otro). El aporte de campo elctrico dE~ que provoca uno de estos discos de ancho infinitesimal
es  
~ 0 dz z
dE = 1 z
20 z 2 + R2
Por lo que el campo total en el punto est dado por
h  
~ 1 ) = 0 z z 0 h p p i
E(P 1 dz = L ( h2 + R2 (L + h)2 + R2 ) z
20 z + R2
2 20
(L+h)

b) Para resolver este problema, se usar una tcnica de analogas. Este problema es un poco
difcil planteando las integrales correspondientes, pero se puede usar otro resultado conocido
como el que genera una barra de densidad lineal 0 de largo L sobre un punto ubicado un eje
perpendicular a ella justo en su mitad.
En este caso, tomando el eje z coincidente con eje de la barra y el origen su punto medio, se
obtiene ~r = hr, ~r 0 = z z con z [ L2 , L2 ] y dq = 0 dz. Entonces
L
2
1 hr z z L
~ 2) =
E(P 3 0 dz = 0 r
40 (h2 + z2) 2 20 h 4h2 + L2
L
2
24 CAPTULO 2. DISTRIBUCIONES CONTINUAS DE CARGA

l0
L
2
L h
P2

Figura 2.3: Situacin anloga al cilindro.

Ahora, volviendo al problema, si V es el volumen de un cilindro es posible afirmar que


dq dq dV dV d
0 = = = 0 = 0 (R2 z) = 0 R2
dz dV dz dz dz
El valor obtenido es equivalente a la carga por unidad de largo que tiene el cilindro del problema.
Finalmente el campo es
~ 2) = 0 R2 L
E(P r
20 h 4h2 + L2
Ley de Gauss
3
I. Problemas Propuestos

Problema 3.1 X S
Una distribucin de carga esfrica se extiende desde
r = 0 a r = R, con
 
r2
= 0 1 2
R R
Calcular:
a) La carga total Q.

b) El campo elctrico en todo el espacio.

c) El potencial elctrico en todo el espacio.

Problema 3.2 X
R1
Considere un cable coaxial infinito y rectilneo, el cual R2
est compuesto por un cilindro central y diferentes cas- R3
quetes cilndricos, de radios R1 , R2 , R3 y R4 respectiva- R4
mente, como se ilustra en la figura. Cada material tiene
R1,1=0
respectivamente una densidad de carga volumtrica 1 ,
R2,2
2 , 3 y 4 (Ver Figura). En el caso que el cilindro y R3,3=0
segundo casquete cilndrico (de radio R3 ) tienen densi- R4,4
dad de carga cero (1 = 3 = 0). Encuentre el campo
elctrico en todo el espacio.

Problema 3.3 X
Considere la siguiente distribucin de volumtrica de car-
ga en coordenadas esfricas a
 r nr
n 0 < r < a
(r) =
arb b
Donde es una constante y n es un entero no negativo.
Encuentre el campo elctrico en todo el espacio.

25
26 CAPTULO 3. LEY DE GAUSS

Problema 3.4 X
Se tiene una placa infinita no conductora de espesor
despreciable la cual posee una densidad superficial de
carga , y continua a ella, un bloque infinito de espesor
s
D con una densidad de carga uniforme +. Todas las
cargas estn fijas. Calcule la direccin y la magnitud del
campo elctrico:
+r D
a) a una distancia h encima de la placa cargada ne-
gativamente.

b) dentro del bloque a una distancia d debajo de la


placa cargada negativamente (d < D).

c) a una distancia H bajo fondo del bloque.

Problema 3.5 X
Considere dos placas paralelas cargadas con densidades
s
+ y de ancho d como muestra en la figura. Se
arroja una carga +q y masa m horizontalmente por el
espacio entre las placas con una velocidad vx . Despre- vx x +s
ciando efectos de borde, encuentre la trayectoria seguida
por la partcula cargada y el ngulo que hace su vector
d
de velocidad con la horizontal en el momento de salir.
Asuma que la separacin de las placas es suficiente para
que la carga pueda salir de ellas.
z

Problema 3.6 X a

Considere una esfera maciza de radio R y carga Q. De- a


termine el flujo de campo elctrico sobre el cuadrado de
lado a mostrado en la Figura. y
a

Problema 3.7 X a
O0
Considere una esfera maciza de densidad de carga 0 y
radio R la cual posee una perforacin esfrica de radio R d~
a < R2 a una distancia de d entre sus centros. Demuestre O
que el campo elctrico es constante en cualquier punto
dentro de la cavidad y determine su valor.
I. PROBLEMAS PROPUESTOS 27

r y +r
(a)
Problema 3.8 X S
R0
Considere dos cilindros infinitos de radios R0 los cuales
poseen sus ejes paralelos al eje z (entran y salen de la x0 x0 x
hoja de papel). Las densidades de carga volumtricas de
los cilindros son y y sus ejes centrales pasan por
los puntos (x0 , 0) y (x0 , 0), respectivamente. Consi-
derando que x0 < R0 . y
s (q )
a) Determine el campo elctrico en la zona de inter- (b)
seccin.
q
b) Si x0  R0 , ambos cilindros quedan infinitesimal-
x
mente cerca, creando un nico cilindro equivalen-
te de radio R0 con una densidad de superficial de
carga (). Encuentre el valor de esa densidad.
Cilindro Equivalente (x0 R0 )

Problema 3.9 X
Dentro de una esfera de radio a centrada en el origen
hay un campo elctrico
 2
r(r)? a
~ a) = E0 r r
E(r
0 a
Para r > a hay vaco. Se pide determinar
a) La distribucin de carga (r) para r a.
~ y el potencial elctrico para r > a
b) El campo E

c) El potencial elctrico V (r < a).

z
Problema 3.10 X S
Un cilindro infinito de radio R tiene su eje coincidente
con el eje z. El cilindro posee una densidad volumtrica
(r) = ar donde a es una constante positiva y r es la
distancia desde el eje del cilindro.
a) Calcule la carga contenida en un cilindro centrado
en el eje z, de radio r y altura h para los casos r
r(r) =
r < R y r > R. a

~
b) Determinar el campo elctrico E(r) en todo el
espacio.

c) Calcular el potencial elctrico V (r) en todo el es-


pacio. Tome como referencia V (r = 0) = 0.
~
d) Grafique |E(r)| y V (r) en funcin de r.
R
28 CAPTULO 3. LEY DE GAUSS

Problema 3.11 X
Considere un cable infinito cargado con una densidad l0
lineal de carga 0 rodeada por un casquete cilndrico
infinito de radio R de densidad superficial homognea
0 . Si la densidad lineal coincide con el eje del cilindro,
determine:
a) El campo elctrico en todo el espacio, es conti-
nuo el campo elctrico?.
s0
b) El potencial elctrico en todo el espacio, es con-
tinuo el potencial elctrico?. (Use como referencia
V (r = R) = 0)

c) Si el alambre se desplazara una distancia del eje


del cilindro, cmo determinara el nuevo valor del R
campo elctrico?.

Problema 3.12 X
Use el teorema de Gauss para encontrar el campo elc-
z r(z)
trico debido a una distribucin de carga

= 0 ek|z|

con 0 y k constantes positivas.


a) Muestre que el campo es de la forma E ~ =
(0, 0, E(z)), con E(z) = E(z) para z > 0.

b) Encuentre el campo elctrico en todo el espacio.


l

P
Q
Problema 3.13 X S a h
a
s
Se tiene una fuente cargada que consiste en una recta
infinita cargada, con densidad uniforme y un plano
infinito cargado con densidad de carga uniforme . La
recta forma un ngulo agudo 2 con el plano. Considere
(a) Vista Isomtrica
un punto P est a una altura h sobre el plano. Determine
a) El campo elctrico total en un punto P sobre la
recta que bisecta al ngulo entre la recta y el
plano.

b) El trabajo necesario para mover una carga puntual P


q0 desde el punto P hasta el punto Q el cual est Q s
ubicado a una distancia h2 sobre el plano. a
a
h h
2

(b) Vista Frontal


II. SOLUCIONES 29

II. Soluciones

Solucin 3.1 P X

a) La carga total en el tomo est dada por



Q= (r)dV

2 R  
r2
= 0 1 2 r2 sin drdd
R
0 0 0
R  
r4 2
= 40 r 2 dr
R
0
 3 
R R3
= 40
3 5
8
= 0 R3
15
En general Q 6= 34 a3 (r) (hacer esto es un error muy comn en el curso). Esto es verdad
solamente cuando (r) = cte.
b) El campo elctrico est dado por Ley de Gauss, para ello se deben considerar dos posibles casos
(dentro y fuera de la esfera). En ambos casos se asume por simetra esfrica que E~ = E(r)r.

r R r R

W1 W2

Figura 3.1: Superficie Gaussiana 1 para r R Figura 3.2: Superficie Gaussiana 2 para r < R

r R: En este caso se toma como superficie Gaussiana 1 , la cual representa un casquete


esfrico de radio r > R.

~ = qenc
~ dS
E
0
1
2 R  
2 1 r2
4r E(r) = 0 1 2 r2 sin drdd
0 R
0 0 0

~ 20 R3
E(r) = r
150 r2
30 CAPTULO 3. LEY DE GAUSS

r < R: En este caso se toma como superficie Gaussiana 2 , la cual representa un casquete
esfrico de radio r < R.

~ = qenc
~ dS
E
0
2
2 r  
1 r2
4r2 E(r) = 0 1 2 r2 sin drdd
0 R
0 0 0
 
2 40 r3 r 5
4r E(r) =
0 3 5R2
 
~ 0 r r3
E(r) = r
0 3 5R2
Notar que dado que r es arbitrario, el lado de izquierdo de la ecuacin de ley de Gauss valdr
siempre 4r2 en problemas de simetra esfrica.
c) Mientras que el potencial (considerando V (r ) = 0)

rR
r
V (r) = E(r R)dr

r
20 R3
= dr
150 r2

20 R3
=
150 r
r<R
R r
V (r) = E(r R)dr E(r < R)dr
R
R r
 
20 R3 0 r r3
= dr dr
150 r2 0 3 5R2
R
 
20 R2 0 1 2 2 1 4 4
= (r R ) (r R )
150 0 6 20R2

Solucin 3.8 P X

a) Primero se debe calcular el campo elctrico producido por un cilindro infinito cargado (radio R,
densidad constante) centrado en el origen. Usando Ley de Gauss:

~ = E(r)r en coordenadas cilndricas, luego:


Dada la geometra del problema: E

Para r < R:
2 h
~ dS
~= Q
E Erddz = 2rhE =
0
0 0
II. SOLUCIONES 31

Por otro lado



r2 h ~ < R) = r r
Q= dV = r2 h = 2rhE = = E(r
0 20

El caso r > R es irrelevante para esta pregunta, luego no es necesario calcularlo ya que la zona
donde se pide calcular el campo es tal que r < R siempre.

r y +r

~r1 ~r2
2x0 x x

Figura 3.3: Vectores posicin en el rea de interseccin

~1 y E
Ahora volviendo al problema original, sean E ~ 2 los campos producidos por los cilindros
cargados con y + respectivamente, usando el resultado anteriormente encontrado:

~ 1 = r1 r1 = ~r1
E
20 20
~ 2 = r2 r2 = ~r2
E
20 20
Donde los vectores ~r1 y ~r2 se miden desde el centro del cilindro correspondiente a cada uno de
ellos (Figura 3.3). Finalmente, usando que:

~r1 ~r2 = 2x0 x

Se obtiene:
~ =E
E ~ 2 = (~r2 ~r1 ) = x0 x
~1 + E
20 0
b) Si x0  R entonces en el borde del Cilindro equivalente posee un espesor r = r1 r2 .
Donde los segmentos de r1 , r2 y 2x0 forman un tringulo, cuyo ngulo opuesto al lado r2 es .
As, usando el teorema del coseno (y, por simplicidad, definiendo d = 2x0 ) :
q
r2 = r12 + d2 2r1 d cos

d
Luego como x0 0 entonces se realiza una aproximacin de Taylor en torno a d = 0, de lo
que resulta:

1
r2 r1 + p 2 (2d 2r1 cos )|d=0 (d 0) = r1 d cos
2
2 r1 + d 2r1 d cos
Por lo tanto:
r = r1 r2 d cos
32 CAPTULO 3. LEY DE GAUSS

Por otra parte, en un pequeo volumen dV en el borde del Cilindro equivalente existe una
carga Q, que cumple:
Q = dV = Ar = Ad cos
Finalmente, de la relacin anterior es posible definir una densidad de carga superficial en el
borde del Cilindro equivalente cuando x0  R:

= r = d cos = 2x0 cos


Nota: observar que dicha densidad es consistente en sus signo con la distribucin de las cargas
en el Cilindro equivalente pues es proporcional a cos .

Solucin 3.10 P X

a) La carga contenida ser para r R:


h 2 r
r 2hr3
Qenc (r R) = (r)dV = rdrddz =
a 3a
0 0 0

mientras tanto, en el caso r > R


h 2 R
r 2hR3
Qenc (r > R) = (r)dV = rdrddz =
a 3a
0 0 0

b) Usando la ley de Gauss para r R:


3
~ = Qenc = E(r) 2rh = 2hr = E(r)
~ dS ~ r2
E = r
0 3a0 3a0
De igual forma, para r > R:
3
~ = Qenc = E(r) 2rh = 2hR = E(r)
~ dS ~ R3
E = r
0 3a0 3a0 r

c) El potencial V (r), usando como referencia V (r = 0) = 0 , estar dado por


r
V (r) = V (r) V (0) = E(r)dr
0

Para un radio r R el potencial est dado por


r
r2 r3
V (r) = dr =
3a0 9a0
0

de forma parecida, el potencial para un radio r > R es


R r    
r2 R3 R3 r 1
V (r) = dr dr = ln +
3a0 3a0 r 3a0 R 3
0 R

d) Los grficos estn dados por


II. SOLUCIONES 33

|~E(r)| V (r)
R
r
R2
3ae0
R3
9ae0

R r
Figura 3.4: Campo Elctrico Figura 3.5: Potencial Elctrico

Solucin 3.13 P X

a) Dado que el sistema est compuesto por una recta infinita y un plano infinito, se determina
cual es el campo elctrico que genera cada uno de ellos y luego se proceder a la rotacin del
resultado. Como resultados generales para un plano infinito y una recta se tiene que:

E ~ = Qenc = 2AE(z) = A = E(z)
~ dS ~ =

z
0 0 20


E ~ = Qenc = 2rLE(r) = L = E(r)
~ dS ~ =

r
0 0 2r0

En este problema se debe tomar como referencia la siguiente Figura

l
y

~E1
y
~E2
2a
a y
a s
x

Figura 3.6: Campos Elctricos en un punto sobre la bisectriz

Se toma un punto P arbitrario a una distancia y desde la bisectriz hasta el plano. Sobre ese punto
actan dos campos elctricos (E ~1 y E
~ 2 provocados por el plano y por la recta, respectivamente),
de modo que la superposicin de ambos campos es el campo total en el punto. Usando el sistema
de referencia de la Figura 3.6, el campo total vale:

~ T = |E
E ~ 1 |y |E
~ 2 | cos(2)y + |E
~ 2 | sin(2)x
34 CAPTULO 3. LEY DE GAUSS

Donde
~ 1| = ~ 2| =
|E , |E
20 2y0
Ntese que dado la recta donde se quiere conocer el campo es bisectriz, la distancia del plano
al punto debe ser igual a la de la recta cargada a la bisectriz (congruencia de tringulos).
Finalmente el campo total cuando y = h vale
 
~ sin (2) cos (2)
ET (h) = x + y
2h0 20 2h0

b) El trabajo necesario para mover la carga est dado por la siguiente expresin

B
W = q E ~
~ T (y) dl
A

En esta caso A = h y B = h2 , adems dado que no es ni un camino horizontal ni vertical, se


~ = dxx + dy y, donde en la recta bisectriz se cumple que dy = tan (pendiente de
tiene que dl dx
la bisectriz). Reemplazando

h
2      
sin (2) cos (2) dy
W = q x + y x + dy y
2y0 20 2y0 tan
h
h h
2 2  
sin (2) dy cos (2)
= q + dy
2y0 tan 20 2y0
h h
  
sin (2) h cos (2)
=q ln(2) + ln(2)
20 tan 40 20
 
qh q ln(2) sin 2
= + cos 2
40 20 tan
Conductores, Condensadores y Energa
4
I. Problemas Propuestos

Problema 4.1 X S

a) Calcule la fuerza elctrica que acta sobre las pla- Q


cas de un condensador de placas planas, cargado
con carga Q.
+Q
b) Considere que la carga Q sobre las placas del con-
densador se mantiene y que su capacidad es C.
Calcule el trabajo que se realiza al llevar las placas
a la mitad de la distancia original, manteniendo
la carga constante. Puede suponer que las placas
poseen una rea A muy grande.

c) Este nuevo condensador se conecta en paralelo


con otro condensador inicialmente descargado e
igual al condensador de la parte (a). Calcule la
diferencia de potencial entre las placas del con-
densador equivalente.
l

Problema 4.2 X S R

Un alambre infinito tiene una distribucin lineal de carga


> 0. El alambre se encuentra ubicado en el centro de
una superficie cilndrica conductora infinita muy delgada
de radio R conectada a tierra como se muestra en la
Figura.
a) Encuentre la densidad de carga superficial induci-
da en la superficie interior conductora.

b) Encuentre el campo elctrico en todo el espacio.

c) Encuentre el potencial elctrico en todo el espacio.

35
36 CAPTULO 4. CONDUCTORES, CONDENSADORES Y ENERGA

Problema 4.3 X S
Se desea disear un condensador esfrico a partir de a
un casquete conductor esfrico de radio exterior a, que
sea capaz de almacenar la mayor cantidad de energa
posible, sujeto a la restriccin que el campo elctrico en
la superficie de la esfera conductora interior, concntrica b
con el casquete y de radio b < a, no pueda exceder un
valor dado E0 . Calcule, en funcin de E0 , a y constantes,
el valor que debe tener el radio b y la magnitud de la
energa que puede almacenar el conductor.

Problema 4.4 X S
R1
Se tienen dos esferas conductoras de radio R1 y R2
separadas entre si una distancia suficientemente grande
que asegura que cualquier carga sobre ellas se distribuye

s
jo
uniformemente, sin que la presencia de una esfera afecte

Le
uy
a la otra. Se desea distribuir una carga Q entre las dos
M R2
esfera de manera que la energa potencial electrosttica
del sistema de las dos esferas sea mnima. Claramente,
en una esfera habr Qq y en la otra q. Cunto vale q,
cul es energa total y cul es el potencial de cada esfera
cundo se alcanza la condicin de energa mnima?.

Problema 4.5 X
Una esfera metlica se encuentra inicialmente descarga-
da. Ahora imagine que una carga positiva q es colocada
en algn lugar (no necesariamente el centro) dentro de
la esfera y sin tocar las paredes.
Conductor
a) Qu carga se induce en la pared interior y exterior
de la esfera?. Indicar cualitativamente la concen-
tracin de densidad de carga inducida.
q
b) Suponga que se mueve la carga q dentro de la
cavidad. Cambia la distribucin en la superficie
exterior de la esfera?.

c) Ahora se coloca una carga q en contacto con la


superficie interior de la esfera. Cmo queda la
distribucin de carga en la superficie interior y ex-
terior?.

d) Qu sucede si ahora se acerca otra carga q 0 cerca


de la superficie exterior del conductor?.
I. PROBLEMAS PROPUESTOS 37

Problema 4.6 X S
Un in es acelerado desde el reposo hasta una diferencia
de potencial V0 para luego entrar en una regin entre r0 a
dos electrodos cilndricos muy largos A y B, de radios b
a y b respectivamente (a < b). El in recorre una media
B A
circunferencia de radio r0 haciendo una trayectoria cir- V =0
cular. Despreciando efectos de borde y asumiendo que V = VO
los cilindros son muy largos en comparacin al espa-
cio que los separa, encuentre la diferencia de potencial
VBA . Fuente de Iones

Problema 4.7 X
Uno de los primeros modelos de tomo, como un ente
compuesto de partes cargadas, lo propuso el descubridor
del electrn Joseph John Thomson en 1904. Este mo-
delo, tambin conocido como el modelo del pastel de
fresas, concibe al tomo como una esfera de carga po- +
-
sitiva, en la cual estn incrustados los electrones. En el R2
espritu del modelo del pastel de fresas, modelemos un
tomo de hidrogeno (en equilibrio esttico) como una R1
esfera de radio R1 de carga negativa e uniformemente
distribuida en su volumen (el electrn fresa), rodeada
de una esfera ms grande (concntrica a la primera), de
radio R2 > R1 , con carga positiva +e uniformemente
distribuida en el volumen comprendido entre R1 y R2 .
Determine la energa de formacin de este tomo (i.e.
el trabajo necesario para formarlo trayendo las cargas
desde el infinito).

Problema 4.8 X S
Considere una esfera maciza conductora de radio a se
encuentra a un potencial V0 en toda su superficie con
respecto al infinito. La esfera esta recubierta por un cas-
a c
quete esfrico conductor de radio interno b y radio ex-
terno c.
a) Determine el campo elctrico y el potencial elc- b
trico en todo el espacio. Adems encuentre las
densidades de carga inducidas en los conductores.

b) Si el casquete esfrico se conecta a tierra, cmo


cambian sus respuestas anteriores?.
38 CAPTULO 4. CONDUCTORES, CONDENSADORES Y ENERGA

Problema 4.9 X S
Considere una esfera maciza conductora de radio R con R
una burbuja esfrica excntrica de radio c. El centro de c
la burbuja est a una distancia b del centro de la esfera b
metlica. La esfera tiene una carga Q. Determine el po-
tencial en el interior de la burbuja. Cmo se modifica
el resultado si la burbuja no es esfrica?

Problema 4.10 X
Considere dos esferas conductoras de radios a y b. Las
esferas estn lo suficientemente lejos una de otra como
para despreciar su interaccin, (i.e. el equilibrio electro- a
esttico de una esfera no se ve afectado por el campo
que genera la carga contenida en la otra).
a) Suponga que las esferas tienen cargas Q1 y Q2 ,
respectivamente. Las esferas se ponen en contac-
to mediante un cable lo suficientemente largo, el
cual posee un interruptor. Se conectan las dos es-
feras y se espera hasta que el sistema alcance el Interruptor
equilibrio electroesttico, para luego desconectar
el interruptor. Determine la carga que posee ca-
da esfera luego que se desconecta el interruptor.
Qu esfera queda con mayor carga?.

b) Considere ahora que las esferas estn descargadas


y desconectadas. Suponga ahora que la distan-
cia que separa las esferas es d  a, b desde sus b
centros. Considerando que dos conductores cua-
lesquiera pueden formar un capacitor, determine
la capacitancia de esta configuracin.

A B

Problema 4.11 X
Considere dos condensadores cilndricos como se indican
en la figura. Los condensadores tienen radios R1 y R2 L
y el otro R3 y R4 (ver figura), determine la capacitan-
cia equivalente entre los puntos A y B. Suponga que
R1 , R2 , R3 , R4  L.
R2 R4
R1 R3
I. PROBLEMAS PROPUESTOS 39

Problema 4.12 X S
Un sistema consiste de dos cascarones conductores ci-
lndricos concntricos de longitud L  d (a, b, c, d
definidos en la figura). El cascarn interior contiene una
carga total +Q y el exterior una carga total Q. Deter- L
mine: (a) Vista Exterior

a) La densidad carga en cada una de las cuatro su-


perficies conductoras.

b) El potencial en todo el espacio. a


d b
c) La diferencia de potencial de los conductores ci-
lndricos. c

(b) Vista Interior

Problema 4.13 X
Una carga +Q se encuentra inserta en un alambre con-
ductor de largo L y radio R0 muy pequeo. Un cascarn
cilndrico conductor neutro de radios interno R1 y ex-
terno R2 y largo L es ubicado simtricamente alrededor
del alambre (ver figura). Tener en cuenta que: R0  R1 ,
R2  L. Calcule:
a) La densidad lineal de carga del alambre.

b) La densidad superficial de carga en la capa interna


y externa del cascarn; y la densidad volumtrica
de carga dentro del conductor.

c) El campo elctrico en todo el espacio. R2

Ahora deposite una carga Q en el cascarn cilndrico,


R0
calcule: R1

d) Las nuevas densidades de carga superficiales en las


capas interna y externa del cascarn, y tambin la
densidad volumtrica dentro de ste.

e) El nuevo campo elctrico en todo el espacio.

f) La diferencia de potencial entre el cilindro y el


alambre V = Vcilindro Valambre .

g) La capacidad (o capacitancia) del sistema.

h) La energa almacenada en el sistema.

i) La capacidad C 0 si ahora el alambre tiene carga


+2Q y el cascarn tiene carga -2Q.
40 CAPTULO 4. CONDUCTORES, CONDENSADORES Y ENERGA

Problema 4.14 X S
Q
Una esfera maciza de radio a y carga Q uniformemen- a
te distribuida es blindada por una capa conductora de
espesor uniforme a. La carga neta de la capa es nula. 2a
Calcule y grafique el potencial (r) en todo el espacio.
Considere nulo infinitamente lejos de la esfera.

Problema 4.15 X S
Un bloque macizo infinito en sus coordenadas x e y,
posee su espesor entre z = a y z = a. En el espacio
existe una densidad de carga dada por


  0
   |z| > a
(z) = z+a za z
0 exp + exp |z| a

con 0 y constantes positivas conocidas.


z=a
a) El campo elctrico en todo el espacio. r(z)
y
z= a
b) Si  a, determine nuevamente el campo elc-
trico en el espacio qu tipo de comportamien-
to presenta el material?. Justifique su respuesta.
Dibuje las lneas de campo en la prximidad del
material considerando la aproximacin.

c) Usando el campo elctrico calculado en b) deter-


mine el potencial electrosttico en todo el espacio.
Use como referencia V (z = a) = V0 . Dibuje
claramente las superficies equipotenciales.

Problema 4.16 X S
Sean dos cilindros infinitos concntricos conductores,
uno de ellos macizo de radio R1 , y el otro un cascarn
de radios R4 y R5 conectado a tierra, como muestra la
figura. Se coloca una densidad volumtrica de carga 0 r0 R1 R2
R3
entre los cilindros de ancho (R3 R2 ).
a) Determine el campo elctrico en todo el espacio R4
y las densidades de cargas inducidas en las super- R5
ficies conductoras.

b) Calcule la diferencia de potencial entre los con-


ductores.
II. SOLUCIONES 41

II. Soluciones

Solucin 4.1 P X

2
a) La energa de un condensador es U = 21 QC , dado que un condensador es de placas paralelas su
capacitancia est dada por C = Az 0 donde A es el rea de las placas y z la distancia que las
separa, por lo que la fuerza es
U 1 Q2 1 Q
F~ = z = z = z
z 2 0 A 2 0
A0
b) Si se desea disminuir a la mitad la distancia entre las placas del condensador C = z , se tiene
que la nueva capacitancia vale
A0
C 0 = z = 2C
2
Por lo que el trabajo puede ser calculado por la diferencia de energas que experimenta el
condensador
1 Q2 1 Q2 Q2
W = Ufinal Uinicial = =
2 2C 2 C 4C
c) En este caso los condensadores son conectados en paralelo, por lo que la capacitancia del
condensador equivalente es
Ceq = C + 2C = 3C
donde la diferencia de potencial queda determinada por
Q
V =
3C

Solucin 4.2 P X

a) Para iniciar el problema hay calcular el campo elctrico usando la ley de Gauss. Para r R se
tiene que:
E ~ = q = E(r)2rL = L = E
~ dS ~ = r
0 0 20 r
De este resultado ya puede deducirse la densidad de carga, dado que en el borde de un conductor
se tiene que E~ = n = (r). El resultado anterior tiene que ser igual a E(R)
~ =

r,
0 0 20 R
por lo que

=
2R
b) Luego para r > R : Dado que la capa externa del conductor est conectada a tierra, ste posee
potencial cero en toda su superficie lo que provoca que solamente exista la carga negativa
anteriormente calculada. Por lo tanto, volviendo a usar ley de Gauss:

E ~ = q = L + ( 2R ) 2RL = 0 = E
~ dS ~ =0
0 0
Notar que la conexin a tierra provoca fsicamente una descarga de la capa exterior del conduc-
tor.
42 CAPTULO 4. CONDUCTORES, CONDENSADORES Y ENERGA

c) Para el clculo de potenciales hay que notar que para r > R se tiene que

~ = V = 0 = V (r) = Cte.
E

Luego como el potencial es continuo y V (R) = 0 (conexin a tierra) se deduce que

V (r) = V (R) = 0

Finalmente para r < R se tiene que:


r r r

V (r) = V (r) V (R) = E(r)dr = dr = ln
20 r 20 R
R R

Solucin 4.3 P X

Suponiendo una carga Q en la superficie conductora de radio b, entonces el campo elctrico a una
distancia r del origen ser de la forma:
 
Q Q 1 1
E(r) = = V =
40 r2 40 b a

Donde V es la diferencia de potencial entre la superficie de radio interior y exterior. De esta manera,
se obtiene la capacidad del sistema:
Q 40 ab
C= =
V ab
Por otro lado, tomando el caso lmite r = b en el cual el campo elctrico alcanza su mximo valor
posible E(r = b) = E0 , se tiene:

b2 E0
E0 = = Q = 4b2 = 4b2 0 E0 = E(r) = 2
0 r

Luego se usan estos valores obtenidos para calcular la energa del sistema:

Q2 20 b3 E02 (a b)
U= =
2C a
Ahora para encontrar su mximo (o mnimo):

dU 20 E02 3a
= (3ab2 4b3 ) = 0 = b =
db a 4
Luego calculando la segunda derivada:
   
d2 U 20 E02 2 00 3a 20 E02 18a2 27a2
= (6ab 12b ) = U b = = <0
db2 a 4 a 4

Por lo tanto, U b = 3a
4 es mximo, con:
 
3a 270 E02 a3
U b= =
4 128
II. SOLUCIONES 43

Solucin 4.4 P X

Se supondr que la esfera conductora de radio R1 tiene una carga Q q y que la esfera conductora
de radio R2 tiene una carga q. Los campos elctricos que generan ambas esferas son

~ 1 (r1 > R1 ) = Q q r1
E ~ 2 (r2 > R2 ) =
E
q
r2
40 r12 40 r22
Donde r1 y r2 son vectores unitarios en coordenadas esfricas con origen en las esferas R1 y R2 ,
respectivamente. Dado que las dos esferas estn muy alejadas entre s, se considera que el campo
elctrico de una no afecta sobre la otra, por lo que la energa de la esfera de radio R1 es

0
U1 = |E1 |2 d
2
Todo el Espacio
2  2
0 Qq
= r12 sin dr1 dd
2 40 r12
0 0 R1

(Q q)2 dr1
=
80 r12
R1
(Q q)2
=
80 R1
q2
Anlogamente, se tiene que U2 = . Luego, la energa total es
80 R2
(Q q)2 q2
UT = U1 + U2 = +
80 R1 80 R2
Dado que se desea encontrar un mnimo o un mximo
 
dUT 1 Q q q QR2
= + = 0 = q =
dq 40 R1 R2 R1 + R2
Lo que es efectivamente mnimo ya que
 
d2 UT 1 1 1
= + >0
dq 2 40 R1 R2
Por lo que la energa mnima total est dada por

(Q q )2 q2 Q2
Umin = + =
80 R1 80 R2 80 (R1 + R2 )
Por lo tanto, los potenciales a mnima energa son

Q q Q
V1 (R1 ) = E1 (r1 )r1 dr1 r1 = =
40 R1 40 (R1 + R2 )
R1


q Q
V2 (R2 ) = E2 (r2 )r2 dr2 r2 = =
40 R2 40 (R1 + R2 )
R2
44 CAPTULO 4. CONDUCTORES, CONDENSADORES Y ENERGA

Solucin 4.6 P X

Dada la configuracin de la figura, se llamar VA y VB a los potenciales de la placa interior y exterior


respectivamente. Dado que inicialmente el in de masa m y carga q se encuentra en reposo y es
acelerado mediante una diferencia de potencial, se tiene que por conservacin de la energa
1 2 1
Uinicial + Kinicial = Ufinal + Kinicial = mvinicial + qVinicial = mvf2inal + qVf inal
2 2
De este modo si vinicial = 0, Vinicial = V0 , Vf inal = 0, se obtiene que la velocidad con la cual el in
entra a las placas cilndricas es r
2qV0
v=
m
Por otra parte, si se supone una densidad de carga a (a priori desconocida) en el electrodo interior
y se obvian efectos de borde, el campo elctrico que existe dentro de los electrodos semicilndricos es
el mismo en el que existira en un capacitor cilndrico completo, por lo tanto por ley de Gauss para
r [a, b] se tiene que

E ~ = q = E(r) 2rL = a 2aL = E(r)
~ dS ~ =
a a
r
0 0 0 r
Volviendo a analizar el movimiento del in, si se desea que cumpla un movimiento circunferencia de
radio r0 , su aceleracin en coordenadas cilndricas debe estar dada por
v2
~a = r0 2 r = ~a = r
r0
Por Newton
2
~ 0 ) = m~a = q a a = m v = a = 2V0 0
F~ = q E(r
0 r0 r0 a
Reemplazando
~ 2V0
E(r) = r
r
Finalmente
b  
2V0 b
VB VA = dr = 2V0 ln
r a
a

Solucin 4.8 P X

a) Se supondr conocida la densidad de carga a de la esfera de radio a. Dada la simetra esfrica


y que el conductor de radios b y c est descargado se tiene por ley de Gauss que
2 2
~ = E(r) 4r2 = 4a a = E(r) = a a
~ dS
E
0 0 r2
En resumen

0 0<r<a

a a2
~ 0 r2
r ar<b
E(r) =

0 br<c

a a2
0 r2
r r c
II. SOLUCIONES 45

En los intervalos [0, a) [b, c) el campo elctrico es 0 debido a que en ese espacio hay un
conductor.
Luego dado que el potencial sobre en la esfera es V0 se tiene que

a c b a  
~ =
~ dl a a2 a a2 a a2 1 1 1
V0 = E r drr 0 drr r drr = +
0 r2 0 r2 0 c b a
c b

De la ltima expresin

0 V0
a = 1

a2 c 1b + a1

Dado que esta carga induce las otras cargas de induccin en el otro conductor, se cumple que

4a2 a = 4b2 b (induccin de carga en la superficie interior)

4b2 b = 4c2 c (induccin de carga en la superficie exterior)

Por lo tanto
a2 0 V0
b = a = 2 1

b2 b c 1b + a1

b2 0 V0
c = b = 2 1

c2 c c 1b + a1

Usado lo calculado en la parte anterior se tiene que el campo elctrico es

Si r [0, a) [b, c)
~ =0
E

Si r (a, b] (c, ).
2
~ = a a r =
E
V0
 r
1
0 r2 c + a1 r2
1
b

El potencial estar dado por intervalos

Si r (c, ).

c
a a2 a a2 V0
V (r) = r drr = = 1

0 r2 0 r c + a1 r
1
b

Si r (b, c].

c r
a a2 a a2 V0
V (r) = r drr 0r drr = = 1

0 r2 0 c c 1b + a1 c
c
46 CAPTULO 4. CONDUCTORES, CONDENSADORES Y ENERGA

Si r (a, b].
c r r  
a a2 a a2 a a2 a a2 1 1
V (r) = r drr 0r drb r drr = +
0 r2 0 r 2 0 c 0 r b
c b

V0 ( 1c 1b + 1r )
V (r) = 1 1 1

c b + a

Si r [0, a].

V (r) = V0

d) La conexin a tierra provoca los siguientes cambios:

La conexin a tierra provoca por definicin potencial nulo en cada punto del conductor,
luego para que esta condicin se cumpla es necesario que cargas negativas suban de la
tierra y anulen la carga que existe en la capa de radio c del conductor. En consecuencia
se tendr c = 0, con lo cual se cumple que el potencial es nulo en el conductor, ya que
b anula el campo elctrico es producido por a (las cuales se mantienen constantes). De
esta forma, el potencial es nulo para cualquier radio r > c.
El campo elctrico en ese caso vale
Si r (0, a) [b, ).

~ =0
E
Si r [a, b).

~ = V0
E 1
 r
c + a1 r2
1
b

El potencial en este caso vale


Si r [b, ).

V (r) = 0
Si r [a, b).
 
V0 1 1
V (r) = 1 1 1

c b + a
r b
Si r [0, a).
 
V0 1 1
V (r) = 1 1 1

c b + a
a b

Solucin 4.9 P X

La esfera conductora tiene una carga +Q que se puede distribuir tanto en su superficie exterior como
dentro de la cavidad interior. Dado que no existe ninguna carga dentro de la burbuja, la carga que se
II. SOLUCIONES 47

acumula en el borde de la misma es nula. Luego, la carga +Q se distribuye homogneamente en la


superficie exterior de radio R.

De lo anterior, tambin se deduce que el campo elctrico es nulo en cualquier punto dentro de la
esfera (r R), incluyendo los puntos dentro de la burbuja.

Entonces, el campo elctrico para r R puede ser determinado por Ley de Gauss

E ~ = Qenc = E
~ dS ~ = Q r
0 40 r2

Lo anterior implica que el potencial en V (R) es

R
Q Q
V (R) = 2
dr =
40 r 40 R

Como el campo elctrico es nulo en todos los puntos r R, el potencial es constante en todos los
puntos (incluyendo la cavidad) y tiene un valor de V (R) (valor del borde). Ntese el hecho que no
es necesario una forma esfrica para que este anlisis siga siendo vlido.

Solucin 4.12 P X

a) Dada la simetra cilndrica del problema, es posible usar la Ley de Gauss. Para encontrar las
densidades de cargas inducidas en las superficies conductoras a y b se aplica Gauss para un
r [a, b]
E ~ = qenc = a 2aL
~ dS
0 0

Por una parte dentro de un conductor siempre E ~ = 0, por lo que E ~ = 0. Luego se debe
~ dS
cumplir que
a 2aL
= 0 = a = 0
0
Como el conductor interior est cargado con +Q, las densidades de sobre sus superficies deben
cumplir que
Q
a 2aL + b 2bL = Q = b =
2bL
Anlogamente, al aplicar Gauss para r [c, d] se tiene que

E ~ = qenc = Q + c 2cL = 0 = c = Q
~ dS
0 0 2cL

Finalmente como el conductor est cargado Q se tiene que

c 2cL + d 2dL = Q = d = 0

b) Para determinar el potencial, previamente hay que determinar el campo elctrico. En base a lo
anterior para r < b se tiene qenc = 0, luego

~ =0
E
48 CAPTULO 4. CONDUCTORES, CONDENSADORES Y ENERGA

Para r (b, c) se tiene qenc = Q, por lo que



E ~ = qenc = E(r) 2rL = Q = E
~ dS ~ = Q
r
0 0 20 rL

Luego para r > c nuevamente qenc = 0 (notar que toda la carga Q se acumula en la superficie
en r = c), entonces
E~ =0

Entonces tomando el potencial V (r ) = 0, se obtiene que


r
V (r) = E ~
~ dl

Para un r > c se tiene que


r
V (r) = E(r ~ =0
~ > c) dl

Para r [b, c] se tiene que


c r r c
~
~ > c) dl ~ =
~ < r < c) dl Q Q
V (r) = E(r E(b dr = ln
20 rL 20 L r
c c

Finalmente para r < b

c b r b c
~
~ > c)dl ~
~ < r < c)dl ~ =
~ < b)dl Q Q
V (r) = E(r E(b E(r dr = ln
20 Lr 20 L b
c b c

c) La diferencia de potencial est dada por

Q c
V (c) V (b) = ln
20 L b

Solucin 4.14 P X

Para iniciar este problema se debe primero intuir como ser el potencial en todo el espacio, para ello
se debe notar lo siguiente:

La funcin potencial es continua en todo el espacio.

Un conductor es una equipotencial, por lo que en cualquier punto dentro de l la funcin


tendr el mismo valor. Por lo tanto para r [a, 2a] la funcin valdr un valor 0 constante.

La Ley de Gauss es ciega, slo le importa la cantidad de carga encerrada dentro de la


superficie gaussiana que se este tomando (respetando la simetra del problema) por lo que
para r > 2a el campo elctrico ser el mismo si existiese o no el conductor. Evidentemente
pasar lo mismo para r < a .
II. SOLUCIONES 49

Q
r
a
Q r
a
2a
W
2a

Figura 4.1: Campo Elctrico para r > 2a Figura 4.2: Campo Elctrico para r < a

Dadas las consideraciones anteriores y la simetra esfrica, procedemos a calcular el campo elctrico
en todo el espacio (para luego calcular el potencial).

Para r > 2a.



~ dS
E(r) ~ = Q = E(r) 4r2 = Q = E(r)
~ =
Q
r
0 0 40 r2

Para a < r < 2a.


~
E(r) =0

Para r < a.
3
~ dS
E(r) ~ = qencerrada = E(r) 4r2 = Qr = E(r)
~ =
Qr
r
0 0 a3 40 a3

Luego el potencial estar dado por

Para r > 2a.


r
~ dr ~ = Q Q
(r) = E(r) dr =
40 r2 40 r

Para a r 2a. Dado que el potencial es continuo, tendr el mismo valor que tiene en la
frontera para el caso anterior, es decir:
Q
(r) = (2a) =
80 a
Para r < a.

r 2a a r
Q Qr Q Q(a2 r2 )

(r) = E(r)dr = dr + E(r)dr + dr = +
40 r2 40 a3 80 a 80 a3
2a a
| {z }
=0
50 CAPTULO 4. CONDUCTORES, CONDENSADORES Y ENERGA

Finalmente el grfico estar dado por:

f (r )
Q
4pe0 a

Q
8pe0 a

a 2a r

Figura 4.3: Grfico de la funcin (r)

Solucin 4.15 P X

a) Usando la geometra del problema, se puede deducir que el campo elctrico es de la forma
E~ = E(z)z. En particular cabe notar que como (z) = (z) el campo elctrico en un
punto z0 sobre el eje z apuntar en el sentido inverso en punto z0 de modo que |E(z ~ 0 )| =
~ 0 )| = E(z
|E(z ~ 0 ) = E(z 0 )z (ie. existe una simetra del campo elctrico con respecto al
plano xy). Para encontrar el valor del campo elctrico es necesario usar la ley de Gauss tomando
un cilindro centrado en origen con tapas de rea A. Luego

Para |z0 | a

E(z0 )z

A
z=a

z = z0

z = z0
z= a
A

E(z0 )z

Figura 4.4: Superficie Gaussiana para |z| a


II. SOLUCIONES 51

Usando ley de Gauss para calcular el campo elctrico en punto z0 [a, a]



E ~ = qencerrada
~ dS
0
Tapas

1
E(z0 )z Az + E(z0 )z Az = (z)dV
0
Cilindro
z0     
A z+a za
2A E(z0 ) = 0 exp + exp dz
0
z0

z0   z0  
0 z+a za
E(z0 ) = exp dz + exp dz
20
z0 z0
"   z0   z0 #
0 z + a za

= exp + exp
20 z0 z0
    
0 z0 + a z0 a
= exp + exp
0
    
0 z0 + a z0 a
= exp exp
0

~ 0 ) = E(z0 )z para z0 [0, a] y E(z


Ahora se puede concluir que E(z ~ 0 ) = E(z0 )z para
z0 [a, 0], pero dado que E(z0 ) = E(z0 ) , se resume que
    
~ 0 z0 + a z0 a
E(z0 ) = exp exp z
0
Para |z0 | > a

E(z0 )z

z = z0
z=a

z= a
z = z0

E(z0 )z

Figura 4.5: Superficie Gaussiana para |z| > a


52 CAPTULO 4. CONDUCTORES, CONDENSADORES Y ENERGA

Anlogo al clculo anterior, se tiene que para un punto z0 fuera del bloque macizo se tiene

~ = qencerrada
~ dS
E
0
Tapas
a     
1 z+a za
E(z0 ) = 0 exp + exp dz
20
a
a
  a  
0 z+a za
= exp dz + exp dz
20
a a
   a   
0 z+a z a a
= exp + exp
20
   a a
0 2a
= 1 exp
0
Por lo que se concluye que
  

0 2a

1 exp z z0 > a
0
~ 0) =
E(z
  

0 2a

1 exp z z0 < a
0
a
b) La aproximacin dice que  a por lo que se asumir que . Para |z| > a, se tiene que:

  
0 2a 0
E(z0 ) = 1 exp
0 0

Por lo tanto

0

z z0 > a
0
~ 0) =
E(z


0 z z0 < a

0
Para el caso |z| < a se puede hacer de ms de una forma, una es analizando la expresin
    
0 z0 + a z0 a
E(z0 ) = exp + exp
0
0  a  h  z0   z i
0
= exp exp + exp
0
 a
Se tiene que exp 0, por lo que E(z0 ) 0. Sin embargo hay que tener cuidado ya
z 
0
que cuando z0 a (en las cercanas del borde), el valor de exp se vuelve muy grande y

anula el efecto anterior dejando E(z0 a) constante (la aproximacin provoca que la funcin
E(z0 ) crezca muy rpido en las cercanas de a !).
II. SOLUCIONES 53

El anlisis tambin en vlido para z0 a por simetra.

Otra forma mucho ms claro para verlo es usar una aproximacin segn Taylor al primer orden,
es decir

    
0 z0 + a z0 a
E(z0 ) = exp + exp
0
0 h  a  z0  a   a z  a i
0
exp + exp + exp + exp
0
20 z0  a 
exp
0

Si examina la expresin, se sabe que el valor z0 est acotado por a por


 a lotanto la expresin
no puede crecer infinitamente y no puede contrastar el hecho que exp 0.

~ = 0 para z0 (a, a).
Se concluye que E

Como ya se vio, los casos interesantes ocurren en los valores z0 = a o z0 = a, en el cual


aparece una discontinuidad entre los valores de campo elctrico. Dado los resultados anteriores,
podemos que concluir que el material se comporta como un bloque conductor, dentro de l
se tiene E~ = 0 y fuera de l se tiene un valor constante (debido a la carga en sus caras). El
trmino 0 corresponde a la densidad de carga superficial en las caras del conductor. Las
lneas de carga se representan en la Figura 4.6.

z=a
r(z)
y
z= a

Figura 4.6: Lneas de Campo perpendiculares al bloque (en azul).

Las lneas de campo poseen esta forma debido a que siguen la direccin del campo elctrico
(paralelo al eje z). Dentro del conductor no hay lneas debido a que no hay campo elctrico.
54 CAPTULO 4. CONDUCTORES, CONDENSADORES Y ENERGA

c) Usando de referencia V (z = a) = V0 se tiene que la expresin genrica para el potencial


elctrico es

z z
V (z) V (a) = E(z)dz = V (z) = E(z)dz + V0
a a

Analizando por intervalos:

a z a
z
V (z) = E(z) dz + V0 = V0
| {z }
a 0
z>a
a z z
0 0
V (z) = E(z) dz E(z)dz + V0 = V0 dz = V0 (z a)
| {z } 0 0
a 0 a a

z < a
z z
0
V (z) = E(z)dz + V0 = V0 dz = V0 + (z + a)
0 0
a a

Notar que el valor del potencial es simtrico con respecto al plano xy en el caso que
|z| > a, ya que se cumple que V (z0 ) = V (z0 ). Lo anterior es intuitivo, ya que el trabajo
de mover una carga desde el origen hasta z0 ser el mismo que del origen a z0 , todo
debido a la simetra.

z=a
r(z)
y
z= a

Figura 4.7: Equipotenciales paralelas al plano xy (en rojo).

Las equipotenciales son las mostradas en la Figura 4.7, estas superficies son planos paralelos al
xy, ya que el potencial depende slo del valor de la altura z.
II. SOLUCIONES 55

Solucin 4.16 P X

a) Se consideran los distintos casos segn la distancia r desde el centro de la esfera interior.
~ < r < R1 )
E(0
Dado que el cilindro interior es un conductor,
~ =0
E
~ 1 < r < R2 )
E(R
Considerando la simetra del problema, se puede hacer uso de la Ley de Gauss:

E ~ = Qenc
~ dS
0
S

Ntese que no hay carga encerrada en el interior de un radio r < R2 , antes de la densidad
de carga. Luego se tiene:
E ~ = 0 = E
~ dS ~ =0
S
La densidad de carga sobre un conductor puede ser obtenida de

~ 1 ) = (r = R1 ) r = (r = R1 ) = 0
E(R
0
~ 2 < r < R3 )
E(R
Haciendo uso de la Ley de Gauss con un procedimiento anlogo al anterior, se tendr que:

E ~ = Qenc
~ dS
0
S
h
2 r 2 h
1
Erddz = 0 rdrddz
0
0 0 R2 0 0

0 r2 R22 h
2rhE =
0

~ = 0 r R22
2
E r
2r0
~ 3 < r < R4 )
E(R
Haciendo un clculo similar al anterior, pero considerando que como r > R3 , la carga
encerrada se considerar siempre hasta R3 . Luego:

~ = Qenc
~ dS
E
0
S
2 h R32 h
1
Erddz = 0 rdrddz
0
0 0 R2 0 0

~ = 0 R32 R22
E r
2r0
0 (R32 R22 )
~ 4) =
Por otro lado, E(R (r=R4 )
(r) = (r = R4 ) = .
0 2R4
56 CAPTULO 4. CONDUCTORES, CONDENSADORES Y ENERGA

~ 4 < r < R5 )
E(R
Por tratarse del espacio interior entre conductores,
~ =0
E

~ 5 < r)
E(R
Como en R5 se tiene un casquete conectado a tierra, para r R5 el campo se anular.
Por lo cual (r = R5 ) = 0.

b) La diferencia de potencial esta dada por la integral de camino del campo entre R1 y R4 :

R4
V = E ~
~ dr
R1
R2 R3 R4
= E(R1 < r < R2 )dr E(R2 < r < R3 )dr E(R3 < r < R4 )dr
R1 R2 R3
R3  R4 
0 r2 R22 0 R32 R22
= dr dr
2r0 2r0
R2 R3
R
3 R3 2 R4 2 2
0 R2 R3 R2
= rdr + dr + dr
20 r r
R2 R2 R3
    
0 R32 R22 R3 R4
= + R22 ln + (R32 R22 ) ln
20 2 R2 R3
5
Ecuacin de Laplace/Poisson y Mtodo de las
Imgenes

I. Problemas Propuestos

z
Problema 5.1 X S
Una lmina no conductora coincide con el plano xy.
Las nicas cargas en el sistema estn sobre la lmina.
Se sabe que en el semiespacio z > 0 el potencial es
V (x, z) = V0 ekz cos kx, donde V0 y k son constantes.
a) Verifique que este potencial satisface la ecuacin
de Laplace en el semiespacio z > 0. y
b) Encuentre la ecuacin para las lneas de campo
elctrico

c) Encuentre la distribucin de carga sobre la lmina. x


Problema 5.2 X
La parte medular de una memoria RAM y otros dispo-
sitivos semiconductores es la juntura np, que puede
modelarse con la distribucin de carga bosquejada en
la figura, en que la linea punteada representa el plano x
x = 0:
Material Tipo n Material Tipo p
r(x) 6= 0
r(x) = 0 r(x) = 0
+0 si a < x < 0
(x) = 0 si 0 < x < a


0 en el resto del espacio

a) Determine la diferencia de potencial entre el seno


x= a x=a
de la zona n y el de la zona p (es decir, entre un
punto x1 < a y otro x2 > a).

b) Si a = 300 nm y 0 = 1200 C/m3 , calcule nu-


mricamente el valor de la diferencia de potencial
(considere 0 9 1012 F/m).

57
58 CAPTULO 5. ECUACIN DE LAPLACE/POISSON Y MTODO DE LAS IMGENES

Problema 5.3 X
Considere la configuracin mostrada en la Figura, la cual
consiste en dos planos infinitos conectados a tierra ubi-
cados en x = b y x = b, y bloque infinito con una b a +a +b x
densidad de carga volumtrica constante que ocupa el
espacio entre x [a, a]. Usando la ecuacin de Lapla-
r
ce y Poisson, determine el potencial elctrico entre las
dos placas.

Problema 5.4 X S
Considere una esfera maciza de radio a con carga des-
conocida, la cual se encuentra totalmente inmersa en un
fluido que posee una densidad volumtrica de carga con
a
simetra radial dada por (r) = 0 k 2 (r) (r > a)
donde k es una constante, 0 la permitividad del vaco
y (r) el potencial elctrico. Experimentalmente se ha
determinado que el potencial en el borde de la esfera es
V0 con respecto al infinito (() = 0) y la distribucin
volumtrica de carga dentro de la esfera es uniforme.
Determine la densidad de carga (r) y el potencial elc-
trico (r) en todo el espacio.

Problema 5.5 X S V
Considere dos placas conductoras cuadradas de lado a,
las cuales forman un pequeo ngulo . Las placas estn
a una distancia d del vrtice como se ilustra la figura y
a
se encuentran una diferencia de potencial V . Determine a
la capacidad del sistema. d

Problema 5.6 X S
R1
Considere dos condensadores formado por dos cas- R2
quetes esfricos conductores concntricos de radios
R1 ,R2 ,1 ,2 respectivamente. Cada conductor en su po-
lo sur tiene una pequea perforacin para conectar el
casquete inferior (ver figura). Si apropiadamente se co- A
necta cables a los casquetes exteriores y a los interiores,
como se ilustra en la figura, encuentre usando la ecua- r1 C
cin de Laplace la capacitancia del condensador entre r2
los puntos A y C.
I. PROBLEMAS PROPUESTOS 59

Problema 5.7 X S
Considere un plano conductor z = 0, conectado a tierra
y frente al cual se ha colocado una carga q en el punto
x = 0, y = 0, z = h. Z
q
a) Calcule la densidad de carga sobre el plano. Ex-
prese su resultado en funcin de la distancia del h
origen a un punto cualquier sobre el plano.

b) Calcule la carga encerrada en un disco de radio d Y


dibujado sobre el plano conductor con centro en
el origen. Para qu valor de d la carga encerrada
pro el disco es 2q ?.
X
c) Calcule el trabajo que es necesario realizar para
llevar una carga q desde x = 0, y = 0, z = h
hasta x = 0, y = 0, z = 2h, en presencia del
plano conectado a tierra.

Problema 5.8 X S +s
Considere un casquete esfrico cargado de radio R y R
b
con una densidad de carga superficial . Si el centro del
casquete esfrico se sita a una distancia horizontal a
y vertical b con respecto a un plano conductor infinito
doblado en 90 . Encuentre la densidad de carga x y y
sobre los ejes y bosqueje su forma aproximada. a

Problema 5.9 X
Considere una carga puntual q, la cual es colocada en la
bisectriz de dos conductores ideales planos que forman
un ngulo de 45 grados (ver figura). Si la carga tie- d
ne una una distancia d a los conductores, encuentre la
forma del potencial electrosttico entre los conductores. d
45o

Problema 5.10 X S
R
q
Una carga puntual q se ha puesto a una distancia d del
centro de una esfera maciza metlica. Si la esfera se
encuentra conectada a tierra, determine la densidad de d
carga sobre la esfera y la carga total inducida en ella.
60 CAPTULO 5. ECUACIN DE LAPLACE/POISSON Y MTODO DE LAS IMGENES

Q
Problema 5.11 X S
Una carga puntual q se ha puesto a una distancia d R
q
del centro de una esfera maciza metlica. Si la esfera
posee una carga neta Q, determine la fuerza que siente
la carga q. d

Problema 5.12 X S
Considere una esfera metlica de radio R que se encuen-
tra conectada a una fuente a potencial V0 . Frente a ella
se coloca un pndulo de largo ` atado a una muralla a
q, m
distancia d del centro de la esfera. El pndulo lleva en
su extremo una carga puntual q de masa m que forma R `
f
un ngulo con respecto a la horizontal. Despreciando
todos los efectos de la gravedad.
+

a) Determine el mdulo de la fuerza que siente la V0


carga.
d
b) Considere ahora que la fuente se apaga (V0 = 0).
Determine la frecuencia de pequeas oscilaciones
del pndulo si es perturbado dbilmente con res-
pecto a la horizontal.

Problema 5.13 X
En un tnel minero existe un cable que atraviesa toda su
longitud, a una distancia d del techo del tnel. El tnel
puede ser modelado como un cilindro infinito de radio
R, de modo que el cable se mantiene siempre paralelo
al eje imaginario del tnel. En cierto instante, el cable
d
adquiere una densidad de carga lineal + en toda su
extensin. R
a) Encontrar una expresin para el potencial V (r, ) q
dentro del tnel, en trminos de r y (coordena-
das polares).

b) Determinar la densidad de carga () en la pared


del tnel.

c) Cul es la carga total por unidad de longitud


inducida en la pared del tnel?

d) Cul es la fuerza por unidad de largo que siente


el cable?
I. PROBLEMAS PROPUESTOS 61


2p
V = V0 cos x
Problema 5.14 X S a
a
Considere una gua de onda, la cual es una tubera me-
tlica de seccin rectangular de ancho a y alto b. Las
placa inferior y laterales estn conectadas a tierra, es
decir, a potencial cero. La placa metlica superior tie-
b
ne una tensin peridica de perodo 2n a (donde n es
un nmero entero), V (x, y = b) = V0 cos( 2n a x), V0
da cuenta de la intensidad de la tensin. Encuentre la y
tensin al interior, V (x, y). x

V = V0
+

Problema 5.15 X
Se tiene una gua rectangular infinita de lados a y b,
compuesta por cuatro lminas planas conductoras. Dos
de ellas se conectan a tierra, mientras que en las restan-
tes existe un potencial contante de valor V0 , tal como
+
V = V0
se indica en las Figuras.
a) Cules son las condiciones de borde del proble-
ma? b
b) Calcule una expresin general para el potencial en- V = V0
tre las placas usando el mtodo de separacin de
variables. Muestre todos los casos posibles e indi-
que el caso que cumplen las CB. Realice el clculo a V =0 V =0
considerando que cada lado acta por si solo y fi-
nalmente superponga las soluciones encontradas.

V = V0

Problema 5.16 X
y
Usando el mtodo de separacin de variables, calcular el
potencial V (x, y) en el interior de un recinto plano como
el indicado en la figura 1, con las siguientes condiciones
de borde: b

V (0, y) = 0; V (x, 0) = 0;

V V
= 0; = E0
x x=a y y=b a x
62 CAPTULO 5. ECUACIN DE LAPLACE/POISSON Y MTODO DE LAS IMGENES

II. Soluciones

Solucin 5.1 P X

a) Se debe verificar que el potencial V (x, z) = V0 ekz cos kx, cumple la ecuacin de Laplace. En
efecto,

2 V (x, z) 2 V (x, z)
2 V = +
x2 z 2

= (V0 ekz k sin kx) + (V0 kekz cos kx)
x z
= (V0 ekz k 2 cos kx) + (V0 k 2 ekz cos kx)
=0

b) En primera instancia, el campo elctrico para z > 0 es


~ = V = V0 ekz k sin kx x + V0 kekz cos kx z
E

Luego las lneas de campo estn dadas por la solucin de la siguiente EDO

dz Ez cos kx ln(| sin kx|)


= = = z(x) = +C
dx Ex sin kx k
con C R.

c) Dado que el plano es no conductor, la lneas de campo deben ser simtricas con respecto a l
tanto para z > 0, como para z < 0. De este modo, el campo elctrico generado por la densidad
superficial de carga sobre el plano debe ser simtrico con respecto al plano xy. Luego, usando
la condicin de borde sobre la componente normal del campo elctrico, es decir

En1 En2 =
0
donde E1n = E2n = V0 k cos kx, se tiene que

= 20 V0 k cos kx
II. SOLUCIONES 63

Solucin 5.4 P X

Consideremos inicialmente el caso r > a, por Poisson debe cumplirse que

(r)
2 =
0

adicionalmente como (r) = 0 k 2 (r) y hay simetra esfrica, entonces la ecuacin anterior se
transforma en  
1 2
r = k2
r2 r r

Usando el cambio de variables a (r) = r1 (r), la ecuacin se transforma en


    
1 2 2 1 (r) 2 1 (r)
r r (r) + r = k r = (r) + r = k 2 r(r)
r2 r r r r

Volviendo a derivar nuevamente


(r) (r) 2 (r) 2 2 (r)
+ +r = k r(r) = k 2 (r) = 0
r r r2 r2

La ltima ecuacin diferencial tiene por solucin (r) = Aekr + Bekr , donde A y B son constantes
por determinar. Devolvindose con con el cambio de variables
1
(r) = (Aekr + Bekr )
r
Por condiciones de borde (r ) = 0, es decir, debe cumplirse que A = 0 (el potencial no
puede diverger en el infinito). Por otro lado (a) = V0 , luego B = V0 aeka , por lo que el potencial y
la densidad de carga para r > a vale

V0 a k(ar) V0 a k(ar)
(r) = e = (r) = k 2 0 e
r r
Para r < a existe una densidad de carga uniforme (o constante) la cual se denotar como 0 (por
determinar). El campo elctrico dentro de una esfera de radio a, debe valer
3
~ = Qenc = E(r) 4r2 = 4r 0 = E
~ dS
E ~ = 0 r r
0 30 30

Dado que no existen densidades de carga superficiales en r = a, el campo elctrico debe ser continuo
en ese punto. De modo que se cumple que
 
0 a V0 a k(ar) 30 V0 (ka + 1)
= e = 0 =
30 r r r=a a2

Finalmente, el potencial para r < a estar dado por


r  
V0 (ka + 1)r V0 (ka + 1) r 2 a2 V0 (ka + 1)(r2 a2 )
(r)V0 = 2
dr = = (r) = V0
a a2 2 2 2a2
a
64 CAPTULO 5. ECUACIN DE LAPLACE/POISSON Y MTODO DE LAS IMGENES

Solucin 5.5 P X

Solucin 1:

Para encontrar la solucin debe usarse la ecuacin de Laplace. Se supone a priori que los efectos de
borde del condensador son nulos porque el ngulo es pequeo (entre ms separadas estn las placas
del condensador el campo elctrico fuera del condensador deja de volverse despreciable).

Sea la funcin potencial entre las placas, en el espacio dentro del condensador se cumple la ecuacin
de Laplace 2 = 0, donde ( = 0) = 0 y ( = ) = V . Por geometra del problema, el potencial
dentro de las placas debe ir aumentando desde = 0 hasta = con una dependencia nica del
ngulo (en coordenadas cilndricas). Luego

1 2
2 = = 0 = (r) = A + B
r2 d2
V
donde A y B son constantes por determinar. Usando las condiciones de borde, se obtiene que A =
y B = 0, por lo que
V
(r) =

Por consiguiente, el campo elctrico es

~ = = V
E
r
conocido el campo elctrico dentro de las placas, puede despejarse el valor de la densidad de carga
sobre la placa inferior como

~
= 0 E ~ = 0) y = 0 V y y = 0 V
n = 0 E(
borde x x
La carga total de una placa del condensador es

a
d+a  
0 V 0 aV d+a
Q= dS = dxdy = ln
x d
0 d

Finalmente, la capacitancia es  
|Q| 0 a d+a
C= = ln
V d

Solucin 2:

El condensador de placas no paralelas puede descomponerse en pequeos condensadores de placas


paralelas en paralelo (Figura 5.1) . Cada uno de estos pequeos condensadores tiene un ancho infini-
tesimal dx, un espesor a y una separacin de placas x tan , de esta forma cada pequeo condensador
tiene una capacidad
0 A 0 a dx
dC = =
d x tan
La capacitancia final del condensador de placas no paralelas va a ser la suma infinita de las capaci-
tancias de todos los pequeos condensadores. Finalmente, dado que es pequeo tan , por lo
II. SOLUCIONES 65

que se concluye que



a+d  
0 a dx 0 a d+a
C= dC = = ln
x d
d

x tan a

a x

d a
dx
Figura 5.1: Divisin Infinitesimal del Condensador.

Solucin 5.6 P X

En primera instancia se puede notar que ambos condensadores siguen la misma geometra esfrica,
por lo que bastara calcular la capacitancia de uno y el resultado sera anlogo para el otro. Adems,
dada las conexiones de los condensadores, el circuito puede ser modelado de forma que se indica en
la Figura 5.2.

C1

A C

C2
Figura 5.2: Modelo equivalente de los condesadores

Los condensadores C1 (radios R1 y R2 ) y C2 (radios 1 y 2 ) estn en paralelo, por lo que capacitancia


total entre entre A y C es CT = C1 + C2 .
La capacitancia de C1 (y tambin la de C2 ), al ser un condensador esfrico, depende netamente de su
geometra. Para determinar la capacitancia de un condensador de este tipo debe suponerse que una
diferencia de potencial conocida entre las placas, sin perdida de generalidad se asume que el potencial
en el casquete interior es V0 y el exterior 0. Entre las placas se cumple la ecuacin de Laplace (ya que
66 CAPTULO 5. ECUACIN DE LAPLACE/POISSON Y MTODO DE LAS IMGENES

V =0

2V = 0

V = V0
R1

R2

Figura 5.3: Capacitor Esfrico C1

no hay densidades volumtricas de carga), es decir


 
2 1 2 V A
V = 2 r = 0 = V (r) = + B
r r r r

Aplicando las condiciones de borde se obtiene que


V0 R1 R2 V0 R1
V (r) =
r(R2 R1 ) R2 R1

Luego, el campo elctrico es


~ = V = V0 R1 R2
E 2
r
r (R2 R1 )
Por lo que la densidad superficial de carga sobre el conductor interior es
0 V0 R2
= 0 E~ n =
borde R1 (R2 R1 )

Luego la capacitancia de C1 es

Q 1 40 R1 R2
C1 = = dS =
V V0 R2 R1
Casquete

Finalmente la capacitancia total est dada por


40 R1 R2 40 1 2
CT = +
R2 R1 2 1

Solucin 5.7 P X

a) La carga imagen de este problema debe ir a una distancia h bajo el plano, tal como indica
la Figura 5.5. Conocida esa configuracin, se puede calcular el campo que habra justo en la
II. SOLUCIONES 67

Z
q

Figura 5.4: Configuracin Original

Z
q

Y
h

X q

Figura 5.5: Configuracin Equivalente con Carga Imagen

superficie del plano superponiendo el campo elctrico producido por la carga ms su carga
imagen. Dado que la carga q se encuentra en la posicin (0, 0, h) y la carga imagen q en
(0, 0, h) se concluye que el campo elctrico en un punto sobre el plano xy estar dado por:
" #
~ q xx + y y + (z h)z xx + y y + (z + h)z
E(x, y, 0) =
40 (x2 + y 2 + (z h)2 ) 23 (x2 + y 2 + (z + h)2 ) 2
3

~ = 0+ ) = z se
Entonces, usando el hecho que en el borde del conductor se tiene que E(z
0
puede despejar el valor de :

qh
(x, y) = 3
2(x2 + y 2 + h2 ) 2

b) Conocido se integra en un circulo de radio d, para ello se usa el cambio de variables de


68 CAPTULO 5. ECUACIN DE LAPLACE/POISSON Y MTODO DE LAS IMGENES

coordenadas cartesianas a coordenadas cilndricas con r2 = x2 + y 2 y dxdy = rdrd

2 d
qh qh
Qdisco = 3 rdrd = q
2(r2 + h2 ) 2 d2 + h2
0 0

q
Luego si se desea Qdisco = se iguala con la expresin anterior, dando por resultado:
2
qh q
q = = d = h 3
d2 + h2 2

c) Para esta parte del problema se debe considerar que cuando la carga es movida una distancia
hacia arriba en la direccin +z su carga imagen se mueve la misma distancia en la direccin
z. Por consiguiente, si la carga se pusiese en la posicin ~r = z z su carga imagen se encontrar
en ~r0 = z z, por lo que el campo elctrico que sentir la carga en ese punto ser:

~ q 2z z q
E(z) = = z
40 (2z)3 160 z 2

Luego, la carga es movida por un agente externo desde z = h a z = 2h bajo la oposicin del
campo elctrico variable ya calculado, lo que matemticamente resulta como

2h  
q q 1 1
V = V (2h) V (h) = 2
dz =
h 160 z 160 h 2h
Finalmente como el trabajo est dado como W = qV :
 
q2 1 1 q2
W = =
160 h 2h 320 h

Solucin 5.8 P X

El problema nos presenta la siguiente configuracin:

+s
R
b

Figura 5.6: Plano rectangular frente a una esfera


II. SOLUCIONES 69

s +s

b R

a a

+s s

Figura 5.7: Configuracin de las Imagnes

En primera instancia, y haciendo un anlisis slo en el plano xy, se tiene que 2 V = 0, V (x, 0) =
V (0, y) = 0 para x, y 0 y que V (||(x, y)|| ) = 0. Ahora, el problema equivalente que sigue
las mismas condiciones anteriores es mostrado en la Figura 5.7.

En ambos casos tanto el potencial elctrico como el campo elctrico, tendrn el mismo valor para
los puntos que cumplan x, y 0. La configuracin de imgenes estn dada por 3 esferas de radio R
(una con carga + y dos con carga ) ubicadas como se muestra en la Figura ??. Para un manto
esfrico de radio R y densidad se puede determinar el campo elctrico generado por ley de Gauss
de la siguiente forma:
2 2
~ = qenc = E(r) 4r2 = 4R = E
~ dS
E ~ = R r
0 0 0 r2

Dado que todas las esferas estn descentradas, la forma general para determinar el campo que generan
ser
2
E~ = R (~r ~ri )
0 ||~r ~ri ||3
dnde ~r es la posicin donde se desea conocer el campo y ~ri es la posicin donde se encuentra el centro
de la esfera. Para el problema se tiene que ~r1 = (a, b), ~r2 = (a, b), ~r3 = (a, b) y ~r4 = (a, b),
y que ~r = (x, y). Usando el principio de superposicin se tiene que el valor del campo es

4
~ R2 X (1)i1 (~r ~ri )
E(x, y) =
0 ||~r ~ri ||3
i=1

~ R2 (x a, y b) (x + a, y b)
E(x, y) = 3 3
0 ((x a)2 + (y b)2 ) 2 ((x + a)2 + (y b)2 ) 2
!
(x + a, y + b) (x a, y + b)
+ 3 3
((x + a)2 + (y + b)2 ) 2 ((x a)2 + (y + b)2 ) 2
70 CAPTULO 5. ECUACIN DE LAPLACE/POISSON Y MTODO DE LAS IMGENES

En particular en los ejes x e y se tiene que el campo elctrico es, respectivamente:


!
R 2 (0, 2b) (0, 2b)
~
E(x, 0) = 3 + 3
0 ((x a)2 + b2 ) 2 ((x + a)2 + b2 ) 2
!
R 2 (2a, 0) (2a, 0)
~ y) =
E(0, 3 + 3
0 (a2 + (y b)2 ) 2 (a2 + (y + b)2 ) 2

Finalmente como en el borde de un conductor se cumple que = 0 E ~ n , se tiene que
borde

!
2 (0, 2b) (0, 2b)
x (x) = R 3 + 3 (0, 1)
((x a)2 + b2 ) 2 ((x + a)2 + b2 ) 2
!
2 1 1
= 2bR 3 + 3
((x a)2 + b2 ) 2 ((x + a)2 + b2 ) 2

!
2 (2a, 0) (2a, 0)
y (y) = R 3 + 3 (1, 0)
(a2 + (y b)2 ) 2 (a2 + (y + b)2 ) 2
!
2 1 1
= 2aR 3 + 3
(a2 + (y b)2 ) 2 (a2 + (y + b)2 ) 2

Las variaciones de densidad en los ejes se muestran a continuacin (en ambos casos la funciones x
y y son siempre negativas y tiene sus mnimos en los valores de a y b, respectivamente).
sy (y)

+s
R
b

a sx (x)

Figura 5.8: Variaciones de x y y en los ejes

Solucin 5.10 P X

En este caso se debe usar el mtodo de las imgenes para reemplazar la esfera conductora. Se propone
una carga q 0 a una distancia d0 , del centro de la esfera como imagen (ver Figura 5.9).
II. SOLUCIONES 71

r r1
r2
R
q0 q
q
z
d0
d

Figura 5.9: Configuracin Carga Imagen Esfera conectada a tierra

Para encontrar en valor de la carga imagen y del potencial, se determina previamente cual es el valor
del potencial que generan la superposicin de estas dos cargas en el espacio:
 
1 q q0
V (r, ) = +
40 r1 r2

Por teorema del coseno r1 = r2 + d2 + 2rd cos y r2 = r2 + d02 + 2rd0 cos , por lo tanto
 
1 q q0
V (r, ) = +
40 r2 + d2 + 2rd cos r2 + d02 + 2rd0 cos

Ahora tiene que considerarse que V (r = R, ) = 0, ya que se supone que la esfera est conectada a
tierra.
R2 + d2 + 2Rd cos q
V (R, ) = 0 = = 0 = cte.
R2 + d02 + 2Rd0 cos q
De aqu se deduce que, dado que el valor de las cargas es fijo, el lado izquierdo de la ecuacin no
debe depender de ya que para cualquier ngulo el cuociente tiene un valor fijo. Se debe escoger un
valor de d0 de tal forma que la expresin de izquierda deje de depender de .

Reescribiendo q
2
d( Rd + d + 2R cos )
q = cte.
0 R2 0
d ( d0 + d + 2R cos )

Para que la expresin anterior deje de depender de (es decir, poder simplificar el factor dependiente
2
de cos ), se puede elegir d0 = d (no sirve ya que implica q = q 0 ) o d0 = Rd . Por lo que se deduce
que q 0 = Rd q que es la carga efectiva de la esfera. La densidad de carga (), puede ser determinada
como
V
~
= 0 E n = 0 V |r=R r = 0
borde r r=R
Finalmente la densidad de carga es
 
R2
R d q
() = 3
4(R2 + d2 2Rd cos ) 2
72 CAPTULO 5. ECUACIN DE LAPLACE/POISSON Y MTODO DE LAS IMGENES

Como ltimo comentario, cabe mencionar que

2
R
()R2 sin dd = q
d
0 0

Es decir, la carga total inducida en la esfera es igual a la de la carga imagen.

Nota: Existe ms de un mtodo para determinar el valor de las cargas imgenes en este tipo de
problema. Otra posibilidad, es haber igualado la expresin del potencial para dos ngulos distintos
(por ejemplo V (R, = 0) = V (R, = ) = 0) y despejar los valores q 0 y d0 .

Solucin 5.11 P X

El problema puede ser analizado de la siguiente forma, suponga que la esfera estuvo inicialmente
descargada y conectada de tierra. Habr sobre la esfera una carga q 0 (imagen) distribuida en forma
no homognea sobre la superficie (debido a la presencia de q). Ahora, suponga que se desconecta
la esfera y se pone una carga Q q 0 sobre el conductor (de modo que su carga total sea Q), la
carga puesta se distribuye homogneamente sobre la superficie del conductor debido a que las fuerzas
electrostticas presentes ya han sido anuladas gracias a la carga q 0 (visto de otra forma, esa carga es
puesta sobre una superficie equipotencial, por que se repartir en forma equilibrada sobre ella). Por
lo tanto, el potencial debido a Q q 0 ser el mismo que el debido a una carga puntual de ese valor
puesta en el origen, por lo que la configuracin de cargas imgenes es la que se propone en la Figura
5.10.

R
q00 q0 q
x
d0
d

Figura 5.10: Configuracin Esfera Conductora Cargada

En conclusin, la esfera de radio R y carga Q es reemplazada por dos cargas imgenes q 0 y q 00 puestas
en x = d0 y en x = 0, respectivamente. Los valores de cada uno de los trminos anteriores son

qR qR R2
q0 = , q 00 = Q + , d0 =
d d d

Luego, por ley de Coulomb la fuerza sobre la carga q es


 
q q 00 q0
F~ = + x
40 d2 (d d0 )2
II. SOLUCIONES 73

Solucin 5.12 P X

a) Para resolver el problema debe colocarse dos cargas imgenes: una carga que anule el potencial
en el borde y otra que lo suba al borde de la esfera, de modo que se cumpla 2 V = 0,
V (r = R) = V0 , V (r ) = 0.
La primera carga imagen se coloca en la lnea recta entre la carga y el centro de la esfera (el
objetivo de esta carga es V (r = R) = 0. La distancia entre el centro de la esfera y la carga es

L
q, m

L0 `
q0 B
f

Figura 5.11: Configuracin Carga Imagen Esfera Cargada


p
L = d2 + `2 2d` cos . Sea q 0 el valor de la carga imagen y L0 su distancia al centro. Se
debe cumplir que:
q0 q
V (A) = 0
+ =0
40 (L + R) 40 (L + R)
q0 q
V (B) = 0
+ =0
40 (R L ) 40 (L R)
Resolviendo para q 0 y L0 se obtiene:
R R2
q0 = q L0 =
L L
La segunda carga imagen debe elevar el potencial en r = R a V0 , luego se debe cumplir:
q 00
Vq00 (R) = = V0 = q 00 = 40 V0 R
40 R
Con estas dos cargas q 0 y q 00 se cumplen la condiciones de borde (solucin nica). Finalmente
la fuerza es por Coulomb:  
1 qq 0 qq 00
|F | = +
40 (L L0 )2 L2
qq 0
b) Si V0 = 0 = q 00 = 0, luego |F | = 4
1
0 2 . La fuerza se descompone en radial y tangencial
0 (LL )
al pndulo. Dado que nos piden pequeas oscilaciones usamos la componente tangencial (o
normal)
|F | = |F | sin( + )
Usando el teorema del Seno se tiene:
sin sin sin( ( + )) sin( + ) d
= = = = sin( + ) = sin
L ` d d L
74 CAPTULO 5. ECUACIN DE LAPLACE/POISSON Y MTODO DE LAS IMGENES

Fr
f q, m

F
L Ft `
f
d
+

V0

Figura 5.12: DCL al pndulo

Luego se tendr:
d
|F | = |F | sin
L
1 qq 0 d
= 0 2
sin
40 (L L ) L
R2
Reemplazando q 0 = R 0
Lq y L = L

1 q 2 Rd 1 q 2 Rd
|F | =  2 sin = sin
40 R2 2 40 (L2 R2 )2
L L L

Ahora
1 q 2 Rd2 q 2 Rd sin
m` = 2 2 2
sin = + =0
40 (L R ) 40 m` (L R2 )2
2

Si 0, sin y cos 1. Con la ltima aproximacin se tiene para el valor de L:


L2 = d2 + l2 2d` cos d2 + `2 2d` = (d `)2 . Con lo que finalmente se concluye:
r
q 2 Rd q Rd
+ 2 2 2
= 0 = =
4 m`((d `) R ) (d `)2 R2 40 m`
| 0 {z }
2

Solucin 5.14 P X

En este caso se puede usar la ecuacin de Laplace dentro de la gua de ondas ya que no existen
densidades volumtricas de carga. Matemticamente el problema puede ser planteado de la siguiente
forma
2V 2V
2 V (x, y) = + =0
x2 y 2
Sujeto a las siguientes condiciones de frontera

V (x, 0) = V (0, y) = V (a, y) = 0


II. SOLUCIONES 75

 
2x
V (x, b) = A cos
a
para 0 x a y 0 y b.

En este tipo de problemas se usa el mtodo de separacin de variables, es decir, se supone que el
potencial es de la forma V (x, y) = X(x)Y (y), lo cual implica que

d2 X d2 Y 1 d2 X 1 d2 Y
2 V = Y + X = 0 = =
dx2 dy 2 X dx2 Y dy 2
En este punto se puede notar que las variables estn separadas"(no hay dependencia entre ellas, el
lado izquierdo de la igualdad depende slo de x y el derecho slo de y). Por lo tanto, ambos lados de
la igualdad deben ser iguales a una misma constante, es decir

1 d2 X 1 d2 Y
= =k
X dx2 Y dy 2
Ahora se debe analizar por casos:

k = 0 En este caso las soluciones son de la forma

X(x) = Ax + B

Y (y) = Cy + D
Usando las condiciones de borde

V (0, y) = B(Cy + D) = 0 = B = 0 o C = D = 0

En el primer caso C = D = 0, entonces Y (y) = 0 por lo que V (x, y) = 0. Dado que no se


buscan soluciones no triviales de V (x, y) ese caso no sirve. Luego V (x, y) = Ax(Cy + D).

Usando la segunda condicin de borde

V (x, 0) = ADx = 0 = A = 0 o D = 0

Nuevamente si A = 0 se obtiene V (x, y) = 0 por lo que V (x, y) = ACxy.

Aplicando la ltima condicin de borde

V (a, y) = ACay = 0

De aqu es directo que A o C debe ser 0, por lo cual necesariamente se tendr V (x, y) = 0.
Finalmente se deduce que este caso no arroja soluciones.

k > 0 Suponiendo que k = 2 se tiene que en este caso las soluciones son de la forma

X(x) = A cosh x + B sinh x

Y (y) = C cos y + D sin y


Usando las condiciones de borde y recordando que cosh 0 = 1 y sinh 0 = 0

V (0, y) = A(C cos y + D sin y) = 0 = A = 0 o C = D = 0


76 CAPTULO 5. ECUACIN DE LAPLACE/POISSON Y MTODO DE LAS IMGENES

Nuevamente si C = D = 0, entonces Y (y) = 0 por lo que V (x, y) = 0. Luego

V (x, y) = B sinh x(C cos y + D sin y)

Usando la segunda condicin de borde

V (x, 0) = BC sinh x = 0 = B = 0 o C = 0

Nuevamente si B = 0 se obtiene V (x, y) = 0 por lo que V (x, y) = BD sinh x sin y.

Aplicando la ltima condicin de borde

V (a, y) = BD sinh a sin y = 0

Nuevamente se tiene que B o D deben ser 0, por lo cual necesariamente se tendr V (x, y) = 0.
Este caso tampoco arroja soluciones.

k < 0 En el ltimo caso, suponiendo que k = 2 se tiene que las soluciones son de la forma

X(x) = A cos x + B sin x

Y (y) = C cosh y + D sinh y


Aplicando que

V (0, y) = A(C cosh y + D sinh y) = 0 = A = 0 o C = D = 0

Al igual que los casos anteriores, el valor vlido es A = 0, de modo que

V (x, y) = B sin x(C cosh y + D sinh y)

Ahora aplicando que

V (x, 0) = BC sin x = 0 = B = 0 o C = 0

Nuevamente el valor vlido es C = 0, por lo que la forma general de la solucin es

V (x, y) = BD sin x sinh y

Aplicando la condicin de borde


n
V (a, y) = BD sin a sinh y = 0 = a = n = = , nN
a
Finalmente se deduce este caso si provoca soluciones y son una combinacin lineal de la forma

X  n   n 
V (x, y) = Kn sin x sinh y
a a
n=1

Para encontrar los trminos Kn es necesario usar la ltima condicin de borde



X  n     
nb 2x
V (x, b) = Kn sin x sinh = V0 cos
a a a
n=1
II. SOLUCIONES 77

Ahora se usar el hecho que


a (
 n   m  0 si n 6= m
sin x sin x dx = a
a a 2 si n = m
0

m

La expresin anterior debe multiplicarse a ambos lados por sin a x
   m  X  n   m   
2 nb
V0 cos x sin x = Kn sin x sin x sinh
a a a a a
n=1

e integrando en 0 y a (y suponiendo que la integral entra sin problemas a la sumatoria)


a
a    m  X  n   m   
2 nb
V0 cos x sin x dx = Kn sin x sin x dx sinh
a a a a a
0 n=1 0

La serie de la derecha tiene puros trminos nulos excepto cuando n = m, por lo tanto
a    m   
2 a mb
V0 cos x sin x dx = Km sinh
a a 2 a
0

Pero
a    m 
2 am(cos(m) 1)
cos x sin x dx = n 6= 2
a a (m2 4)
0

y para n = 2 se tiene:
a      
2x 2x a 2x a a 
cos sen = cos2 = cos2 (2) cos2 (0) = 0
a a 4 a 0 4
0

Luego
2V0 n(cos(n) 1)
Kn = nb
 n 6= 2
sinh a (n2 4)
Finalmente
 n  sinh 
X n
2V0 n(cos(n) 1) a y
V (x, y) = sin x n
(n2 4) a sinh a b
n=1,n6=2
78 CAPTULO 5. ECUACIN DE LAPLACE/POISSON Y MTODO DE LAS IMGENES
Dipolo Elctrico
6
I. Problemas Propuestos

Problema 6.1 X
z
Considere dos esferas de radio b y densidades de carga
0 y 0 , las cuales estn dispuestas en el eje z, a una
distancia 2a entre ellas, segn se muestra en la figura.
Se pide: 2b

a) Estimar el potencial en el punto (0, L, 0), supo- r0

niendo L  a, b
2a
L y
b) Si se ubica una carga q en la posicin (0, L, 0),
r0
estime la fuerza sobre la carga.

c) Calcule el trabajo necesario para llevar la carga x

desde la posicin (0, L, 0) a (L, 0, 0).

Problema 6.2 X
Considere un aro de radio R sobre el plano xy, el cual
tiene una de sus mitades con una densidad lineal de
carga uniforme (y > 0), y la otra con la densidad
z
opuesta (y < 0), tal como se muestra en la Figura.
a) Calcule el momento dipolar del aro y potencial
electrosttico en la aproximacin del campo le-
jano. Cul es el valor del potencial en el plano l l
xz?. R
b) Determine el valor del campo elctrico en el plano
y
xz usando la aproximacin del campo lejano del
punto anterior. Es consistente este resultado con x
el valor obtenido para el potencial en dicho plano?.

c) En general, o sea no necesariamente lejos del aro,


Cul es el valor del potencial en el plano xz?

79
80 CAPTULO 6. DIPOLO ELCTRICO

Problema 6.3 X S
Un dipolo est compuesto por dos cargas opuestas. Para
modelar un dipolo considere dos esferas macizas de ra- r0 r0
dios R, de densidad de carga constante respectivamente
0 y 0 , las cuales estn conectadas en un punto (ver
R
figura).
z
a) Encuentre el campo elctrico en todo el espacio,
use como origen el punto de contacto entre ambas
esferas.

b) Para distancias muy lejanas, De que forma decae


el campo elctrico?.

~E0
Problema 6.4 X
Considere un dipolo elctrico inmerso en un campo elc-
~ 0 . El dipolo posee un momento dipolar
trico constante E ~p
p y un momento de inercia I. Encuentre la frecuencia de
pequeas oscilaciones del dipolo cuando es perturbado q
dbilmente con respecto a su posicin de equilibrio.
+

z
Problema 6.5 X S
Un dipolo de momento dipolar p~ = px est colocado a ~p

una distancia z0 de un plano conductor infinito conec- z0


tado a tierra tal como indica la Figura. Determinar el
potencial elctrico en cualquier punto (x, y, z) del espa-
x
cio y la densidad superficial de carga inducida sobre un
punto del plano conductor.

Problema 6.6 X S
Una barra de largo L yace a lo largo del eje x con el
origen al lado izquierdo de ella. La barra tiene una den-
sidad de carga no uniforme (x) = x, donde es una y
constante positiva.
a) Exprese la carga total Q de la barra en trminos
de y L. `
x
b) Calcule el campo elctrico en el punto P de la P
L
Figura tomando el lmite `  L. Cmo se com-
porta el campo elctrico tomando ese lmite?.

c) Calcule el potencial elctrico en el punto P to-


mando el lmite `  L. Cmo se comporta el
potencial elctrico tomando ese lmite?.
II. SOLUCIONES 81

II. Soluciones

Solucin 6.3 P X

a) Es importante notar que el problema carece de simetra esfrica (ie. no puede suponerse que
el campo es de la forma E~ = E(r)r), por lo tanto es imposible determinar el campo elctrico
usando directamente ley de Gauss. Para abordar el problema es necesario primero analizar el
campo elctrico que genera una esfera por s sola (independiente del problema actual), para ello
se calcular el campo a una esfera de radio a y densidad de carga homognea .

a a
r
r r r

Figura 6.1: Casos de clculo de campo elctrico para r < a y r a

Por ley de Gauss r < a (puntos dentro de la esfera):



~ ~ q 1 4 ~ r ~r
E dS = = dV = E(r)4r2 = r3 = E(r) = r =
0 0 3 30 30

y para r a (puntos fuera de la esfera)



~ ~ q 1 4 ~ a3 a3~r
E dS = = dV = E(r)4r2 = a3 = E(r) = r =
0 0 3 30 r2 30 |~r|3

En ambos resultados se ha dejado en funcin del vector posicin ~r medido desde el centro de
la esfera, de modo que sea ms fcil determinar el valor del campo elctrico en un punto para
un problema que se tenga ms de un sistema de referencia. Volviendo al problema original, si
se desea calcular el campo elctrico en punto P arbitrario, existen 3 posibilidades en el espacio:

Caso 1: Fuera de ambas esferas


Caso 2: Dentro de la esfera positiva y fuera de la esfera negativa
Caso 3: Dentro de la esfera negativa y fuera de la esfera positiva

Para cada uno de estos casos, cada esfera provoca un campo elctrico en el punto P , por lo
cual hay que usar el principio de superposicin basndose en los resultados ya obtenidos para
una esfera cualquiera.
82 CAPTULO 6. DIPOLO ELCTRICO

Caso 1
Fuera de ambas esferas se tiene la siguiente configuracin que se muestra en la Figura 6.2.
En este caso se tiene que el campo elctrico en el punto P es:

~r+
~r ~r

O O O+ z

Figura 6.2: Campo Elctrico para un punto fuera de ambas esferas

3 3
~ ) = 0 R ~r+ 0 R ~r
E(P
30 |~r+ |3 30 |~r |3
Notando adems que ~r+ = ~r Rz y ~r = ~r + Rz por lo que
3
 
~ ) = 0 R ~r Rz ~r + Rz
E(P
30 |~r Rz|3 |~r + Rz|3
Caso 2
De forma parecida al caso anterior se toma un punto dentro de la esfera positiva y fuera
de la negativa, lo que resulta como
3
~ ) = 0~r+ 0 R ~r
E(P
30 30 |~r |3
Luego  
~ ) = 0 R3
E(P (~r Rz) (~r + Rz)
30 |~r + Rz|3
Caso 3
Finalmente el ltimo caso est dado por un punto dentro de la esfera negativa y fuera de
la positiva, es decir
3
~ ) = 0 R ~r+ 0~r
E(P
30 |~r+ |3 30
de modo que  
~ 0 R3
E(P ) = (~r + Rz) (~r + Rz)
30 |~r Rz|3
II. SOLUCIONES 83

b) En este caso hay que hacer un anlisis para distancias mucho ms grandes de R. Para ello
se debe hacer previamente una aproximacin segn Taylor de los trminos que componen la
expresin. Es decir,
  32
3 2 2 23 1 R2 2R~r z
|~r Rz| = (|~r| + R 2R~r z) = 3 1+ 2
|~r| |~r| |~r|2
  
1 3 R2 2R~r z
3 1
|~r| 2 |~r|2 |~r|2
Dado que se est estudiando el caso |~r|  R se obtiene que el segundo trmino de la expresin
puede ser ignorado (R2 /|~r|2 0). Por lo tanto
    
~ 0 R3 1 3R~r z 1 3R~r z
E= (~r Rz) + (~r + Rz)
30 |~r|3 |~r|5 |~r|3 |~r|5

Manejando algebraicamente la expresin y volviendo a hacer la aproximacin anterior, se llega


a    
~ 0 R 3 2Rz 6R(~r z) 0 R3 2Rz 6R(r z)
E= 3 + ~r = 3 + r
30 |~r| |~r|5 30 |~r| |~r|3
Por lo que el campo elctrico desciende como 1/r3 para distancias muy grandes con respecto
al radio de las esferas.

Solucin 6.5 P X

Usando el mtodo de las imgenes, se obtiene la configuracin equivalente mostrada en la siguiente


Figura 6.3

~p

z0

z0

~p

Figura 6.3: Configuracin Equivalente con dipolo-imagen


84 CAPTULO 6. DIPOLO ELCTRICO

El dipolo se refleja de modo que su imagen tiene un momento dipolar ~ p y se encuentra en z = z0 .


Luego el potencial en un punto ~r = xx + y y + z z est dado por la superposicin del potencial dipolo
y su imagen, es decir

px (xx + y y + (z z0 )z) px (xx + y y + (z + z0 )z)


V (x, y, z) = 3
+
4 |xx + y y + (z z0 )z| 4 |xx + y y + (z + z0 )z|3
| 0 {z } | 0 {z }
Vdipolo Vimagen

!
px 1 1
V (x, y, z) = 3 3
40 (x2 + y 2 + (z z0 )2 ) 2 (x2 + y 2 + (z + z0 )2 ) 2

Ahora, el campo elctrico sobre el plano xy debe ser de la forma E ~ = E(x, y)z ya que ste debe ser
siempre perpendicular a la superficie de un conductor, luego basta saber solamente la derivada segn
z de V (x, y, z) para conocer el campo elctrico que existe sobre un punto en el plano infinito.
!
~ z = V z = 3px z z0 z + z0
E 5 5 z
z 40 (x2 + y 2 + (z z0 )2 ) 2 (x2 + y 2 + (z + z0 )2 ) 2

Finalmente, dado que el plano es conductor, se tiene que

3pz0 x
~ z
(x, y) = 0 E = 5
z=0 20 (x2 + y 2 + z02 ) 2

Solucin 6.6 P X

a) Para encontrar la carga total de la barra se debe integrar la densidad de carga dq, esta ltima
se puede calcular de la definicin de densidad lineal:

dq
(x) = = x = dq = xdx
dx

Luego la carga total es:


L L
L2
Q= dq = xdx =
2
0 0

b) Para calcular el campo elctrico se considera:



~ = 1 ~r ~r 0
E dq
40 |~r ~r 0 |3
barra

Donde dq = xdx con x (0, L), ~r = `x y ~r 0 = xx. Con lo anterior se calcula el campo en
II. SOLUCIONES 85

el punto P :
L
~ = 1 x
E xdx
40 (` + x)2
0
L
xx
=
40 (` + x)2
0
 
`
= + ln(x + `) |L
0 x
40 x + `
   
L L
= ln 1 + x
40 ` L+`
L
Para `  L, `  1 por lo que al hacer una expansin de Taylor:
 
L L L2
ln 1 + = 2 + ...
` ` `
L L L L2
= L
 = 2 + ...
L+` ` 1+ ` ` `
Luego en el lmite, el campo elctrico est dado por:
 
~ L L2 L L2 L2
E= 2 + 2 x = x
40 ` ` ` ` 40 2`2
Usando el resultado de la parte anterior:

~ = Q
= E x
40 `2
Es decir, el campo elctrico se comporta como el generado por una carga puntual Q.
c) El potencial est dado por:
1 dq
V =
40 |~r ~r 0 |
barra

Donde dq = xdx, ~r = `x y r~0 = xx. Luego


L
1 xdx
V =
40 (x + L)
0

= (x ` ln(x + `)) |L
0
40
  
L
= L ` ln 1 +
40 `
Usando la aproximacin igual que en la parte anterior, en el lmite el potencial ser:
  
L 1 L2 L2
V = L` =
40 ` 2 `2 40 2`
Y usando el valor de Q
Q
V =
40 `
que corresponde al potencial generado por una carga puntual Q.
86 CAPTULO 6. DIPOLO ELCTRICO
Parte II

Corriente Elctrica

87
Medios Conductores y Ecuacin de Continuidad
7
I. Problemas Propuestos

Problema 7.1 X
Considere dos esferas conductoras concntricas de radios
a y 3a. La regin entre a < r < 2a es llenada con un a 2a
material de conductividad g1 y la regin entre 2a <
r < 3a tiene material de conductividad g2 . Asuma que V0
3a
ambos materiales tienen una permitividad igual a 0 . La
esfera interior se encuentra a un potencial V = V0 y la g1
exterior V = 0. Determine g2
a) La resistencia del sistema.

b) La densidad superficial de carga en r = 2a.

Problema 7.2 X
Una densidad de corriente, como funcin de la posicin r(~r,t)?
~r y tiempo t tiene la forma:
2
J~ = C~ret||~r||

Donde C y son constantes. Mostrar que la ecuacin de


~ r,t)
J(~
conservacin de la carga se satisface con una densidad
de carga de la forma:
2
= (f + tg)et||~r||

Donde f y g son funciones por determinar que dependen


de la posicin.

89
90 CAPTULO 7. MEDIOS CONDUCTORES Y ECUACIN DE CONTINUIDAD

Problema 7.3 X
y
Entre dos placas conductoras de radio a existe una barra
conductora cilndrica de radio a y longitud
 L, permiti-
vidad 0 y conductividad g = g0 1 + Ly . Se aplica un a g = g(y)
potencial V0 entre las placas.
a) Calcular la densidad de corriente J~ y el campo
L

+
~ dentro del cilindro.
elctrico E V0
b) Calcular la potencia disipada en un disco de espe-
sor e cuyo centro est situado justo en la mitad
del conductor.
z

Problema 7.4 X z = z2

Un material de conductividad g tiene forma de parabo-


loide cortado transversalmente entre los planos z = z1
y z = z2 . Si el paraboloide es de ecuacin x2 + y 2 = kz, z = z1
determine la resistencia del material entre sus caras pla-
nas.
y

Problema 7.5 X S
Una esfera metlica de radio a est rodeada por un cas- g = k|~E|
carn conductor esfrico de radio interior b, donde b > a.
El espacio entre la esfera y el cascarn est lleno de un a
material cuya conductividad elctrica g vara la magni- b
tud del campo elctrico E,~ con la ecuacin g = k|E|, ~
donde k es una constante. Una diferencia de potencial
constante V se mantiene entre la esfera y el cascarn
conductor de radio b. Calcule la corriente elctrica y la
densidad volumtrica de carga entre la esfera y el cas-
carn. Exprese el resultado en funcin de los datos del +
problema. V

d
Problema 7.6 X
Un semianillo de seccin transversal cuadrada est hecho r
de un material conductor uniforme de conductividad g.
Encuentre la resistencia elctrica del semianillo entre sus
caras cuadradas. Utilice las dimensiones que se indican
en la figura.
I. PROBLEMAS PROPUESTOS 91

Problema 7.7 X
Considere un condensador cilndrico de radio interior a y
radio exterior b y largo L. La superficie cilndrica inter-
na del condensador se encuentra a potencial V0 mientras
que la exterior se encuentra a un potencial nulo. El con-
densador tiene dentro de l dos materiales conductores g1
de conductividad g1 y g2 . El material con conductividad
g1 subtiende un ngulo en el condensador, mientras
que el otro ocupa todo el volumen restante. Si el sistema
ha alcanzado el rgimen estacionario, determine
b
a) La corriente elctrica que circula por ambos me- a
dios.

b) Determine la resistencia que opone cada medio al b


paso de la corriente. Demuestre que el valor de la
resistencia total est dado por g2
 1
1 1
RT = +
R1 R2

donde R1 y R2 es la resistencia del medio 1 y 2,


respectivamente.

Problema 7.8 X S
Un bloque de material conductor en forma cubo de
lado a tiene una conductividad no uniforme dada por
g(x) = ga0 (x + a) y una permitividad 0 , donde g0 es
una constante. Asuma que la corriente fluye a lo largo
del eje x desde la cara S hasta la cara opuesta S 0 .
z
Suponga que potencial elctrico V dependiente slo de
x, y que el valor del campo elctrico en el borde es
~ = 0, y, z) = E0 x.
conocido E(x
a) Considerando que se ha alcanzado el rgimen es- S0 a
tacionario, encuentre una ecuacin diferencial de S x
segundo orden que describa el comportamiento del y
potencial dentro del cubo. Use lo anterior para de- a a
terminar la diferencial de potencial entre las caras
S y S 0.

b) Determine la resistencia y la corriente total que


circula entre S y S 0 . Encuentre en valor de la
carga Q que se acumula en el cubo.
92 CAPTULO 7. MEDIOS CONDUCTORES Y ECUACIN DE CONTINUIDAD

e0 , g
Problema 7.9 X
Una esfera de radio a centrada en un punto P se en-
r0
cuentra inmersa en un medio de conductividad g y per-
mitivilidad 0 . En el instante inicial, t = 0, en la esfera a
se sita una densidad de carga uniforme e igual a 0 .
Esta carga, debido a las fuerzas de repulsin electrost- r
tica, se dispersa. Calcular la densidad de corriente sobre
la superficie esfrica de radio r (r > a) y de centro P ,
en el instante t = g0 .

A
Problema 7.10 X
e0
Considere un condensador de forma arbitraria, el cual
tiene dentro de l un medio de conductividad g y per-
mitividad 0 . Sea R y C la resistencia y la capacitancia
entre los terminales A y B, respectivamente. Demuestre
Material
que
RC =
0 g
g

Problema 7.11 X
L
Considere un circuito puesto a tierra consistente en dos
esferas perfectamente conductoras, como es mostrado a1 a2
Aire
en la Figura. Los radios de las esferas son a1 y a2 y la
Tierra
distancia entre ellas es L, donde L  a1 , a2 . Una mitad
de cada esfera est inmersa en tierra, la cual posee una g
conductividad g. Determine la resistencia total entre las
esferas.
I. PROBLEMAS PROPUESTOS 93

Problema 7.12 X S
Considere el circuito mostrado en la figura. El circuito
consta de una fuente de voltaje V0 , un condensador ciln- L
drico, un switch que puede cambiar entre las posiciones
1 y 2 y una resistencia RL desconocida. El condensador
cilndrico tiene largo L y posee un radio interior a y un
radio exterior c (L  a, c). En el interior del condensa-
dor hay dos medios materiales hmicos de conductividad 2
constante g1 y g2 y permitividad aproximadamente igual

+
V0 1
a 0 . Las posiciones respectivas de cada material al in- RL
terior del condensador se muestran en la figura parte
(b). Si el switch inicialmente se encuentra el posicin
1 y el sistema ya ha alcanzado el rgimen estacionario, (a) Vista Exterior
determine
a) El vector densidad de corriente J~ y el campo elc-
trico E~ dentro del condensador.

b) La carga total de cada placa cilndrica del conden-


sador.

c) La corriente que circula por el sistema. a


b
d) La resistencia del sistema.
c
Si el switch se cambia a la posicin 2 y el sistema nue- g1
vamente ha alcanzado el rgimen estacionario.
g2

e) Determine el valor de RL tal que maximice la po-


tencia disipada en dicha resistencia. Cul es el (b) Vista Interior
valor de esa potencia?. (Indicacin: Recuerde la
potencia disipada en una resistencia puede ser cal-
culada como P = V I)
94 CAPTULO 7. MEDIOS CONDUCTORES Y ECUACIN DE CONTINUIDAD

II. Soluciones

Solucin 7.5 P X

Asumiendo que el sistema est en estado estacionario y la simetra radial se tiene que

2
~ J~ = 1 (r Jr ) = 0

r2 r

C
Luego J~ = 2 r. Usando lo anterior, se procede a relacionar J~ con E,
~ de modo que
r

J~ = g E
~ = (kE)E
~ = kE 2 r

Por lo tanto
r
C2 1 C
kE = 2 = E =
r r k

Ahora, para encontrar C se hace desde

r  
~ = V =
~ dl C b kV 2
V = E ln = C = 2 b
k a ln ( a )

Donde se deduce que


~ = V
E r
r ln( ab )

Ya teniendo el campo, es directo que la corriente encontrada se calcula como

2 2
~ = kV 2 4kV 2
I= J~ dS 2 2
kE r sin()dd == r 2
sin()dd =
r2 ln2 ( ab ) ln2 ( ab )
0 0 0 0

La densidad de carga est dada por

~ = V
= 0 E
r2 ln( ab )

Solucin 7.8 P X

a) De la ecuacin de continuidad y considerando que se ha alcanzado un rgimen estacionario, se


tiene
~ J~ = 0

II. SOLUCIONES 95

Considerando la ley de Ohm y desarrollando la expresin:

~ (g(x) E(x))
~ =0
 
V
g(x) =0
x x
g V 2V
+ g(x) 2 = 0
x x x
g0 V g0 (x + a) 2 V
+ =0
a x a x2
2V 1 V
= + =0
x2 x + a x

Ntese que se ha encontrado la ecuacin diferencial pedida. Para resolverla se toma el cambio
de variable = V
x :

d 1
+ =0
dx x + a
d dx
= = ln = ln(x + a) + C
x+a
C1
= =
x+a
dV C1
= =
dx x+a
= V (x) = C1 ln(x + a) + C2

Para determinar la diferencia de potencial entre S y S 0 , se encuentra el valor de C1 usando la


condicin de borde dada
E~ = V x = C1 x
x x+a
C
~ = 0) = 1 x = E0 x
E(x
a
= C1 = E0 a

= E ~ = E0 a x
x+a
Finalmente, al reemplazar C1 , V (x) = E0 a ln(x + a) + C2

= V (S) V (S 0 ) = E0 a ln(a) + E0 a ln(2a)


= V (S) V (S 0 ) = E0 a ln(2)

~ =
b) Considerando que E E0 a g0 (x+a)
x+a x y g(x) = a , se tiene que

g0 (x + a) E0 a
J~ = x = g0 E0 x
a (x + a)

Con esto es posible calcular corriente considerando que el flujo atraviesa un rea cuadrada de
lado a:
a a
I= ~ ~
J dS = g0 E0 dydz = g0 E0 a2
0 0
96 CAPTULO 7. MEDIOS CONDUCTORES Y ECUACIN DE CONTINUIDAD

V
Adems como R = I , usando el resultado recin obtenido para I y el de V de la parte (a),
se concluye:
V E0 a ln(2) ln(2)
R= = =
I g0 E0 a2 g0 a
Finalmente, la carga encerrada se calcula usando Ley de Gauss:

~ ~ Qenc ~ ~ ~ = Qenc
~ dS
E dS = = E dS + E
0 0
cubo S S0
E0 2 Qenc
E0 a2 + a =
2 0
0 E0 a2
= Qenc =
2

Solucin 7.12 P X

a) Aplicando la condicin de solucin estacionaria en la ecuacin de conservacin de carga, se tiene


que:
~ J~ = 1 (rJ(r)) = 0 = rJ(r) = A = J(r)
~ A
= r
r r r

De esta manera, debido a lo anterior, la geometra cilndrica y la continuidad de J~ (por condicin


de borde), se puede asumir una corriente I = cte circulando por el sistema, con:

A
I= J~ dS
~= rddz = 2LA = I = 2LA = cte.
r
Por lo tanto, asumiendo una corriente I en el sistema, se tendr:

I I I I
J~ = r = r = ~1 =
E r; ~2 =
E r
rea 2rL 2rLg1 2rLg2

Donde E ~1 y E~ 2 son los campos elctricos en las zonas con conductividades g1 y g2 respecti-
vamente. A partir de estos campos es posible calcular la diferencia de potencial V0 entre los
radios a y c:

a b a  
~ d~r = ~ 2 d~r ~ 1 d~r = I ln(c/b) ln(b/a)
V0 = E E E +
2L g2 g1
c c b

Y de lo anterior se despeja I:
2LV0 2LV0 g1 g2
I= =
ln(c/b)
+ ln(b/a) g1 ln(c/b) + g2 ln(b/a)
g2 g1

~ E
Finalmente reemplazando el ahora conocido valor de I en las expresiones anteriores para J, ~ 1,
yE~ 1:
~1 = V0 g2
E r
r(g1 ln(c/b) + g2 ln(b/a))
II. SOLUCIONES 97

~2 = V0 g1
E r
r(g1 ln(c/b) + g2 ln(b/a))
V0 g1 g2
J~ = r
r(g1 ln(c/b) + g2 ln(b/a))
~ n, se tiene que en la placa interior, dnde el radio es r = a:
b) Usando el hecho que que = E
E1 (a) 2LV0 g2
a = = Qa = 2aLa =
0 (g1 ln(c/b) + g2 ln(b/a))
De manera anloga en r = c:
E2 (c) 2LV0 g1
c = = Qc = 2cLc =
0 (g1 ln(c/b) + g2 ln(b/a))

c) De los resultados anteriores:


2LV0 2LV0 g1 g2
I= =
( ln(c/b)
g2 + ln(b/a)
g1 )
g1 ln(c/b) + g2 ln(b/a)

d) La resistencia, se obtiene del cuociente:


 
V0 1 ln(c/b) ln(b/a)
R= = +
I 2L g2 g1

e) Aqu primero se debe observar que las resistencias RL y R estn en serie. Luego, sea V1 el
voltaje en la resistencia RL , por LVK y LCK se tiene:

V0 = I(R + RL ); V1 = IRL
Eliminando I de estas dos ecuaciones resulta:
RL
V1 = V0
R + RL
A este resultado genrico, de la caida de potencial en dos resistencias en serie con una fuente se
le conoce como Divisor de voltaje. Ahora se puede calcular la potencia disipada en la resistencia
RL :

V12 RL V02
P = V1 I = =
RL (RL + R)2
Ahora, para maximizar la potencia en funcin de RL :

   
dP 1 2RL 1 ln(c/b) ln(b/a)
= V02 2
= 0 = RL = R = +
dRL (R + RL ) (R + RL )3 2L g2 g1

Es la resistencia RL que maximiza la potencia disipada en ella. Y dicha potencia mxima vale:

V02 V02 Lg1 g2


P = =
4R 2(g1 ln(c/b) + g2 ln(b/a))
98 CAPTULO 7. MEDIOS CONDUCTORES Y ECUACIN DE CONTINUIDAD
Circuitos Elctricos
8
I. Problemas Propuestos

R1 R1
Problema 8.1 X
En el circuito de la figura, encuentre el valor de R1 de
manera que la resistencia equivalente entre las termina- R1 R0
les (donde est abierto el circuito) sea igual a R0 .

Problema 8.2 X S 10 W 15 W A
Una resistencia R se conecta entre los terminales A y B
del circuito de la figura. Calcule el valor de la resistencia
R de manera que:
+

120 V 10 W R
a) La potencia disipada en el circuito sea mxima.
Determine el valor de esa potencia.

b) La potencia disipada en la resistencia R sea m-


xima. Determine el valor de esa potencia. B

Problema 8.3 X
Nueve resistencias R de 10 estn conectadas como se R R R
muestra en la figura y una diferencia de potencial 20 V.
se aplica entre los puntos a y b . a R R R b
a) Calcule la resistencia equivalente de este circuito
entre los puntos a y b. R R R

b) Calcule la corriente de cada una de las 9 resisten-


cias.

99
100 CAPTULO 8. CIRCUITOS ELCTRICOS

100 W
A
Problema 8.4 X
R
En el circuito de la figura, el ampermetro A registra una 50 W
corriente I cuandos ambos interruptores estn abiertos
o ambos cerrados. Hallar el valor de la resistencia R. 300 W
+

1.5 [V ]

Problema 8.5 X
Considere un cubo el cual posee en cada arista una re-
sistencia R. Determine la resistencia equivalente entre
dos vrtices opuestos A y B.
B

Problema 8.6 X R
Encuentre la prdida de energa en forma de calor en
el circuito de la figura. Demuestre que ste nmero no
depende de la resistencia R, teniendo en cuenta que ini- C1 C2
cialmente el condensador de capacidad C1 tiene carga
Q0 y que el segundo condensador se encuentra descar-
gado.

Problema 8.7 X
Considere el circuito de la figura. Antes de t = 0, el
switch est en la posicin 1 por un tiempo muy prolon-
R 1
gado. En t = 0, el switch S es movido a la posicin 2.
Calcule: 2

a) La carga en el condensador en t = 0, Q(0). R


R
+

b) La corriente en t = 0 despus de que el switch es V


movido a la posicin 2, I(0). C

c) La corriente para t > 0, despus de que el switch


fue movido a la posicin 2.

d) La energa U almacenada en el condensador en


t = 0.
I. PROBLEMAS PROPUESTOS 101

Problema 8.8 X
Un condensador, de capacidad C, formado por placas
circulares de radio b est cargado y con voltaje V0 . La se-
+Q0
paracin entre las dos placas paralelas es pequea com-
parada con b. En t = 0 se cierra el switch y el condensa-
dor comienza a descargarse. Calcule, en funcin de V0 , C R
C, b, R, t:
a) La carga Q(t) en funcin del tiempo para la placa Q0
cargada positivamente del condensador.
~ en el espacio entre las dos
b) El campo elctrico E
placas del condensador.

Problema 8.9 X S
En una red elctrica hay n resistencias que cruzan desde
la lnea superior a la lnea inferior, y hay n 1 resisten-
cias a lo largo de la lnea superior e inferior. El nmero
total de resistencias es 3n2. Todas las resistencias son
iguales y de valor R. Determine el valor de R1 , R2 , R3
y R (Ri es la resistencia equivalente vista desde los
terminales para el caso n = i). En la Figura se muestra
el caso n = 4.
102 CAPTULO 8. CIRCUITOS ELCTRICOS

II. Soluciones

Solucin 8.2 P X

a) Primero se debe calcular el valor de la resistencia R, tal que el valor de la potencia total
disipada en el circuito se maximice. En este caso, se calcula la resistencia equivalente del
circuito considerando un valor desconocido R. Luego
 1
1 1 400 + R
Req = 10 + + = (en )
10 15 + R 25 + R
Entonces la potencia est dada por
V2 25 + R
P (R) = = 1202 (en [W ])
Req 400 + 20R
Para maximizar, se deriva la expresin anterior con respecto a R
dP 400 + 20R 20(25 + R) 1202 100
= 1202 = <0
dR (400 + 20)2 (400 + 20R)2
Dado que la funcin potencia tiene derivada negativa, es siempre decreciente para R 0, por
lo que su mximo lo alcanza en su mnimo valor, o sea R = 0 . Por lo tanto la potencia
disipada vale
P = 900 [W ]
b) Ahora hallar el valor de la resistencia R de modo que se mxime la potencia disipada en en si
misma. En este caso es necesario plantear las leyes de Kirchoff:

0 = (15 + R)IR 10(I IR )


120 = 10I + 10(I IR )
Donde I es la corriente que sale de la fuente, e IR es la corriente sobre la resistencia desconocida.
Reordenando las ecuaciones:
10I = (25 + R)IR
120 = 20I 10IR
De estas dos ecuaciones se obtiene que:
 2
60 2 60
IR = = PR = RIR =R
20 + R 20 + R
Luego:
 
dP (20 + R)2 2R(20 + R)
= 602 = 0 = R = 20
dR (20 + R)4
Por lo tanto, el valor de la resistencia que maximiza la potencia disipada en ella es R = 20
y dicha potencia vale:
 2
60
P = 20 = 45 [W ]
40
II. SOLUCIONES 103

Solucin 8.9 P X

En un principio hay que encontrar la recurrencia de la red de resistencias. Para el caso n = 1, siguiendo
todos los datos del enunciado se tiene

Figura 8.1: Red de Resistencias para n = 1

Donde evidentemente R1 = R. Nuevamente cumpliendo los requisitos del enunciado R2 es

Figura 8.2: Red de Resistencias para n = 2


 1
1 1 3
En este caso R2 = + = R. Finalmente el caso R3 estar dado por
R R+R+R 4

Figura 8.3: Red de Resistencias para n = 3


 1
1 1 11
Donde R3 = + = R. En este punto, es posible verificar que la recurrencia
R R + R2 + R 15
de de resistencias est dada por
 1
1 1
Rn = +
R 2R + Rn1

(Nota: Si se desea ser riguroso este resultado debera ser demostrado por induccin, sin embargo para
no escapar de los contenidos del curso no se realizar la demostracin) Ahora, se debe suponer la
sucesin de redes de resistencias converge a un valor R , de modo que

lm Rn = lm Rn1 = R
n n
104 CAPTULO 8. CIRCUITOS ELCTRICOS

Luego, para n suficientemente grande se cumple que


 1
1 1 2

R = + = R + 2RR 2R2 = 0 = R = ( 3 1)R
R 2R + R
Parte III

Magnetosttica

105
Ley de Biot-Savart
9
I. Problemas Propuestos

Problema 9.1 X I

Considere el alambre ABCDA que se muestra en la C B


figura. Por l circula una corriente I en la direccin in-
dicada. Si BC y DA son arcos de circunferencia sub-
tendidos por un ngulo de modo que OA = OD = R D A
y OB = OC = 2R. Calcule el campo magntico B ~ que
a
produce en el centro O. O
y

Problema 9.2 X P

Por un conductor rectilneo semi-infinito circula una co-


rriente elctrica de intensidad I en el sentido que se b
muestra en la figura. Calcule el campo magntico B ~ en
el punto P . I
O x
a

Problema 9.3 X
La figura muestra una espira cuadrada de lado L. Cal- I
cule el campo magntico B ~ en un punto P , que est a
una distancia x del centro del cuadrado, a lo largo de su
x
eje, si por l circula una corriente elctrica de intensidad
I en la direccin indicada.
P

107
108 CAPTULO 9. LEY DE BIOT-SAVART

Problema 9.4 X S P

Un alambre en forma de L lleva corriente I. El alambre D


coincide con el semeje x positivo y con el eje semiposi-
tivo y. Calcule el vector campo magntico en un punto O y
I
P del eje z, ubicado en z = D.
I
x

Problema 9.5 X S
Susana, Rodrigo y Marcel, equipo docente de curso de Susana:
electromagnetismo de una cierta universidad, tienen una 2I
discusin sobre el campo magntico generado por un
R
alambre en un mismo punto P . Susana tiene un alam-
bre infinito por el cual fluye una corriente 2I, Rodrigo, P
al ver el alambre lo toma y lo dobla, formando un cuar-
Rodrigo:
to de circunferencia de radio R ms otros dos tramos
semi-infinitos (ver Figura). Rodrigo le asegura a Susa-
na que si la corriente disminuye a la mitad, el campo R
magntico producido por esta nueva geometra tendr P
una mayor magnitud en el punto P (por lo cual Susa-
na se muestra muy en desacuerdo). Marcel, asombrado
por la discusin de su cuerpo docente, les dice a ambos
estn equivocados ya que falta informacin suficiente I
para aseverar en cual de los dos casos el campo magn-
tico es mayor. Ayude al cuerpo docente de este curso a
determinar quin tiene la razn.

y

ln(2)
r = exp q
2p
Problema 9.6 X
Considere un alambre que ha sido doblado de la forma
que se indica la figura, siguiendo la curva de la funcin
ln(2) 1 2 x
r() = e 2 en coordenadas cilndricas. El alambre da
una vuelta completa y es cerrado por otro trozo que va
desde x = 1 hasta x = 2 sobre el eje x. Si por el alambre
se hace circular una corriente I en sentido antihorario,
determine el valor del campo magntico en el origen.
I. PROBLEMAS PROPUESTOS 109

Problema 9.7 X S w
Considere un disco de radio R que posee una densidad de
carga superficial , uniformemente distribuida. Partien-
do del reposo, el disco comienza a girar, hasta alcanzar
una velocidad angular constante en torno a su eje de
simetra. Encuentre una expresin para el campo mag-
ntico en z = R. (Propuesto: Cunto vale el campo
s R
magntico para cualquier punto en el eje?).

Problema 9.8 X S

Considere un segmento AC recorrido por una corriente


O
de intensidad I como se muestra en la Figura.
a) Determine el valor del campo magntico en el pun- aC
to O en funcin A , C y r
aA
b) Use el resultado calculado en a) para determinar
el campo magntico producido por un polgono r
regular de n lados en su centro. Considere que
el polgono es de lado a y es recorrido por una
corriente I.

c) (Propuesto) Demuestre que si n , el cam- A P I C


po magntico en el centro coincide con el de una
espira circular de radio r.

I
Problema 9.9 X
Encuentre el campo magntico en el punto P sobre el a
P q1
eje de una bobina cilndrica de radio a con una densidad
z q2
de n vueltas por unidad de largo que lleva una corriente
I. Exprese su respuesta en trminos de 1 y 2 (ver
Figura).
110 CAPTULO 9. LEY DE BIOT-SAVART

II. Soluciones

Solucin 9.4 P X

Para el calculo del campo magntico se usar la definicin


~
~ 0 I dl (~r ~r 0 )
B=
4 |~r ~r 0 |3
Primero se calcular la contribucin de campo magntico que genera el cable que est sobre el semi-eje
positivo x. Ntese que en este caso ~r = z z y ~r 0 = xx y dl~ = dxx. Luego
~
~1 = 0 I dl (~r ~r 0 )
B
4 |~r ~r 0 |3

0 Idxx (z z xx)
=
4 |z z xx|3
0

0 I zdx
= 3 y
4 (z 2 + x2 ) 2
0

Usando el cambio de variables x = z tan = dx = z sec2 d, por lo que si x se tiene que



.
2
2
~ 1 = 0 I z 2 sec2 d
B 3 y
4 (z 2 + z 2 tan2 ) 2
0

2
0 I
= y cos d
4z
0
0 I
= y
4z
El otro aporte de campo magntico en ese punto est dado por la generacin de campo magntico
en el semi-eje y. El campo estar dado por
~
~ 0 I dl (~r ~r 0 )
B2 =
4 |~r ~r 0 |3
0
0 Idy y (z z y y)
=
4 |z z y y|3


0 I zdy
= 3 x
4 (z 2 + y 2 ) 2
0

La ltima integral es idntica a la que ya se calcul en la parte anterior (slo cambia la direccin).
Finalmente el campo total es

~ = D) = B
B(z ~ 2 (z = D) = 0 I (x + y)
~ 1 (z = D) + B
4D
II. SOLUCIONES 111

Solucin 9.5 P X

En este caso se elige el mismo sistema de referencia para ambos problemas, en este caso tomamos el
origen en el punto P y con eje x creciente a la derecha e y creciente hacia arriba. Para encontrar los
campos magnticos se usar definicin
~
~ = 0 I dl (~r ~r 0 )
B
4 |~r ~r 0 |3

~ = 2Idx~x con x (, )
Caso Susana: ~r = 0, ~r 0 = Ry + xx, I dl


~ S (P ) = 0 2Idxx (Ry xx) 0 IRz 1 0 IRz 2 0 I z
B 3
= 3 dx = 2 =
4 | Ry xx| 2 (x2 + R2 ) 2 2 R R

(la ltima integral puede resolverse con el cambio de variables x = R tan )


Caso Rodrigo: En este caso hay que separar en tres caminos:

~ = Idxx con x (, 0)
Recta semi-infinita sobre el eje x: ~r = 0, ~r 0 = Ry + xx, I dl
0 0
~ 1 (P ) = 0 Idxx (Ry xx) 0 IRz x 0 IRz 1 0 I z
B 3
= 3 dx = 2 =
4 | Ry xx| 4 (x2 + R2 ) 2 4 R 4R

~ = IRd con ( , 0)
Trozo de cuarto de circunferencia: ~r = 0, ~r 0 = Rr, I dl 2

0 0
~ 2 (P ) = 0 IRd (0 Rr) 0 I z 0 I z
B = d =
4 | Rr|3 4R 8R

2 2

~ = Idy y con y (0, )


Recta semi-infinita sobre el eje y: ~r = 0, ~r 0 = Rx + y y, I dl
0
~ 3 (P ) = 0 Idy y (Rx y y) 0 IRz y 0 IRz 1 0 I z
B 3
= 3 dy = 2 =
4 | Rx y y| 4 (y 2 + R2 ) 2 4 R 4R
0

(Idntico al primer clculo)

Por lo tanto el campo total es:


 
~ R (P ) = B
B ~1 + B ~ 3 = 0 I
~2 + B 1 1
+ z
2R 4

Finalmente, debe notarse que


~S| 0 I
|B R 8 ~ S | > |B
~ R|
=  = > 1 = |B
~ R|
|B 0 I 1
+ 1 4+
2R 4

Dado que todos los datos de la expresin son conocidos, Susana tiene la razn.
112 CAPTULO 9. LEY DE BIOT-SAVART

Solucin 9.7 P X

Calculando el campo magntico por definicin, se tiene que


~ (~r ~r 0 )
0 I dl
~ =
dB
4 |~r ~r 0 |3
aqu
~
I dl ~ = dq dl~ = dq dl = dS ~v = rdrd ~v
dt dt
0
Luego, dado que ~r = Rz, ~r = rr y ~v = ~ ~r 0 = r se tiene que
2 R
2 R 2 drd 2 R 3 drd
~ 0 rdrd r (Rz rr) 0 Rr r
B(Rz) = 3 = 3 r + 3 z
4 2
(R + r ) 2 2 4 (R2 + r2 ) 2 (R2 + r2 ) 2
0 0 0 0 0 0

Por asuntos de simetra la primera integral es nula, por lo que el campo magntico es
R
~ = 0 r3 dr
B 3 z
2 (R2 + r2 ) 2
0

Usando el cambio de variables = R2 + r2 = d = 2rdr, por lo tanto


2 2

2R2
2R
2R
2R2
~ = 0 ( R2 )d 0 ( R2 )d 0 1 3
B 3 z = z 3 = z 2 d R2 2 d
4 2 4 2 4
2
R 2 R 2 2 R R

Resolviendo las dos integrales anteriores, resulta que



~ = 0 (3 2 4)Rz
B
4

Solucin 9.8 P X

a) Se fija un sistema de referencia de centro P para luego usar la definicin de campo magntico.
r
tan A r
tan A
~ 0
~ = 0 I dl (~r ~r ) = B
dB ~ = 0 I dxx (ry x x)
=
0 Irz dx
4 |~r ~r | 0 3 4 3
4 3
(r2 + x2 ) 2 (r2 + x2 ) 2
r tan C r tan C

De este modo, usando el cambio de variables x = r tan = dx = r sec2 d


A

~ = 0 I z
B cos d =
0 I
(sin (A ) sin C )z =
0 I
(sin A + sin C )z
4r 4r 4r
C

b) En el caso del polgono regular se pueden formar n tringulos issceles, de forma que A =
C = n y adems r = 2 tan a
. El aporte de campo magntico en el centro del polgono es
n
realizado por n segmentos como el calculado en la parte a), por lo que el resultado final es
   
~ = 0 nI tan sin z
B
a n n
Fuerza de Lorentz
10
I. Problemas Propuestos

~B = B0 z
Problema 10.1 X






Un protn se dispara a una zona de campo magntico






constante tal como se muestra en la figura. Si el protn






de carga +e y masa m incide con un ngulo a la
zona de campo, determine el valor de la distancia d que a d b
recorre y el ngulo con el cual sale de la zona de ~v0
campo (ver Figura).
+e, m

~B
Problema 10.2 X

Una partcula neutra se encuentra sometida a campo
magntico constante B = B0 z. En t = 0 la partcula se
rompe (decae) en dos cargas +q y q con igual masa

m, las cuales salen disparadas con velocidades en direc-

ciones contrarias. Determine el tiempo en ambas part-
culas vuelven a chocar (exprese su resultado en funcin
de q, m y B e ignore la interaccin coulombiana entre
ellas).
+q q
Problema 10.3 X
z
Una partcula de masa m y carga q que est sometida a
la influencia simultnea de un campo elctrico oscilatorio
en la direccin vertical E~ = E0 cos(t)z y un campo
~ = B0 x
magntico constante en la direccin horizontal B ~E
como se ilustra en la figura. S la partcula parte en el
resposo,
a) Encuentre las componentes x, y y z de la acele-
y
racin de la partcula. q, m
b) Encuentre la velocidad en funcin del tiempo.
~B
c) Encuentre la trayectoria y discuta que ocurre x
cuando la frecuencia es = qB
m

113
114 CAPTULO 10. FUERZA DE LORENTZ

Problema 10.4 X
Una partcula de masa m, con carga q > 0, ingresa
horizontalmente a una regin de ancho L, donde existe
un campo magntico uniforme B ~ = B z como se indica Zona de Campo
en la figura.

a) Calcule el valor crtico de la rapidez inicial ~v0 =
v0 z de la partcula que le permite decidir si ella

atraviesa la regin, o se devuelve.

b) Determine la trayectoria que realiza la partcula ~v0

para los casos en que la rapidez inicial sea mayor
o menor que el valor crtico encontrado anterior- m q







mente.
L
c) Suponga ahora que ~v0 = v0 x + vB z, determine la
altura con que sale la carga de la zona de campo
(considere distintos casos a partir del valor encon-
trado en la parte anterior).

Problema 10.5 X
Un electrn de carga e y masa m se acelera a travs
de una diferencia de potencial de V = 104 [V] antes d
de entrar a una regin limitada por dos placas paralelas
que tienen una diferencia de potencial igual a V0 = 100 d
[V], separadas por una distancia d = 1 [mm] y de largo 2
L = 1 [m]. El electrn entra a esta regin en un plano
equidistante de las placas. Ignorando efectos de bordes:
a) Cul es la velocidad del electrn al llegar a la
regin limitada por las dos placas?
~ se debe aplicar en la
+

b) Qu campo magntico B V0
regin limitada por las dos placas para que el elec-
trn contine con trayectoria rectilnea? Indique la
direccin de dicho campo en la figura.

c) Suponga que, una vez aplicado el campo calculado


anteriormente, suponga que cuando el electrn ha
recorrido una distancia L/2 se quita la fuente de DV
V0 = 100 [V]. Alcanza a salir el electrn de las
placas paralelas?.

Indicacin: tome e/m = 18 1010 [MKS] .


I. PROBLEMAS PROPUESTOS 115

Problema 10.6 X
I1
Suponga que el sistema que se ve en la figura se en-
cuentra en un plano vertical, donde g es la aceleracin d
de gravedad. A travs del alambre rectilneo infinito cir- g
cula una corriente I1 en la direccin indicada. La espira
rectangular, de dimensiones a y b, est ubicada paralela a
al alambre, a una distancia d de l y posee una masa
m. Encuentre la corriente I2 que debe circular por la
espira (indique el sentido de ella), para que permanezca b
en reposo en la posicin sealada.

R
R
Problema 10.7 X S
g
En un espacio de campo magntico constante B ~ = B0 z, ~B0
un fotn experimenta un decaimiento e + e+
donde e representa al electrn y e+ a un positrn. La
carga del electrn es e < 0 y la del positrn es +e;
ambos tienen igual masa m. Como resultado del decai-
miento, ambas partculas adquieren trayectorias espira- s
les como se muestra en la figura. El radio de los mantos
cilndricos por los cuales transita cada partcula es R, y
el paso axial en una vuelta es s. Determine la rapidez
con la que se mueve cada partcula. e
e+

d
Problema 10.8 X l
Se tienen dos cables rectilneos infinitamente largos con
densidad lineal de carga , separados por una distancia l
d, movindose a velocidad v constante. Encuentre el ~v
valor de v para que la atraccin magntica se compense
con la repulsin elctrica. Es razonable este resultado?.
~v
116 CAPTULO 10. FUERZA DE LORENTZ

Problema 10.9 X S
Considere dos cables coaxiales de radios a y b como
muestra la figura, cuyo espacio interior se encuentra va-
co. Los cables se encuentran separados a una diferencia
de potencial V0 y entre ellos existe un campo magntico
homogneo B ~ = B z. Desde el cilindro interior se libera
un electrn de carga e y masa m. El objetivo del pro- ~B
blema es encontrar el valor mximo de B ~ de modo que
el electrn liberado no choque con el cilindro exterior ~v
(alcance a dar la vuelta perfectamente). Asuma que la a
velocidad del electrn tiene solamente componentes en
el plano de la figura.
a) Encuentre el momentum angular del electrn en
b
funcin de carga e, el campo magntico B, y la
distancia al eje del cilindro interior r y momentum
angular inicial L0 .
+

b) Asumiendo que los electrones salen con una velo- V0


cidad inicial v0 0, encuentre la velocidad que
tendr en rmax , es decir en r = b.

c) Mediante conservacin de energa, halle otra ex-


presin para la velocidad recin calculada. A partir
de este encuentre el valor que debera tener B, de
modo que a los ms el electrn volviese en r = b.
II. SOLUCIONES 117

II. Soluciones

Solucin 10.7 P X

Las cargas movindose dentro de campos magnticos se mueven siguiendo una circunferencia (velo-
cidad dentro de un plano) o en una espiral (velocidad en tres dimensiones). Para hallar la rapidez del
electrn, primero se impone la velocidad en coordenadas cilndricas

~v = rr + r + z z

debido a que r = R = r = 0, Por otro lado, se tiene que

F~ = m~a = e~v B
~

entonces como B = B0 z, se tiene que

m(R2 r + R + z z) = eB0 Rr

de lo ltimo se deduce que R = z = 0 y adems


eB0
=
m
Para terminar el problema est faltando el valor de z, que puede ser determinado mediante el paso s
que tiene la espiral (cada 2 radianes el electrn baja s metros) de la siguiente forma
dz d s eB0
z = =
d dt 2 m
Finalmente la rapidez vale
q r  s 2
eB 0
|~v | = (R)2 + z 2 = R2 +
m 2
El resultado es el mismo para el positrn, con la nica diferencia que como su carga es +e el giro
ser en el sentido inverso al del electrn.

Solucin 10.9 P X

a) Previamente se debe notar que la fuerza que afecta al electrn en cualquier punto es F~ = e(~v
~ + E),
B ~ donde B ~ = B z, E
~ = E(r)r y ~v = rr + r, luego F~ = e(rB + rBr + E(r)r)
Para encontrar el momentum angular, se usa el hecho que
~
dL ~
dL dr
~ = = ~r F~ = = rr (erB e(rB + E(r))r) = eBr z
dt dt dt
Integrando a ambos con respecto al tiempo se obtiene que


L(r) r
r 2 a2
dL = eB rdr = L(r) = L0 + eB
2
L0 a
118 CAPTULO 10. FUERZA DE LORENTZ

b) Por otro lado


~ = ~r m~v = |L|
L ~ = mr|~v | = L(r) = mrv

sumando al hecho que v0 0 = L0 0 se deduce que

r 2 a2 (b2 a2 )
mrv = eB = v(b) = eB
2 2mb

c) Por anlisis energtico


1 1
Ui + Ki = Uf + Kf = eV (a) + mv02 = eV (b) + mv 2 (b)
2 2
Dado que V (b) V (a) = V0 y v0 0 se tiene
r
1 2eV0
eV0 = mv 2 (b) = v(b) =
2 m
Finalmente se deduce que el valor de campo magntico crtico es Bc
r r
(b2 a2 ) 2eV0 2mb 2eV0
eBc = = Bc = 2 2
2mb m e(b a ) m
Ley de Ampre
11
I. Problemas Propuestos

Problema 11.1 X S
Un cable coaxial muy largo consiste en un cilindro slido
con un radio interior de radio a, rodeado por un casca-
rn cilndrico conductor concntrico de radio interno b
y exterior c. El conductor interior tiene una densidad
de corriente no uniforme J~1 = rz donde es una
constante. El cilindro exterior tiene una densidad de co-
rriente J~2 = z donde es una constante positiva. J~2 J~1 a
Los conductores llevan una igual y opuesta corriente de

magnitud I0 . Entre ambos conductores existe vaco. b
a) Encuentre los valores y en trminos de a, b, c c
y I0

b) Determine el campo magntico en todo el espacio.


Exprese sus resultados en trminos de a, b, c y I0
~ con respecto a la distancia
c) Realice un grfico |B|
desde el eje de simetra r.

Problema 11.2 X S
Considere 3 alambres rectos de seccin transversal des-
preciable, infinitamente largos y separados en s una dis-
tancia d. Cada alambre lleva una corriente I, en la mis-
ma direccin (perpendicular al plano de la hoja y hacia
adentro). d
a) Encuentre la ubicacin de los dos puntos donde el
campo magntico total se anula. y

x
b) Suponga que el alambre central se desplaza rgi-
damente una pequea distancia y (y  d) en d
direccin perpendicular a la lnea a la lnea sobre
la que estn dispuestos inicialmente los alambres,
mientras los otros dos alambres permanecen fi-
jos. Calcule aproximadamente la fuerza neta por
unidad de largo que acta sobre el alambre des-
plazado, al primer orden de y

119
120 CAPTULO 11. LEY DE AMPRE

Problema 11.3 X S
Tres alambres infinitos estn ubicados, mutuamente pa-
2I
(entra) d

(sale)
I


ralelos estn orientados perpendicularmente al plano de
la hoja y ubicados en los vrtices de un cuadrado de la- d
do d. Uno de ellos lleva corriente 2I que entra al plano, P1
mientras que los otros llevan corriente I que emerge del
plano de la hoja. Calcule el vector campo magntico en
los puntos P1 y P2 . I
(sale)

P2

Problema 11.4 X
Se tiene un conductor en la forma de una capa ciln-
drica recta, infinita, de radio interior a y radio exterior
b. Este conductor tiene una densidad de corriente que, a
expresada en coordenadas cilndricas, es:
b
~ r b) = + z
J(a
r
Con y constantes conocidas. Obtenga el campo
magntico en todas partes.

Problema 11.5 X
y
Se tiene un conductor cilndrico de radio R, muy largo,
con corriente I y un conductor plano, tambin muy lar-
go, con corriente superficial I 0 circulando por l. Ambos
conductores son paralelos y el conductor plano y el eje
de la corriente que circula por el cilindro son coplanares. I0 I
Si el plano conductor tiene ancho a y el eje del cilindro
se encuentra a una distancia b del origen, encuentre: O x
a
a) El campo magntico sobre el eje x para x > a. b
b + 2R
b) La fuerza entre ambos conductores por unidad de
largo.

P1
Problema 11.6 X S
P2
Considere dos planas metlicas infinitas paralelas 1 y
2 , separadas una distancia d. En sus respectivas pla- ~1
K ~2
cas fluyen corrientes en direcciones arbitrarias, las cuales K
tienen una ngulo entre ellas como se ilustra en la Fi- b
gura. Encuentre la densidad de fuerza por unidad de rea
e indique si la fuerza es repulsiva o atractiva a partir de
los distintos valores de . d
I. PROBLEMAS PROPUESTOS 121

J~ = J0 k

Problema 11.7 X
R
Un cable de cilndrico de radio R lleva una densidad de
corriente J~ = J0 k. El cable tiene un hoyo cilndrico de b a
radio a paralelo al eje del cilindro a una distancia b de l.
Muestre que el campo magntico dentro de la cavidad
es uniforme, y encuentre su valor.

Problema 11.8 X
J~ = J(r)x
Se tiene dos corrientes, ambas con igual sentido. Una de R
ellas es una corriente plana e infinita de densidad lineal R
K y otra es una corriente cilndrica infinitamente larga 2
P
cuya densidad es J(r) = J0 1 Rr . Encuentre el valor
de J0 en funcin de K que hace que el campo magntico
resultante en el punto P ubicado a una distancia R2 del
~ = K x
K
centro del cilindro sea nulo.

Problema 11.9 X
J~1 = J1 z w
Considere 3 distribuciones como se muestran en la fi-
gura. Un bloque de ancho w lleva un densidad volum- I
trica uniforme de corriente J~1 = J1 z. Un cable infinito
d
de radio R lleva una densidad de corriente no uniforme y
s
J~2 = rz, donde r es la distancia desde el eje del ci- x
lindro y una constante. El bloque se encuentra a una z R
distancia d del cilindro. Si se coloca un alambre con co- J~2 = rz
rriente I a una distancia s del centro del cable (s < d),
determine la fuerza por unidad de largo que siente el
cable con corriente I.

Problema 11.10 X
R1
Considere dos bobinas muy largas de radios R1 y R2 , con
R2
N1 y N2 vueltas por unidad de largo, respectivamente.
La bobina de radio R2 se encuentra inserta dentro de la
bobina de radio R1 compartiendo su mismo eje. Si por
la bobina exterior circula una corriente I1 , determine:
a) La corriente I2 que circula por la bobina interior, I1 , N1
sabiendo que el campo magntico para r < R2 es
nulo.

b) Para la corriente encontrada en la parte anterior,


encuentre la fuerza por unidad de rea que siente
la bobina interior.
122 CAPTULO 11. LEY DE AMPRE

II. Soluciones

Solucin 11.1 P X

a) Usando la definicin de corriente elctrica a partir de la densidad de corriente, se obtiene que

2 a
~ = 2a3 3I0
I0 = J~1 dS rz rdrdz = = =
3 2a3
0 0

2 c
~ = I0
I0 = J~2 dS z rdrdz = (c2 b2 ) = =
(c2 b2 )
0 b

b) Por otro lado, el campo magntico es por ley de Ampere

r < a:
2 r 2 2
~ = 0 Ienl = B(r)2r = 0
~ dl 2 3 ~ = 0 r = 0 I0 r
B rzrdrdz = 0 r = B
3 3 2a3
0 0

a < r < b:
2 a
~ = 0 Ienl = B(r) 2r = 0
~ dl ~ = 0 I0
B rz rdrdz = I0 = B
2r
0 0

b < r < c:
2 r 2 2
~ = 0 Ienl = B(r) 2r = I0
~ dl
B ~ = 0 I0 c r
z rdrdz = B
2r c2 b2
0 b

r > c:
B ~ = 0 Ienl = B(r) 2r = I0 I0 = 0 = B
~ dl ~ =0

c) Un grfico aproximado es el siguiente:

|B(r)|

0 I0
2pa

0 I0
2pb

a b c r

Figura 11.1: Grfico


II. SOLUCIONES 123

Solucin 11.2 P X

a) Para iniciar el problema se debe calcular el campo magntico que produce un cable que lleva
corriente. Usando Ley de Ampre sobre un camino (circunferencia de radio r recorrida en
sentido anti-horario).

B ~ = 0 I = B(r) 2r = o I = B
~ dl ~ = 0 I ()
2r
Se supondr que el punto donde el campo magntico se anula es sobre el eje X . Sea x el
valor medido desde el centro del sistema de referencia. Debido al superposicin de los campos
magnticos, el campo en ese punto vale:

~ 0 I 0 I 0 I
B(x, 0) = B~1 + B~2 + B~3 = (z) + (z) + z
2(x + d) 2x 2(d x)

Es posible verificar que el valor del campo magntico sobre el eje x siempre va en el sentido z
(o z) y depende slo de la distancia que separa el cable al punto donde se calcula el campo.
Dado que se buscamos B(x,~ 0) = 0 , se obtiene que:
1 1 1
+ =0
x+d x dx
d
donde resultan dos puntos simtricos ubicados en x =
3
b) Primero se calcula el campo magntico que genera el cable de la izquierda sobre el punto (0, y).
Recordando que es B ~ = 0 I () (precaucin: la frmula anterior est con respecto al origen,
2r
aqu la usaremos centrada en el punto (d, 0)) se obtiene que:
I
B~1 = p0 (( sin x + cos y))
2 y 2 + d2

De igual forma, para el cable de la derecha se tiene que:


I 0 I
B~2 = p0 (( sin ( )x + cos ( )y)) = p (( sin x cos y))
2 y 2 + d2 2 y 2 + d2
y
Sumando las expresiones anteriores, y notando que sin = p
y + d2
2

~ y) = B~1 + B~2 = p
B(0,
0 I sin
x =
0 Iy
x
2 2 (y 2 + d2 )
y +d

Por lo que la fuerza estar dada por:

~ B(0, 0 Iy 0 I 2 dz y
dF~ = I dl ~ y) = Idz z
2 2
x = y
(y + d ) y 2 + d2

Se pide la fuerza anterior linealizada (primer orden), por lo tanto reescribiendo la expresin
como y
~ 0 I 2 dz 1 d
dF = y
d ( yd )2 + 1
124 CAPTULO 11. LEY DE AMPRE


a
d

y
y
~B1 ~B2
~B
d
a

x
Figura 11.2: Campos Magnticos aplicados al cable.

Como sabemos que y  d tenemos que hacer una aproximacin segn Taylor de la funcin

f () =
2 +1
en un punto cercano a = 0, por lo tanto

df ()
f () f (0) + ( 0) =
d =0

Finalmente la aproximacin es
y
0 I 2 dz 1 0 I 2 dz y
dF~ = d
y 2 y y
d (d) + 1 d2

Solucin 11.3 P X

Para calcular el campo en los lugares pedidos, primero se debe conocer el resultado genrico para el
campo magntico producido por un alambre infinito con corriente I circulando en el sentido positivo
de z. Usando la Ley de Ampre en una superficie circular de radio r en el plano xy, y asumiendo por
simetra B~ = B(r), entonces:

B~ d~r = 0 I = 2rB = B ~ = 0 I
2r
II. SOLUCIONES 125

Ahora volviendo al problema original, primero se calcula el campo en el punto P1 , colocando al origen
de los ejes cartesianos sobre P1 , con x horizontal hacia la derecha, e y vertical hacia arriba:
     
~ 0 I x + y 0 I x y 0 I x y
B= + + d
2 d2 2 2 d2 2 2 2

Por lo tanto:
~ 1 ) = 0 I (x + y)
B(P
d
Para el clculo en el punto P2 , se reubica el origen de los ejes en l, nuevamente con x horizontal
hacia la derecha, e y vertical hacia arriba:
 
~ 0 I 0 I 0 I x y ~ 2) = 0
B= x + y + = 0 = B(P
2d 2d d 2 2

Solucin 11.6 P X

Se nos pide determinar la fuerza entre las placas que puede ser algo complejo de ver a primera vista.
Por qu siente fuerza una placa?, a priori se podra suponer que la fuerza que siente cada placa es
la misma mdulo, ya que es un par accin - reaccin. Ahora, si se toma como referencia la placa
1 , la placa 2 sentir una fuerza por dos motivos: la corriente que circula por K ~ 1 provoca un
campo magntico en todo el espacio y en 2 existe una corriente circulando. Estos dos elementos,
campos magnticos (por lo general externos) y corrientes (o cargas en movimiento)
siempre provocan fuerza sobre las cargas. Ahora para determinar la fuerza se debe usar la forma
diferencial de la fuerza de Lorentz:
dF~2 = dq~v2 B
~1

Donde F~2 es la fuerza que siente la placa 2 . En este caso el diferencial de carga dq = 2 dS, pero
~ 2 = 2~v2 . Juntando las igualdades anteriores se puede reescribir la fuerza como
por otro lado K
~
dF~2 = K ~ 1 = dF2 = K
~ 2 dS B ~2 B
~1
dS
~ 1 de una placa puede ser determinado por Ley de Ampre (Figura 11.3).
El campo magntico B

~B1
z
x z
~1
K
y

G z

L
Figura 11.3: Corriente en el plano 1 y camino de Ampre.

La curva es un rectngulo de ancho L y alto 2z como puede verse en la Figura 11.3. Adems debe
~ 1 (z)| = |B
notarse que por simetra del problema necesariamente se tiene que |B ~ 1 (z)| . Dado que
126 CAPTULO 11. LEY DE AMPRE

el plano que lleva corriente es infinito, el campo magntico siempre apunta en la direccin x (esto es
debido a que las componentes verticales son siempre anuladas consecuencia de la simetra del plano).
Usando la Ley de Ampre se obtiene que

~ = 0 I
~ dl
B

Por otro lado la integral de camino se puede descomponer de la siguiente forma

L z 0 z
B ~ =
~ dl B1 (z)x dxx + B1 (z)x dz z + B1 (z)x dxx + B1 (z)x dz z
0 z L z

De la expresin anterior se anulan las dos integrales verticales (el campo es ortogonal al camino). Por
lo tanto
B ~ = B1 (z) 2L
~ dl

Por otra parte, si se tienen densidades de corriente superficiales, la corriente enlazada se puede
determinar como (n normal de la superficie donde fluye corriente)

L
I= (K ~ =
~ 1 n) dl (K1 y z) dx x = K1 L
0

Luego la ley de Ampere se transforma en



B ~ = 0 I = B1 (z) 2L = 0 K1 L = B1 (z) = 0 K1
~ dl
2

Notar el hecho que el plano 2 est ubicado en z = d, por lo que el campo magntico que afecta al
plano es
~ 1 (z) = 0 K1 x
B
2
Ahora, descomponiendo K ~ 2 = K2 cos x + K2 sin y, se procede a calcular la fuerza por unidad de
rea que siente el plano 2 como

dF~2 0 K1 0 K1 K2 sin
= K2 (cos x + sin y) x = z
dS 2 2
Finalmente, si (0, ), la fuerza es atractiva (corrientes paralelas), si (, 2) la fuerza es
repulsiva (corrientes antiparalelas), y para los ngulos = 0, no existe fuerza al ser las corrientes
ortogonales.
Potencial Vectorial y Momento Magntico
12
I. Problemas Propuestos

I
Problema 12.1 X S
Considere un cilindro hueco infinito de radio R sobre el R
cual circula sobre su superficie una corriente homog-
neamente distribuida I. Encuentre en todo el espacio el
~ Puede usar el hecho que
vector potencial magntico A.
A(r = 0) = 0.

Problema 12.2 X S
Considere un circuito en forma de hlice circular con su
eje en z. Por la hlice circula una corriente I, y est
compuesta por 2N vueltas completas y tiene un paso p.
La ecuacin paramtrica de la hlice es: z

z2
x = R cos

y = R sin

z = p
I
2
R
Las vueltas de la hlice estn repartidas desde la cota
z1 = pN hasta la cota z2 = +pN .
a) Determine la componente axiales del campo mag-
p
y
ntico y del potencial magntico en el origen,
Bz (0) y Az (0).

b) Si la longitud del circuito helicoidal es L = 2pN ,


muestre que Bz (0) puede ser escrita de la for- x z1
ma Bz (0) = B0 f (R, N, p) donde B0 es el campo
magntico creado con solenoide de longitud infi-
nita.

c) Encuentre el valor de f (R, N, p) cuando R  L.


Qu relacin debe haber entre R y L para que el
Bz (0) sea un 99 % del valor de B0 ?

127
128 CAPTULO 12. POTENCIAL VECTORIAL Y MOMENTO MAGNTICO

Problema 12.3 X
R
a) Considere un alambre de largo 2L sobre el eje z, I
donde fluye una corriente I desde z = L hasta
z = L como se muestra en la figura. Determine el z= L O z=L
potencial magntico vectorial sobre un punto en Parte (a)
el eje y a distancia R del origen. Deduzca el valor
del potencial magntico en el lmite L  R. z
L
b) Se coloca un nuevo alambre de largo 2L, paralelo
al alambre de la parte anterior, por el cual circu- I
la un corriente I en el sentido inverso. El nuevo
alambre se ubica a una distancia d del primero,
y
perpendicular al plano xy (ver figura). Determine d
el potencial magntico vectorial para puntos sobre
el plano xy, a una distancias mucho mayores que x I
L
L.
Parte (b)

!0
Problema 12.4 X S
Considere una esfera de radio R, la cual est cargada
uniformemente con una densidad de carga volumtrica
0 constante. La esfera gira entorno a una de sus di- R
metros con una velocidad angular constante 0 , como
se ilustra en la figura. Encuentre
a) El campo magntico B ~ y el potencial magntico 0
~
vectorial A en el centro de la esfera.

b) El momento magntico de la esfera.

Problema 12.5 X

Una densidad de corriente determinada por J~ = J0 z, y


origina un potencial magntico vectorial
z x
~ = z 0 J0 (x2 + y 2 )
A
4

a) Aplique la ecuacin vectorial de Poisson para con- ~B?


~A
firmar el enunciado anterior.
~ para determinar B.
b) Utilice la expresin de A ~

c) Utilice la expresin para J~ junto a la ley de Am-


pre y determine B.~ Compare su respuesta con la
obtenida en la parte anterior.
I. PROBLEMAS PROPUESTOS 129

A A0
Problema 12.6 X
q q
Un circuito cuadrado rgido de lado L y masa M est ~g
~B
pivoteado en torno a uno de sus ejes (AA0 ) en presencia
de un campo magntico B ~ uniforme, y el campo gra-
vitatorio. El circuito lleva una corriente I que es capaz
de mantenerlo en equilibrio en el ngulo . Encuentre el L
mdulo y el sentido de dicha corriente.

Problema 12.7 X
~B
Una esfera no conductora tiene masa m y radio R. Como
se puede apreciar en la figura, una bobina muy compac-
ta, de cinco vueltas se encuentra enrollada en la esfera;
sta se encuentra en un plano inclinado que forma un
ngulo con la horizontal, de manera tal que la bobina
es paralela a ste. Si un campo magntico B ~ apunta ver-
ticalmente en la regin, cul es la corriente necesaria en
la bobina, que permitir a la esfera permanecer en equi-
librio en el plano inclinado?. Muestre que el resultado es
independiente de . q

Problema 12.8 X
Considere una espira de radio R, N vueltas y masa M y
distribuida homogneamente por la cual circula una co- ~B0
rriente I0 . Esta espira se coloca en un campo magntico
constante de B ~ = B0 x orientado de tal manera que el
torque sobre la espira es mnimo.
a) Demostrar que si la bobina se gira un pequeo n-
gulo con respecto al eje x y es puesto en libertad, R
este comenzar a oscilar entorno la posicin mni- q
mo torque. Encuentre la frecuencia de pequeas x
oscilaciones que presentar la espira. I0
b) Si la espira se suelta desde un ngulo 0 0, N
determine la velocidad angular que tendr la espira
cuando pase por su posicin de equilibrio.
130 CAPTULO 12. POTENCIAL VECTORIAL Y MOMENTO MAGNTICO

Problema 12.9 X S e, me
Considere un electrn orbitando a un protn, que man-
tiene una trayectoria circunferencial de radio R debido a R
la interaccin Coulombiana. Tomando la rbita de carga ~B
como un circuito de corriente elctrica, calcule el torque
resultante cuando el sistema est inmerso en un campo
magntico B,~ orientado perpendicularmente respecto al +e
plano donde vive el protn y el electrn. La carga del
electrn y del protn son conocidas, e y e respectiva-
mente , al igual que la masa del electrn me .

Problema 12.10 X
~m0
Considere tres momentos magnticos iguales m ~ 0 , ubi- C
cados en los vrtices
de un tringulo rectngulo ABC
de lados a, a y a 2.
a) Determine el trabajo necesario para invertir la po- a a
sicin del momento magntico ubicado en el vr-
~m0 ~m0
tice C.
p
b) Cul es el torque que sienten los momentos mag- A a 2 B
nticos ubicados en A y C?.

Problema 12.11 X
Un circuito cuadrado de lado a est suspendido en el I
centro de un enorme anillo fijo de radio R (R  a) por i
un hilo que ejerce un torque restaurador de magnitud
= k, con el ngulo de torsin que forman los planos
de ambos circuitos. Si por el circuito pequeo circula
una corriente i y por el grande I, encuentre en forma y
aproximada el valor de la constante k de modo que =

2 sea posicin de equilibrio. x


II. SOLUCIONES 131

II. Soluciones

Solucin 12.1 P X

En este caso se debe usar ley de Ampre para determinar el campo magntico que genera el cilindro.
Para el caso r < R se tiene que

B ~ = 0 I = 0 = B
~ dl ~ =0

Para el caso r R se obtiene que



~ dl
B ~ = 0 I
~ = 0 I = B
2r
~ A
Ahora dado que ~ = B,
~ integrando a ambos lados y aplicando Teorema de Stokes se obtiene
que:
~ ~ ~
( A) dS = ~ ~ ~
B dS = A dl =~ ~
~ dS
B

~ k J~ ) y
donde es el contorno cerrado de la superficie . Suponiendo que A = A(r)z (siempre A
que A(r = 0) = 0 se obtiene que en el caso r < R:

A ~ =
~ dl B ~ = (A(0) A(r))L = 0 = A(r)
~ dS ~ =0
1 1

W1 r z
G1

z=0 z=L
Figura 12.1: Superficie y camino de integracin para r < R

En la Figura 12.1 se indica 1 y 1 donde se determinaron la integral de lnea y superficie.

Anlogamente para el caso r R, se obtiene que


L r L r
~ =
~ dl ~ = (A(0) A(r))L =
~ dS ~ drdz = 0 I 0 IL  r 
A B B drdz = ln
2r 2 R
2 2 0 0 0 R

De esto, se determina que


~ 0 I  r 
A(r) = ln z
2 R
132 CAPTULO 12. POTENCIAL VECTORIAL Y MOMENTO MAGNTICO

z
W2
r
G2
z=0 z=L
Figura 12.2: Superficie y camino de integracin para r R

Solucin 12.2 P X

p
a) La parametrizacin de la hlice en coordenadas cilndricas es ~r 0 = Rr+ 2 z con [2N, 2N ].
~
Mientras que el dl = Rd + dz z, dado que se pide el campo magntico en ~r = 0, se llega a
~
~ 0 I dl (~r ~r 0 )
B=
4 |~r r~0 |3
p
0 I (Rd + dz z) (Rr 2 z)
= 3
4 (R2 + ( 2 p 2 2
) )
2 Rp
0 I (R dz 2 dr Rdz )
= p 2 32
4 (R2 + ( 2 ) )

Dado que slo interesa la componente axial



2N
0 I R2 d
Bz (0) = 3
4 p 2 2
(R2 + ( 2 ) )
2N
p p
Usando el cambio de variables u = 2R = du = 2R d se llega a
pN
R
0 I du
Bz (0) = 3
2p (1 + u2 ) 2
pN
R
pN
0 I u R

= 1
2p (1 + u2 ) 2 pN
R
!
0 I pN pN
= 1 + 1
2p (R2 + p2 N 2 ) 2 (R2 + p2 N 2 ) 2
0 IN
= 1
(R2 + p2 N 2 ) 2
~ est dado por
Por otro lado, como A
~
~ r ) = 0
A(~
I dl
4 |~r ~r 0 |
II. SOLUCIONES 133

para calcular Az , habr que considerar la componente en z de ~r, luego:


2N
p
0 I( 2 )d
Az (0) = = 2 1
4 p
(R2 + 2 ) 2
2N

p p
Usando el cambio de variables u = 2R = du = 2R d se llega a

pN
R
0 I du
Az (0) = 1
4 (1 + u2 ) 2
pN
R
pN
0 I 2 12
R
= ln |(1 + u ) + u|
4 pN R
1
!
0 I (R2 + (pN )2 ) 2 + pN
= ln 1
4 (R2 + (pN )2 ) 2 pN

b) Primero que todo hay que notar que en el exterior de un solenoide el campo magntico es
nulo ya que al darse un camino cualquiera la corriente encerrada siempre ser nula, por lo que
Bext = 0. Por otro lado, si se tiene un solenoide infinito, se puede calcular el campo magntico
mediante ley de Ampre considerando un camino cuadrado de largo L que encierre n espiras
por unidad de largo por las cuales circula una corriente I0 , por lo que:

~ = 0 nIenlaz = B(z) L = 0 nI = B
B dl ~ = n0 I z

Este caso
Nmero de Vueltas 2N 1
n= = =
Largo 2N p p
Por lo tanto el B0 es
0 I
B0 =
p
Si se considera el resultado calculado en (a) y se reordena de manera que pueda colocarse en
funcin de B0 se tiene:
0 IN
Bz (0) = 1
(R + p2 N 2 ) 2
2

0 I Np
=
p (R2 + p2 N 2 ) 21
1
= B0 
 2  21
R
1+ Np

De la expresin anterior es fcil concluir que Bz (0) puede escribirse como Bz = B0 f (R, N, p),
donde f (R, N, p) est dada por:

 2 ! 12
R
f (R, N, p) = 1+
Np
134 CAPTULO 12. POTENCIAL VECTORIAL Y MOMENTO MAGNTICO


2R 2
c) Finalmente, cuando R  L = L  1, y haciendo una aproximacin de Taylor para f :

 2 ! 12
R
f (R, N, p) = 1+
Np
 2  2
1 2R L
1 + O = f (R, N, p) 1 2
2 L R

Luego es posible concluir que para R  L el cuociente entre Bz (0) y B0 esta dado por
 2
Bz (0) R
=12
B0 L

Por ende para que el valor de Bz (0) sea dicho cuociente sea un 99 % del valor de B0 , el cuociente
debe tomar el valor de 0,99 con lo que se concluye la relacin entre R y L para que esto se
cumpla:
 2
Bz (0) R
= 0,99 = 1 2
B0 L
 2
R 1
2 =
L 100
R 1
= =
L 10 2

Solucin 12.4 P X

a) Primero se determina el campo magntico que genera la esfera, para ello se usa la ley de
Biot-Savart ~ 0
0 J(~r ) (~r ~r 0 )dV
B(~r) =
4 ||~r ~r 0 ||3
Esfera

En este caso, ~r = 0, ~r 0 = rr y J~ = 0~v = 0 r sin . Manejando algebraicamente la expresin


anterior
2 R 2 R
~ 0 0 r sin rr 2 0 0
B(0) = 3
r sin drdd = r sin2 drdd
4 r 4
0 0 0 0 0 0

Pero = x cos cos + y cos sin z sin , por lo que las integrales en x e y se anulan
debido a la integral en . Luego el campo slo tiene componente en z y su valor es

2 R
~ 0 0 z
B(0) = d sin3 d rdr
4
0 0 0

La nica integral compleja es



3 2 2 1

3 4
sin d = (1cos ) sin d = sin d cos sin d = cos |0 + cos =
3 0 3
0 0 0 0
II. SOLUCIONES 135

Luego el campo vale


2 2
~ = 0 0 z 2 4 R = 0 0 R z
B
4 3 2 3
Para el caso de potencial magntico se tiene que
~
0 JdV
A(~r) =
4 ||~r ~r 0 ||
Esfera

~ ~r y ~r 0 son los mismos que la parte anterior. Entonces


Los vectores J,
2 R
~ 0 0 r sin 2
A(0) = r sin drdd
4 r
0 0 0

Pero dado que = x sin + y cos , la integral en vuelve anular las dos coordenadas de
modo que
2 R
~ 0 0
A(0) = (x sin + y cos )d sin d r2 dr = 0
2
4
0 0 0

b) Para el caso del momento magntico, este esta dado por



1 ~
m~ = ~r 0 JdV
2
~ ya fueron calculados previamente. Reemplanzando
donde ~r 0 y J,
2 R
1 0 ~ 0
m
~ = ~r JdV = r4 sin2 drdd
2 2
0 0 0

Nuevamente al descomponer el vector se anulan las componentes en x e y. Entonces


2 R
0 z 3 40 R5
m
~ = d sin d r4 dr = z
2 15
0 0 0

Solucin 12.9 P X

Se debe comenzar calculando el momento dipolar m


~ = IAn, donde I es la corriente y A es el rea.
Para este caso la corriente est dada por:
dq e
I==
dt T
donde e es la carga del electrn, me la masa del electrn y T su periodo. Para encontrar el periodo se
puede relacionar con su frecuencia mediante T = 2 v
, con = R . Luego falta encontrar v, para ello
usamos la interaccin coulombiana en el interior de los tomos y la expresin para fuerza centrpeta:
e2 me v 2
Fe = Fc = =
40 R2 R
s
e2
= v =
40 me R
136 CAPTULO 12. POTENCIAL VECTORIAL Y MOMENTO MAGNTICO

Con la ltima expresin es posible concluir que el periodo est dado por:

2R 2R 40 me R
T = =
v e
Conocido T , se puede concluir I:

e e2
I= =
T 2R 40 me R

Para terminar el clculo, se considera que el rea viene dada por una circunferencia, por lo que
A = R2 . Con esto se tiene que
e2 R
~ =
m z
2 40 me R
Finalmente, el torque est dado por
2
r
|~ | = |m ~ sin 90 = e B
~ B| R
2 40 m
Parte IV

Campos Electromagnticos Variantes


en el Tiempo

137
Ley de Faraday-Lenz
13
I. Problemas Propuestos

b
Problema 13.1 X S
Una bobina muy larga de radio b tiene m vueltas por I(t)
unidad de largo y lleva una corriente I(t) = I0 sin t.
a) Encuentre el campo magntico dentro de la bobi-
na.

b) Encuentre el campo elctrico dentro de la bobina.

c) Encuentre el campo elctrico fuera de la bobina.

~B = B0 z
Problema 13.2 X y

Un carrito de masa m que se desplaza con velocidad ~v = v0 x


v0 x, hasta llegar a una regin en que existe un campo
magntico uniforme en x = 0. R
a
a) Si el carrito posee solamente una resistencia R
(Figura a), encuentre la velocidad del carrito como x
funcin del tiempo.
(a) Circuito Simple
b) Si el carrito posee una resistencia R y un conden-
sador C inicialmente descargado (Figura b), en- ~B = B0 z
cuentre la velocidad del carrito como funcin del y
tiempo y la carga del condensador en el tiempo.
~v = v0 x
En ninguno de los dos casos, considere el campo mag-
ntico producido por la corriente en el circuito por ser R
a C
mucho menor al campo externo constante y suponga
que el carrito es muy largo por lo que no debe preo- x
cuparse de lo que ocurre una vez que entra entero al
campo.
(b) Circuito RC

139
140 CAPTULO 13. LEY DE FARADAY-LENZ

Problema 13.3 X S
Un generador de corriente G produce una corriente I(t)
en un circuito formado por un riel conductor sin roce,
en forma de U y una barra conductora de masa m que I(t)
atraviesa los rieles en direccin perpendicularmente. Un
campo magntico constante y uniforme B ~ que apunta
perpendicular al plano del circuito existe en todo el es-


pacio. La corriente I(t) que circula en el circuito vara
en el tiempo de acuerdo con G L

0 t<0
I(t) =

bt 0 < t < T
0 T <t

Para t T , determine la fuerza electromotriz inducida


en el circuito y la velocidad de la barra en funcin del
tiempo.

Problema 13.4 X
z
Un disco de material aislante de radio R tiene en su
borde una carga uniformemente distribuida [C/m] y
est suspendido horizontalmente de un hilo que coincide
con su eje. Dentro de un crculo ms pequeo de radio
a < R existe un campo magntico B ~ uniforme paralelo
al eje. El disco est inicialmente en reposo. En t = 0 se l
R a
desconecta la fuente del campo magntico, el que cae a
cero despus de un corto intervalo de tiempo. Si I es el
momento de inercia del disco, encuentre:
a) La velocidad angular final del disco

b) Depende sta de la forma en que cae a cero el


campo B?

Problema 13.5 X D

Una bobina circular plana de N vueltas tiene dimetro D


y resistencia R. La bobina se orienta con su eje paralelo
a un campo magntico B uniforme y los extremos de la
bobina se conectan a un dispositivo capaz de medir la N
carga que pasa a travs de l. Si la bobina se gira en
Dispositivo
180 sobre un eje perpendicular al campo magntico el Medidor de Carga
dispositivo mide una carga Q. Encuentre el valor de B
en trminos de N , D, R y Q.
I. PROBLEMAS PROPUESTOS 141

Problema 13.6 X S
Una barra conductora de masa m desliza sin roce sobre
dos rieles conductores paralelos separados una distancia
b. El circuito posee un switch que puede cambiar entre
dos posiciones. Si el switch se encuentra en la posicin 1,
los rieles quedan conectados nicamente mediante una ~B = B0 z

resistencia R, mientras que si se mueve a la posicin 2,

los rieles quedan conectados por la misma resistencia R
y una batera que provee un diferencia de potencial V0
(ver Figura). Hay un campo magntico uniforme B ~ = b
R
B0 z uniforme perpendicular al plano de la Figura.

a) Considere que inicialmente el switch se encuentra
1
en la posicin 1 y se le imparte una velocidad v0

a la barra en t = 0. Determine la velocidad en +
2
funcin del tiempo de la barra y el desplazamiento V0
mximo que alcanza la misma.

b) Suponiendo que la barra alcanz su mximo des-


plazamiento, el switch se cambia la posicin 2 y
se reinicia el tiempo. Determine nuevamente la
velocidad de la barra en el tiempo.

Problema 13.7 X S
Los hornos de induccin se usan para fundir metales.
Ellos consisten en grandes contenedores donde caben
hasta 30 toneladas, aislados trmicamente y rodeados
de una gran bobina donde circula corriente. A modo de a
ejemplo, considere el siguiente modelo simplificado. Una
barra conductora de radio a, largo h y conductividad g h
se coloca en la interior de una bobina que tiene n vueltas
por unidad de longitud y que lleva una corriente alterna
I = I0 sin t con la frecuencia angular. Calcule la
potencia disipada en el cilindro. Desprecie efectos de
borde. (Propuesto: Cul es el valor de la corriente que
circula dentro del cilindro?).

Problema 13.8 X a
g
Una espira rectangular de resistencia R, ancho a, largo
L (muy grande) y de masa m cae bajo efecto de la
gravedad bajo un campo magntico B ~ perpendicular al
plano de la espira y no nulo solo en la parte superior del
plano. Asuma que una parte de la espira siempre est
afuera de la regin de campo magntico, determine la ~B

velocidad terminal y el sentido de la corriente inducida
en la espira.
142 CAPTULO 13. LEY DE FARADAY-LENZ

Problema 13.9 X
Una espira circular de radio a, masa M y resistencia R
z
se deja caer desde z = 0 con su eje de simetra vertical
en una zona donde el campo magntico es axialmente
~B
simtrico alrededor del eje z y cuya componente vertical
es Bz = Cz. El eje de la espira coincide con el eje de
simetra del campo.
a) En qu direccin fluye la corriente en la espira
a
mientras cae bajo la accin de la fuerza de grave-
~g
dad?.
M, R
b) Encuentre la corriente en la espira en funcin de
la velocidad. Desprecie las contribuciones al flujo
provenientes de la corriente inducida.

c) Determine las fuerzas que actan sobre la espira


provenientes axial y radial del campo magntico.

d) Encuentre la velocidad de la espira despus que


ha cado un largo tiempo.

Problema 13.10 X
Una barra metlica de largo d y espesor despreciable ~v
se mueve con rapidez v constante, de forma paralela a
una corriente elctrica de intensidad I. Si el extremo
a
izquierdo de la barra est a una distaba a del cable con d
corriente, determine la diferencia de potencial entre los
extremos de la barra. Cul es el valor de esa diferencia
cuando a  d? .

~B = B0 k
Problema 13.11 X S W

El disco de Faraday consiste en un disco conductor de


e
radio R que gira con velocidad angular k bajo la accin
de un campo magntico constante B ~ = B0 k (ver Figu-
ra). Calcule el mdulo de la fuerza electromotriz (fem) R
inducida entre el centro y el borde del disco.
II. SOLUCIONES 143

II. Soluciones

Solucin 13.1 P X

a) En primera instancia se debe usar el hecho que dentro de una bobina se tiene B ~ = B(z)z y
se est trabajando en un rgimen cuasiestacionario (se desprecia la corriente desplazamiento).
Luego, usando Ley de Ampre para el camino recorrido en sentido antihorario y suponiendo
que la bobina tiene una largo L, se obtiene que

~ = 0 Ienlaz = B(z, t) L = 0 mLI(t)
~ dl
B
1

Lo que finalmente implica que


~ = 0 mI0 sen(t)z
B

b) Usando un camino circular de radio r < b, se tiene que por ley de Faraday-Lenz

~ =
~ dl
E ~ = E(r, t)2r = r2 B(t) = E(r,
~ dS
B ~ t) = r0 mI0 cos(t)
t t 2
2

c) Usando el mismo procedimiento anterior para un r b

2
~ =
~ dl
E ~ = E(r, t)2r = b2 B(t) = E(r,
~ dS
B ~ t) = b 0 mI0 cos(t)
t t 2r
2

~B

G2 ~E

G1
Figura 13.1: Senoloide
144 CAPTULO 13. LEY DE FARADAY-LENZ

Solucin 13.3 P X

La fuerza electromotriz inducida es

L x(t)
d d
= = B z dxdyz
dt dt
0 0

de este modo se obtiene que


 = BLx
Por otro lado, considerando que la fuente de corriente es capaz de mantener la corriente constante
(en el sentido de la figura) y sin alteraciones durante cada perodo de tiempo (ie. se ignora la corriente
adicional inducida en el circuito), se tiene que la velocidad de la barra est dada por

~ = I(t)dy y B z = BI(t)dyx = F~ = BI(t)Lx


dF

Entonces
v(t) t
dv BL
m = BLI(t) = dv = I(t)dt
dt m
0 0

Si t < 0
v(t) = 0
Si 0 < t T
t
BL BLbt2
v(t) = btdt =
m 2m
0

Por lo tanto se tiene que la fem inducida en el circuito es



0 t<0

(t) =

B 2 L2 bt2
0<tT
2m

Solucin 13.6 P X

a) La fuerza que mueve a la barra est determinada por el campo magntico B ~ = B0 z (saliendo
de la hoja) y la corriente que circula por el circuito. La corriente depender de la FEM inducida
de la siguiente forma:
 1 d B0 bx
I= = =
R R dt R
Luego la fuerza que siente la barra es

b
~ B B0 bx (B0 b)2 x
F~ = I dl ~ = dy y B0 z = x
R R
0
II. SOLUCIONES 145

Planteando la ecuacin de movimiento de la barra se tiene que


(B0 b)2 x dv (B0 b)2 v
mx = = m =
R dt R
Resolviendo la ecuacin diferencial
v(t) t  
mR dv (B0 b)2
= dt = v(t) = v0 exp t
(B0 b)2 v mR
v0 0

La posicin que se detiene la barra est determinada por (asumiendo que comienza en el origen)
  x(t) t  
dx (B0 b)2 (B0 b)2
v(t) = = v0 exp t = dx = v0 exp t dt
dt mR mR
0 0

de modo que   
v0 mR (B0 b)2
x(t) = 1 exp t
(B0 b)2 mR
Por lo que finalmente la barra en t se quedar en
v0 mR
xf inal =
(B0 b)2

b) En primera instancia, dado que hay una fuente de voltaje fija de valor V0 , existe una corriente
permanente
V0
I0 =
R
Donde se ha considerado positivo el sentido de la corriente antihoraria. La corriente I0 circula
en la direccin positiva debido a la forma de como est conectada la fuente de voltaje. La fuerza
que mueve a la barra est determinada por el campo magntico B ~ = B0 z (saliendo de la hoja)
y las corrientes que circulan por el circuito (corriente inducida y fija). La corriente inducida
depender de la fem inducida de la siguiente forma:
 1 d B0 bx
Iind = = =
R R dt R
Luego la fuerza que siente la barra es
b    
~ B B0 bx 0 (Bb)2 x V0 Bb
F~ = (Iind + I0 )dl ~ = + dy y B0 z = + x
R R R r
0

Planteando la ecuacin de movimiento de la barra se tiene que


(B0 b)2 x V0 B0 b dv (B0 b)2 v V 0 B0 b
mx = + = + =
R R dt mR mR
Resolviendo la ecuacin diferencial con factor integrante
 
dv (B0 b)2 t (B0 b)2 v (B0 b)2 t V0 B0 b (B0 b)2 t d (B0 b)2 t V0 B0 b (B0 b)2 t
e mR + e mR = e mR = v(t)e mR = e mR
dt mR mR dt mR
Finalmente integrando a ambos lados (e imponiendo que la barra parte del reposo)
t  
(B0 b)2 t V0 B0 b (B0 b)2 t V0
(B0 b)2
t
v(t)e mR = e mR dt = v(t) = 1e mR
mR B0 b
0
146 CAPTULO 13. LEY DE FARADAY-LENZ

Solucin 13.7 P X

Al igual que el Problema 13.1, se tiene que el campo magntico dentro la bobina es
~
B(t) = 0 nI0 sin tz
~ ~
En este caso se aplica la ley de Maxwell ~ = B =
~ E ~ E)
( ~ =
~ dS B ~
dS, usando
t t

el teorema de Stokes con la ltima expresin

~ ~ ~
~ dS
E dl = B
t

Asumiendo que E ~ = E(r, t) (el campo elctrico rota en torno a los campos magnticos variables en
el tiempo) y que es un circulo de radio r con normal z, se tiene que

~ ~ ~ = E(r, t) 2r = (0 nI0 sin t r2 ) = E
~ dS ~ = r 0 nI0 cos t
E dl = B
t t 2

Considerando que se trata de un material ohmnico, J~ = g E


~ , luego
gr
J~ = 0 nI0 cos t
2
Finalmente, la potencia que disipa el cilindro es
h 2 a  2
~ JdV
~ r gh(a2 0 I0 cos t)2
P = E = g 0 nI0 cos t rdrddz =
2 8
Cilindro 0 0 0

Solucin 13.11 P X

Dado que el disco esta girando con velocidad angular constante, todas sus cargas en su superficie
sienten una fuerza magntica (carga libre con velocidad bajo un campo magntico). El material crea
un campo elctrico al interior de l, debido a que desea que todas sus cargas se queden en el mismo
lugar y anule la fuerza magntica que se est produciendo. Por lo tanto

~
~ = Fm
E
q
~
= ~v B
= (~ ~
~r) B
= (k rr) B0 k
= B0 rr

Finalmente
R
~ =
~ dl B0 R2
|| = E B0 rdr =
2
0
Inductancia y Energa Magntica
14
I. Problemas Propuestos

Problema 14.1 X a
h
Encuentre el coeficiente de inductancia mutua entre un
alambre infinitamente largo y un tringulo issceles de
altura h dispuestos como se muestra en la Figura.

Problema 14.2 X S
Considere dos espiras de radios a y b (a  b) dispuesta d
de forma que sus centros se ubican en el eje z separadas
por una distancia d, como se muestra en la Figura. En-
cuentre el coeficiente de induccin mutua del sistema. a

Problema 14.3 X S
Dos cables infinitos llevan una corriente I y estn sepa- I
rados una distancia 2a como se muestra en la Figura.
Un anillo circular conductor de radio a yace en el mis-
mo plano entre entre los dos cables totalmente aislados
a 2a
de ellos. Encontrar el coeficiente de inductancia mutua
entre los cables y el anillo circular. Puede ser til
d arcsin x 1
= I
dx 1 x2

147
148 CAPTULO 14. INDUCTANCIA Y ENERGA MAGNTICA

Problema 14.4 X S
Considere una bobina muy larga de radio a y n vueltas
por unidad de largo. Al interior de la bobina, existe un
espira de radio b que forma un ngulo con respecto al
b a
eje de la bobina. a
a) Determine la inductancia mutua entre la bobina y
la espira.

b) Si por la espira empieza a circular un corriente


I(t) = I0 sin(t), cul es la mxima diferencia
de potencial en los terminales de la bobina?.

Problema 14.5 X S
Dos alambres infinitos estn separados una distancia d y
llevan una corriente elctrica I en direcciones opuestas
(ver Figura). La corriente I incrementa a una tasa dI/dt d
suficientemente baja de tal manera que la ley de Ampere
se cumple. Una espira cuadrada de lado d se encuentra
d
dispuesta en el mismo plano de los alambres, tal como
se muestra en la Figura.
a) Calcule la inductancia mutua entre la espira y los d
alambres.
I
b) Indique la direccin de la corriente inducida en d
la espira cuando dI/dt es positivo. Justifique su
respuesta. I

c) Sabiendo que la resistencia de la espira es r, cal-


cule la fuerza que acta sobre la espira (magnitud
y direccin).

Problema 14.6 X S b
a
Un cable coaxial est formado por dos superficies ci-
lndricas infinitamente largas, por las cuales circula una
misma corriente en sentidos contrarios. Si las densidades
superficiales de corriente que circulan por las superficies z
cilndricas de radios a y b son, respectivamente:

~ a = I0 z
K ~ b = I0 z
K ~b ~a
K
2a 2b K
Determine:
a) La energa por unidad de largo.

b) La autoinductancia por unidad de largo del cable


coaxial.
I. PROBLEMAS PROPUESTOS 149

Problema 14.7 X
Considere toroide de seccin transversal rectangular de
N
N vueltas, de radio interior a y exterior b, y altura h.
a) Determine la autoinductancia de este toroide.
h
b) Demuestre que si b a  a entonces la autoin-
ductancia del toroide puede ser aproximada como a
b
0 N 2 h(b a)
L=
2a

Problema 14.8 X S w
~ (t)

Un cilindro de radio R y largo H  R, posee una den- s0


sidad de carga 0 con su eje coincidente al eje z. En R
t = 0 el cilindro comienza a rotar lentamente con velo-
cidad angular ~ (t) = tz, donde es una constante.
Determine la energa elctrica y magntica dentro del
cilindro
H

Problema 14.9 X
z
w
~
Considere inicialmente una espira circular de radio a que
yace sobre el plano xz. En t = 0 la espira comienza
a girar con una velocidad angular ~ = 0 z. Si en el ~B
espacio existe un campo homogneo y constante de valor a
~ = B0 y determine:
B
a) La fem inducida en el circuito. y
b) La corriente en funcin del tiempo que circula por
la espira, si la espira posee una resistencia R y una
x
autoinduccin L.

c) El torque que siente la espira, suponiendo que sta


ha estado rotando un tiempo muy largo.

Problema 14.10 X ~B = B0 z
y
Un carrito de masa m que se desplaza con velocidad
v0 x, hasta llegar a una regin en que existe un campo ~v = v0 x
magntico uniforme en x = 0. El carrito es perfecta- L
mente conductor (ie. resistencia nula) y posee una au-
a
toinductancia L. Determine la velocidad del carrito en
el tiempo y la corriente que circula por ste. Cul debe x
ser la condicin sobre el largo del carrito para que ste
logre entrar completamente a la zona de campo?.
150 CAPTULO 14. INDUCTANCIA Y ENERGA MAGNTICA

Problema 14.11 X
Un alambre recto e infinito lleva una corriente I. Una
espira rectangular de lados a y b es coplanar con el alam-
bre y los lados a son paralelos a l. La espira se traslada I1
en la direccin z con rapidez v(z) = kz(z + b), donde
k es una constante. Determine I2
z R
a) Calcular el flujo que enlaza la espira debido a I1

+
y la fem inducida en ella. V0 L b
b) Escriba la ecuacin diferencial que debe satisfacer
la corriente I2 (t) que circula por la espira. Supo- a
niendo que ya calcul I2 (t).

c) Calcule la corriente I2 (t) si en t = 0 la corriente


I2 (0) = V0 /R

Problema 14.12 X
R1
Considere que interruptor S del circuito mostrado en la
Figura, ha estado abierto un tiempo muy largo. Deter- I1 I2 I3
mine las corrientes I1 , I2 e I3 que se muestran en la
figura, cuando:
+

V0 R2 L
a) Instantneamente una vez que el interruptor es
cerrado.
S
b) Un largo tiempo despus que el interruptor es ce-
rrado.

Problema 14.13 X S
I(t)
En el circuito de la Figura se ha tenido conectada la
batera por mucho tiempo (desde t = ) con el in-
terruptor S abierto. Luego en t = 0 el interruptor se
cierra. Considere que V0 = 120 V, R1 = R2 = 30 y R2
+

L = 50 mH. V0 R1
a) Determine por inspeccin la corriente I en t = 0
S
(justo antes de cerrar S) y en t = (mucho
despus de haber cerrado S) .

b) Obtenga una expresin analtica para I(t), entre L


0 t < . Grafique su resultado.
I. PROBLEMAS PROPUESTOS 151

Problema 14.14 X
Se debe determinar los valores de L y C en un circuito
RLC en serie cuando se conoce R. Para ello se carga
el condensador con una carga Q0 y se cierra el circuito
observndose que la corriente partiendo de cero, oscila L
con una frecuencia angular 0 y amplitud decreciente.
Si al circuito se le reemplaza la resistencia R por otra
de magnitud 2R, ste queda justo en el lmite cuando C
ya no oscila. R
a) Determine los valores de L y C.

b) Escriba la funcin Q(t) en funcin de t.

En ambos casos sus resultados deben quedar expresados


a lo ms en funcin de los datos R, Q0 y 0 .
152 CAPTULO 14. INDUCTANCIA Y ENERGA MAGNTICA

II. Soluciones

Solucin 14.2 P X

Dado que una espira es mucho ms pequea otra, se supondr que el campo magntico que pasa a
~ = d), donde B
travs de la espira de radio b es aproximadamente igual a B(z ~ es el campo magntico
que genera la espira de radio a. Usando la definicin de campo magntico

~ (~r ~r 0 )
~ = d) = 0
B(z
I dl
4 |~r ~r 0 |3
2
0 I1 ad (dz ar)
=
4 |~r ~r 0 |3
0
2
0 I1 a d (dz ar)
=
4 |dz ar|3
0
2
2
0 I1 a d dr a dz
= 3 + 3
4 2 2
(d + a ) 2 (d2 + a2 ) 2
0 0
2
0 I1 a2 dz
= 3
4 (d2 + a2 ) 2
0
0 I 1 a 2
= 3 z
2(d2 + a2 ) 2
Por lo tanto el el flujo en la espira de radio b est dado por
2
2 = B ~ = b2 0 I1 a
~ dS 3
2(d2 + a2 ) 2
Por lo que la inductancia mutua est dada por

2 0 (ab)2
M= = 3
I1 2(d2 + a2 ) 2

Solucin 14.3 P X

~ =
Sabiendo que el campo producido por un alambre infinito con corriente I vale B 0 I
2r , entonces a
una distancia r de uno de los dos cables se tiene que el campo magntico total es:
 
~ = 0 I z 1 1
B +
2 r 2a r
Donde z es el vector unitario que entra a la hoja, perpendicular a ella. De esta manera, se calcula el
flujo magntico, teniendo en cuenta que r = a + x donde x es la coordenada vertical en la figura, e
II. SOLUCIONES 153

y es la coordenada horizontal, con el origen ubicado al centro del anillo.



  a a2 x2  
~ dS
~= 0 I 1 1 0 I 1 1
= B + dydx = + dydx
2 r 2a r 2
a+x ax
a a2 x2

Luego:
a   p a
0 I 2a 20 Ia dx
= (2a a2 x2 )dx = = 20 Ia
2 a2 x2 a x2
2
a a

Y recordando la relacin de induccin mutua = M I, finalmente resulta:

M = 20 a

Solucin 14.4 P X

a) Para determinar la induccin mutua, se supone en primera instancia una corriente I1 que pasa
por la bobina y una corriente nula en la espira. Luego, es necesario determinar el flujo 12
que genera el campo magntico de la bobina sobre la espira. El campo magntico que genera
la bobina dentro de ella es
B~ = 0 nI1 z

y la normal de la espira (dado que est rotada con respecto al eje de la bobina) es

n = cos y + sin z

Por lo tanto el flujo es

2 b
12 = B ~ =
~ dS 0 nI1 z rdrd(cos y + sin z) = 0 nI1 b2 sin
0 0

Luego la induccin mutua es


12
M= = 0 nb2 sin
I1

n
a
z ~B

Figura 14.1: Representacin normal espira y campo magntico.


154 CAPTULO 14. INDUCTANCIA Y ENERGA MAGNTICA

b) La fem inducida en la bobina estar dado por  = ddt 21


. Donde 21 es el flujo de la espira
sobre la bobina. A priori, parece complicado determinar el valor del flujo 21 , sin embargo es
posible utilizar el calculo anterior, ya que por simetra de la inductancia mutua
12 21
M= = = 21 = M I2
I1 I2
Utilizando el hecho que I2 (t) = I0 sin t, se obtiene que
d21 dI2
= = M = 0 nb2 sin I0 cos t
dt dt
Finalmente, el mximo valor que alcanza la fem en la bobina (o equivalentemente, la diferencia
de potencial en sus terminales) es en t =

Vmax = 0 nI0 b2 sin

Solucin 14.5 P X

a) El campo magntico que genera cualquier alambre con corriente I es



~ = 0 I
~ dl
B

B(r) 2r = 0 I
~ 0 I
B(r) =
2r
Extrapolando ese resultado al problema, y considerando que el sistema de referencia est ubicado
con el alambre inferior y = 0

~ tot = B
B ~ cable2 = 0 I z +
~ cable1 + B 0 I
(z)
2y 2(y d)
Luego
d 3d  
~ =
~ tot dS 0 I 1 1
B z dxdyz
2 y yd
0 2d
   
0 I 3
= d ln ln(2)
2 2
 
0 I 3
= d ln
2 4
Finalmente como  
|| 0 d 4
M= = ln
I 2 3

b) Usando la Ley de Lenz y los resultados de la parte anterior = 2
0I
d ln 3
4 < 0.
Luego   
d 0 I 3 dI
= = d ln >0
dt 2 4 dt
| {z } |{z}
<0 >0

Como > 0, la corriente va en el sentido positivo de la normal z (antihorario).


II. SOLUCIONES 155

c) Como la resistencia de espira es r, la corriente que ciercula por ella es


 
0 d 3 dI
I1 = = ln
r 2r 4 dt

La fuerza puede calcularse en los cuatro lados de la espira.


Para los lados verticales la fuerza es simtrica, igual en mdulo pero en sentidos contrarios, por
ende se anulan.
Por otro lado
   
0 I 1 1 0 II1 1 1
~ ~
dF1 = I1 dl Btot (y = 2d) = I1 dxx z = dx(y)
2 y y d y=2d 2 2d d

Integrando en x entre 0 y d,
0 II1
= F~1 = y
4
Anlogamente para la seccin horizontal del cable superior, usando y = 3d
 
~ ~ 0 II1 1 1
dF3 = I1 dl Btot (y = 3d) = dx(y)
2 3d 2d

0 II1
= F~3 = y
12
Finalmente la fuerza est dada por la suma de F~1 + F~3
 
0 II1 0 I 0 d 3 dI
F~tot = y = ln y
6 6 2r 4 dt

Solucin 14.6 P X

Existen dos maneras de resolverlo, mediante la energa magntica o mediante el flujo magntico:

i) Energa magntica: Primero que todo, se calcula el campo magntico en el espacio mediante Ley
de Ampre en una superficie circular de radio r, perpendicular a los dos cascarones cilndricos.
De esta manera, para a < r < b:

~
B = B(r) = B ~ d~l = 2rB = 0 I

Luego, la corriente que atraviesa dicho camino es:



I = (K ~ a n) d~l

Donde n = r es el vector normal a la superficie cilndrica de radio a (superficie a travs de la


cul circula la corriente enlazada), y d~l = rd. Por lo tanto:

2
I0 ~ < r < b) = 0 I0
I= ad = I0 = B(a
2a 2r
0
156 CAPTULO 14. INDUCTANCIA Y ENERGA MAGNTICA

~ < a) = 0 debido a que no hay corrientes enlazadas para


Adems, se debe tener claro que B(r
~
r < a; y B(r > b) = 0 debido a que la suma de corrientes enlazadas es nula para r > b.
Ahora, se calcula la energa magntica asociada a una altura h arbitraria en los cilindros:

h 2 b  
1 ~ 2 dV = 20 I02 0 I02 h b
Um = |B| rddrdz = ln
20 4 2 r2 4 a
0 0 a

As, se obtiene la energa por unidad de largo:


 
Um 0 I02 b
um = = ln
h 4 a

Para calcular la autoinductancia por unidad de largo, se debe recordar que la energa magntica
asociada a una autoinductancia es (recordar que en este caso I = I0 ):
   
LI02 0 I02 h b 0 h b
Um = = ln = L = ln
2 4 a 2 a

Por lo tanto, la autoinductancia por unidad de largo vale:


 
L 0 b
l= = ln
h 2 a

ii) Flujo magntico: Usando los resultados anteriores del campo magntico, se calcula el flujo
~ de lado (b a) y
magntico a travs de una superficie rectangular, perpendicular al campo B,
altura h:
b h  
~ ~ 0 I 0 0 I 0 b
= B dS = drdz = ln
2r 2 a
a 0

Recordando que:
   
0 h b 0 b
= LI0 = L = ln = l = ln
2 a 2 a

Solucin 14.8 P X

~
Dado que la carga comienza a rotar, se genera una densidad superficial de corriente K(t) = 0 (t)R.
Este problema es anlogo al de una bobina con una corriente circulando por ella, por ende el campo
magntico fuera del cilindro es nulo. A partir de eso se obtiene por ley de Ampre:

B ~ = 0 I = B H = 0 KH = B
~ dl ~ = 0 0 tRz

El camino elegido es el mismo que se usa en bobina, un rectngulo de altura H. Luego, usando la ley
de Faraday para un disco de radio r < R

~ ~ d ~ = E(r, t) 2r = d (r2 0 0 tR) = E(r)
~ dS ~ 0 0 Rr
E dl = B =
dt dt 2
II. SOLUCIONES 157

El campo elctrico cambia debido a la induccin que provoca el campo magntico en el tiempo.
Finalmente, la energa magntica es

H 2 R
1 ~ 2 dV = 1 0 (0 tR)2 R2 H
Um = |B| (0 0 tR)2 rdrddz =
20 20 2
0 0 0

y la energa elctrica es

H 2 R  2
0 ~ 2 dV = 0 0 0 Rr 0 (0 0 R)2 R4
Ue = |E| rdrddz = H
2 2 2 4 4
0 0 0
158 CAPTULO 14. INDUCTANCIA Y ENERGA MAGNTICA

Solucin 14.13 P X

a) Para t = 0, el interruptor S ha estado abierto mucho tiempo, por cual la corriente que circula
en el circuito se vuelve constante (la inductancia en un tiempo muy extenso acta como un
cortocircuto, es decir, como un simple cable). Por ende la corriente est dada por

V0
I(t = 0) = = 4 [A].
R2
Para el caso que t , el interruptor se cierra y quedan las dos resistencias en paralelo. Al
igual que el caso anterior se tiene la corriente es
 
1 1
I(t ) = V0 + = 8 [A].
R1 R2

b) Usando un Kirchoff de Voltajes, se obtiene

dI dI R V0
V0 = IR + L = + I=
dt dt L L
Donde R = (R11 + R21 )1 = 15 . Usando factor integrante (ie. multiplicar a ambos lados
R
e L t)

dI R t R R t V0 R t d R V0 R t R V0 R t
e L + Ie L = e L = (I(t)e L t ) = e L = I(t)e L t I(0) = (e L 1)
dt L L dt L R
Reemplazando los valores de R e I(0), se obtiene que

V0 R t V0 R
I(t) = e L + (1 e L t ) = 8 4e300t
R2 R

I(t)
8

Figura 14.2: Grfico de I(t)


15
Corriente Alterna

I. Problemas Propuestos

Problema 15.1 X
Considere el circuito de corriente alterna mostrado en la
Figura. El circuito es alimentado con un generador de
voltaje V (t) = V0 cos t.
C R
a) Demuestre que el mdulo de la diferencia de po-
tencial entre los puntos A y B es independiente V0 A B
de la frecuencia del generador. R C
b) Calcule la fase de esta diferencia de potencial con
respecto al generador.

c) Explique cualitativamente como cambiaran sus


resultados si el generador tuviese una resistencia
interna Rint = R/10.

Problema 15.2 X
Un circuito RLC serie tiene R = 100 , L = 10 mH y
una frecuencia de resonancia de 0 = 3 kHz.
R
a) Cul es el valor de C?

b) Cul es valor de la impedancia total cuando la


frecuencia es de 0 = 5 kHz? V (t) L
c) Cul es la corriente de respuesta a un voltaje
V (t) = 200 cos(104 t)?

d) Cul es la potencia promedio consumida? C


e) Cul es el mximo voltaje en cada uno de los tres
elementos de circuito? Por qu estos nmeros no
suman 200 [V]?

159
160 CAPTULO 15. CORRIENTE ALTERNA

Problema 15.3 X S
Considere el circuito elctrico de la figura, que es alimen-
tado por una fuente de voltaje alterno V (t) = V0 cos t.
Determine: R
a) La impedancia equivalente de la configuracin, es-
pecificando claramente su parte real e imaginaria.
L
b) La frecuencia de resonancia 0 del circuito, tal que V0 C2
haga mxima la corriente que sale de la fuente.
C1
c) La corriente que circula por la resistencia R en
funcin del tiempo (ie. escriba la corriente de la
forma I(t) = I0 cos (t + ), especificando clara-
mente los valores de I0 y ).

d) La potencia promedio entregada por la fuente.

[220 V] [220 V]
Problema 15.4 X [220 V] [110 W] [1100 W]
[1,5 A]
En la figura se esquematiza la conexin de un compu-
tador personal, una lampara de escritorio y una estufa
elctrica a un enchufe mltiple (alargador) el cual es
conectado al enchufe de 220 V en la muralla. Las es-
pecificaciones de cada artculo aparecen en la figura. El
enchufe E conectado a la muralla tolera una corriente E
mxima de 7 A. Examine cuantitativamente la conexin
y determine si el enchufe E es el adecuado.

Problema 15.5 X S
Un circuito tiene dos resistencias R1 y R2 , un con-
densador C y una inductancia L dispuestas de la for-
ma que se indica en la Figura. El circuito es alimenta-
do mediante una fuente de voltaje alterno de ecuacin
V (t) = V0 sin t. Determine R1 R2
a) La impedancia equivalente del circuito y el mdulo V0
de la corriente en funcin del tiempo por cada L C
rama del circuito.

b) La frecuencia de resonancia del circuito, Cul es


el valor de la corriente mxima que sale de la fuen-
te? (exprese su resultado en funcin de la frecuen-
cia de resonancia).
I. PROBLEMAS PROPUESTOS 161

Problema 15.6 X
Los enchufes elctricos chilenos proveen una fuente de
voltaje alterno con Vrms = 220 [V] y una frecuencia
f = 50 [Hz], por otro lado, los enchufes en Estados
Unidos usan Vrms = 120 [V] y una frecuencia f = 60
[Hz]. Si se trae una ampolleta de 60 [W] y 120 [V] de
Estados Unidos a Chile, Cun grande debe ser una in-
ductancia L, en henrys, que debe ser conectada en serie
tal que la ampolleta opere normalmente, produciendo la
misma cantidad de luz como si fuese conectada en Esta-
dos Unidos?. Explique brevemente por qu es ventajoso
corregir la iluminacin con una inductancia en vez de
una resistencia.

Problema 15.7 X S
C R2
Para el circuito en la figura, muestre si
V0 A B
L
R1 R2 = R1 L
C
entonces VA = VB sin importar la frecuencia . Ambos
potenciales estn medidos de la tierra del circuito.

Problema 15.8 X
El circuito de la figura tiene una fuente de voltaje al-
terno V (t) = V0 cos t , conectada en serie con una
resistencia R = 1 y una caja negra, que puede con-
tiene un condensador, un inductor, o ambos. Se mide la
corriente que circula en el circuito con una frecuencia R
angular 1 = 1 rad/s y se observa que la corriente ade-
lanta al voltaje de la fuente en una fase de /4 radianes.
Si se repite el experimento con una frecuencia a 2 = 2
rad/s se encuentra la corriente se atrasa exactamente V0 ?
/4 radianes con respecto a la fuente.
a) Qu hay dentro de la caja negra?.

b) Cul/es es/son los valor/es de inductancia o/y


capacitancia?.

c) Puede ser el circuito resonante?, en caso de serlo,


cul es la frecuencia de resonancia?.
162 CAPTULO 15. CORRIENTE ALTERNA

Problema 15.9 X

a) Un circuito RLC serie contiene una inductancia R


de 0.1 H. y una resistencia y capacitancia de valor
desconocido. Cuando el circuito es conectado a
una fuente AC V (t) = 230 sin 100t, la corriente
que fluye por el circuito es I(t) = 2,3 sin 100t. V0 L
Encuentre: los valores de la resistencia y la capaci-
tancia, la diferencia de potencial en la inductancia
y la potencia promedio total consumida.

b) Un circuito resonante en paralelo consiste en un C


condensador C = 32 pF en paralelo con una in- (a) RLC Serie
ductancia F = 18 H y una resistencia R = 60
, respectivamente. Determine la frecuencia de
resonancia del circuito y el mdulo de la impe-
dancia equivalente en funcin de la frecuencia
del generador.

c) Un generador entrega un voltaje constante con V0 R L C


frecuencia variable de Vrms = 100 [V] (rms) a un
circuito RLC serie que contiene R = 5, L = 4
mH, y C = 0.1 F . La frecuencia del generador
se vara hasta que se obtiene el mximo valor de
la corriente. Determine el valor de la corriente m- (b) RLC Paralelo
xima, la frecuencia en la cual ocurre, y el voltaje
entre la inductancia y el condensador.

Problema 15.10 X
Suponga usted que cuenta con dos herramientas que
operan 110 V. Una de ellas es un taladro elctrico cuyas
especificaciones son (110 V - 2,6 A). La otra herramien-
ta en una sierra elctrica de especificaciones (110 V - 3
A). Para utilizar estas herramientas requiere un trans-
formador que reduzca el voltaje de 220 V a 110 V. Estos
aparatos se encuentran en el comercio y los hay de 50
W, 100 W, 800 W, 1000 W, 1500 W. Mientras mayor es
la potencia tolerable por el transformador mayor es su
precio. Determine el transformador ms econmico que
permita la operacin simultnea de ambas herramientas.
I. PROBLEMAS PROPUESTOS 163

Caso Tpico Sonido Original

Computador

Problema 15.11 X
Los filtros de seales son comnmente usados para ate- Tarjeta de Sonido
nuar algunas frecuencias presentes en las mismas. Con-
sidere el ejemplo de la figura, donde se conecta una
resistencia R y condensador C en los terminales de
la tarjeta de sonido un computador. Por simplicidad,
suponga que entre los puntos A y B hay un voltaje Micrfono

V (t) = V0 cos(2f t) (seal de entrada). Caso (I) Sonido (I)


a) Encuentre la relacin |VEF |2 /|VAB |2para los dos
casos mostrados en la figura (asuma la impedancia Computador

del micrfono es nula).

b) Escuche los sonidos que se indican a continuacin Tarjeta de Sonido


(tambin puede hacerlo escaneando el cdigo QR A B
de la figura):
C

Sonido Original
E F
R
Sonido 1 Micrfono

Sonido 2 Caso (II) Sonido (II)

Usando los resultados de la parte (a) y habiendo Computador

escuchado los sonidos, determine a qu sonido de


filtro corresponden los Casos (I) y (II). Recuerde
que el odo humano escucha las altas frecuencias Tarjeta de Sonido

en forma aguda y las baja frecuencia en forma A B

grave.
R

E F
Micrfono C
164 CAPTULO 15. CORRIENTE ALTERNA

II. Soluciones

Solucin 15.3 P X

a) La impedancia equivalente est dada por


 1
1 1
Z =R+ +
jXL jXC1 jXC2
 
jXC2 + jXL jXC1 1
=R+
(XL XC1 )XC2
(XL XC1 )XC2
=R+j
XC2 + XC1 XL
2 LC1 1
=R+j
(C1 + C2 2 LC1 C2 )

b) La frecuencia de resonancia es tal que la parte imaginaria de la impedancia se vuelve nula, es


decir
1 1
XL = XC1 = 0 L = = 0 =
0 C1 LC1
Ntese el hecho que al elegir esta frecuencia, el mdulo de la impedancia se vuelve mnimo, y
en consecuencia, se maxima el mdulo de la corriente que sale de la fuente.

c) La corriente que circula por la resistencia es


   
V0 ejt V0 ejt V0
I(t) = Re = Re = cos(t )
Z |Z|ej |Z|

donde de la parte anterior se tiene que los valores de |Z| y son


s  2
2 LC1 1
|Z| = R2 +
(C1 + C2 2 LC1 C2 )

 
2 LC1 1
= arctan
R(C1 + C2 2 LC1 C2 )

d) Finalmente, la potencia media que entrega la fuente es

V02
hP i = cos
2|Z|
V2
= 0 2 Re{Z}
2|Z|
V02 R
=  2
2 LC1 1
2R2 + 2 (C1 +C 2
2 LC1 C2 )
II. SOLUCIONES 165

Solucin 15.5 P X

a) La impedancia equivalente est dada por


 1
1 1
Zeq = +
R1 + ZL R2 + ZC

1
donde ZL = jXL y ZC = jXC , con XL = L y XC = C . Luego
 1
1 1
Zeq = +
R1 + jXL R2 jXC
(R1 + jXL )(R2 jXC )
=
(R1 + R2 ) + j(XL XC )
(R1 R2 + XL XC ) + j(XL R2 XC R1 )
=
(R1 + R2 ) + j(XL XC )
[(R1 R2 + XL XC ) + j(XL R2 XC R1 )][(R1 + R2 ) + j(XL XC )]
=
(R1 + R2 )2 + (XL XC )2
(R1 R2 + XL XC )(R1 + R2 ) (XL R2 XC R1 )(XL XC )
=
(R1 + R2 )2 + (XL XC )2
(R1 R2 + XL XC )(XL XC ) + (XL R2 XC R1 )(R1 + R2 )
+j
(R1 + R2 )2 + (XL XC )2

Los mdulos de cada una de las corrientes vale



V0 ejt V V ejt V0
0 0
|I1 (t)| = = p
|I2 (t)| = 1 = q
R1 + jL 2
R1 + L2 2 R1 j C 1 2
R22 + ( C )

Ntese que los mdulos de las corrientes son constantes en el tiempo y slo dependen de los
parmetros fijos del circuito. El aporte temporal est dado netamente por la parte exponencial
compleja de la corriente.

b) La frecuencia de resonancia puede ser calculada al hacer nula la parte imaginaria de la impe-
dancia, de modo que

(R1 R2 + XL XC )(XL XC ) + (XL R2 XC R1 )(R1 + R2 ) = 0

Reemplazando los valores de XL y XC


    
L 1 R1
R1 R2 + 0 L = 0 LR2 (R1 + R2 )
C 0 C 0 C
donde puede despejarse el valor de 0 como
s
1 R12 C L
0 =
LC R22 C L

El valor de la corriente para esa frecuencia vale

V |V0 | (R1 + R2 )2 + (0 L 01C )2


I= = |I| = = V0 L 2
Zeq |Zeq | (R1 R2 + C ) + (0 LR2 01C R1 )2
166 CAPTULO 15. CORRIENTE ALTERNA

Solucin 15.7 P X

Previamente, antes de resolver el problema, es bueno conocer el concepto de Divisor de Voltaje


que es sumamente til en problemas de corriente alterna.

Z1 Vout

Vin Z2

Figura 15.1: Divisor de Voltaje con Impedancias

En la Figura 15.1 es posible apreciar una malla de un circuito CA que posee dos impedancias en
serie Z1 y Z2 . El divisor de voltaje permite obtener el voltaje el punto Vout , que es justo antes
de donde comienza la impedancia Z2 . Ntese que Vout representa la cada de tensin al pasar
la corriente por Z1 . Para determinar el valor de Vout se debe considerar que Vin = I (Z1 + Z2 )
y Vout = I Z2 , juntando ambas ecuaciones se llega a
Z2
Vout = Vin
Z1 + Z2

Volviendo al problema, usando notacin fasorial para la fuente V (t) = V0 ejt , se tiene que
usando la frmula del divisor de voltaje para la rama que contiene a C y R1 :
ZR1 R1 1
VA = V0 ejt = V0 ejt j
= V0 ejt
Zc + ZR1 R1 C 1 R1jC

De la misma manera, para la rama que contiene a R2 y L:


jL 1
VB = V0 ejt = V0 ejt
R2 + jL 1 jR
L
2

L L
Luego, usando el dato que R1 R2 = C o equivalentemente R1 C = R2 , se llega a que

1 1
VA = V0 ejt j
= V0 ejt jR2
= VB
1 R1 C 1 L

que era lo que se quera demostrar.


Leyes de Maxwell y Ondas Electromagnticas
16
I. Problemas Propuestos

Problema 16.1 X S

a) Explique qu es una onda plana monocromtica


e identifique explcitamente la direccin de propa-
gacin, la velocidad de propagacin, la frecuencia,
y la longitud de onda.

b) En el caso que la onda plana sea electromagn-


tica, demuestre que los campos E ~ yB ~ son per-
pendiculares entre si y (ambos) perpendiculares
a la direccin de propagacin. Indicacin: use las
ecuaciones de Maxwell.
Problema 16.2 X
Considere una onda electromagntica que posee el si-
guiente campo elctrico
~ = E0 cos[(10 m1 ) x + (3 109 s1 ) t]z
E

Encuentre:
a) La longitud de onda y el periodo T .

b) La direccin y el sentido de propagacin.


~ asociado a esta onda.
c) El campo magntico B
~
d) El vector de Poynting S.

Problema 16.3 X S
Escriba una expresin para el campo elctrico y magn-
tico de una onda electromagntica plana, cuyo campo
elctrico est contenido en el plano xy y se propaga
desde el origen hasta el punto (0, 3, 3). La frecuencia
angular de la onda es = 100 [rad/s] y la veloci-
dad de la luz es c. La magnitud del campo elctrico es
E0 . Encuentre la potencia promedio que transporta esta
onda

167
168 CAPTULO 16. LEYES DE MAXWELL Y ONDAS ELECTROMAGNTICAS

Problema 16.4 X
Al interior de una gua de ondas se transmite una onda
electromagntica (solucin de las ecuaciones de Maxwell
en ausencia de cargas y corrientes = 0 y J~ = 0) tal
que las nicas componentes del campo magntico son B ~
son Bx y Bz . Su expresin en funcin de las coordenadas y
x, z y del tiempo t es:
aB0 k  x 
Bx = sin sin(kz t)
a
 x 
Bz = B0 cos cos(kz t)
a
x
en donde a, B0 , k y son constantes. La relacin entre
k y es z
r
2 2
k= 2
c2 a
Se sabe que el campo elctrico E~ slo tiene componente
Ey que es funcin solamente de las coordenadas x, z,
t. Encuentre la expresin para este Ey .

Problema 16.5 X
Considere una superposicin de ondas viajando en el la
direccin del eje z, donde

E~ = Re{x E1 ej(kzt) + y E2 ej(kzt) }


 
~ E1 j(kzt) E2 j(kzt)
B = Re y e x e
c c
donde E1 y E2 pueden ser complejos

E1 = C1 ej1 , E2 = C2 ej2

C1 donde y C2 son reales.


a) Calcule el flujo promedio de energa hSi.

b) Suponga que E1 = C y E2 = jC, es decir, C1 =


C2 = C y 1 = 0, 2 = 2 . Determine la direccin
~ en funcin de t, en un punto de plano xy.
de E
Describa el resultado mediante palabras y dibujos.

c) Para el mismo campo en la parte b, determine la


~ en funcin de z, para el instante en
direccin de E
t = 0. Describe el resultado en palabras y dibujos.
I. PROBLEMAS PROPUESTOS 169

Problema 16.6 X S
Dos ondas electromagnticas son emitidas desde dos di-
ferentes fuentes, de modo que
~ 1 (x, t) = E10 cos(kx t)y
E
~ 2 (x, t) = E20 cos(kx t + )y
E c c
y

a) Encuentre el vector de Poynting asociado a la on-


da electromagntica resultante.
x

b) Encuentre la intensidad de la onda.

c) Repita el calculo si la direccin de propagacin


de la segunda onda electromagntica viaja en la
direccin contraria a la primera
~ 1 (x, t) = E10 cos(kx t)y
E
~ 2 (x, t) = E20 cos(kx + t + )y
E

Problema 16.7 X
Una onda electromagntica plana propagndose en el
vaco tiene un campo magntico dado por


~ = B0 f (ax + bt)y 1 si 0 < u < 1
B f (u) =
0 si

Donde a y b son constantes positivas.


a) Cul debe ser la relacin entre a y b para cumplir
con las Ecuaciones de Maxwell?
~ de esta onda electro-
b) Determine el valor de |E|
magntica. En qu direccin y sentido se propaga
la onda?.

c) Determine la magnitud y la direccin del flujo de


energa llevada por la onda. Exprese su resultado
en funcin de B0 y constantes universales.

d) Esta onda choca perfectamente sobre un plano


conductor y es reflejada. Cul es la presin (fuer-
za por unidad de rea) que ejerce la onda mientras
impacta el plano?.
170 CAPTULO 16. LEYES DE MAXWELL Y ONDAS ELECTROMAGNTICAS

Problema 16.8 X
Una onda plana linealmente polarizada se propaga a lo
largo de la direccin 12 (x + y). La polarizacin de la
onda yace sobre el plano XY (ie, el campo elctrico vive
en el plano XY ).Si el mdulo de campo elctrico es E0
y la frecuencia de la onda es , encuentre
a) Las funciones reales que representan E ~ y B.
~

b) El promedio temporal del vector de Poynting.

Problema 16.9 X
En un buen conductor, cuando la frecuencia no es de-
masiado grande, una buena aproximacin consiste en
despreciar el trmino de la corriente desplazamiento. En
tal caso la ecuacin para E~ es

~
E
~ 0 g
2 E =0
t
Considere el caso de una onda plana que ha penetrado
a un medio (interfaz: plano XY ) con conductividad g.
a) Tomando ~k = (0,p 0, k) demuestre de lo anterior
que k = (1 + j) 0p g/2. En lo que sigue se
usar la notacin k0 = 0 g/2

b) Para el campo elctrico de una onda plana en el


medio conductor, luego de una incidencia normal,
~ = E0 ej(kzt) x, (convencionalmente escoja
es E
~
E0 real). Determine el valor de B.

c) Calcule
D E el flujo temporal del vector de Poynting
~
S
II. SOLUCIONES 171

II. Soluciones

Solucin 16.1 P X

a) Una onda plana es una onda tal que se propaga en una sola direccin en el espacio, o equivalen-
temente, su frentes de ondas son infinitos planos perpendiculares a la direccin de propagacin.
Un onda plana es monocromtica debido a que posee una nica frecuencia de propagacin. La
forma matemtica para expresarla (solucin de la ecuacin de ondas) es
~
A(~r, t) = Aej(k~rt)
donde:
La direccin de propagacin est dada por el vector ~k,
La velocidad de onda puede ser obtenida al reemplazar la expresin anterior en la ecuacin
de ondas, resultando v = ~ .
|k|
La frecuencia angular est dada por = 2f , donde f es la frecuencia.
2
La longitud de onda est dada a partir del mdulo del nmero onda como |k| = .

b) Se debe considerar un campo elctrico arbitrario como


~ 0 ej(~k~rt) = (E0x x + E0y y + E0z z)ej(~k~rt)
~ =E
E

Reescribiendo ~k = kx x + ky y + kz z se tiene que E


~ = (E0x x + E0y y + E0z z)ej(kx x+ky y+kz zt) .
Tomando la divergencia de este campo

~ = Ex + Ey + Ez = j (kx E0x + ky E0y + kz E0z )ej(kx x+ky y+kz zt)


~ E

x y z | {z }
~kE
~

~ E
Por la primera ley de Maxwell, se sabe que ~ ~k = 0 = E
~ = 0 = E ~ ~k, de igual forma
~ ~ ~ ~
como B = 0 = B k. Por otro lado
     
~ ~ Ez Ey Ex Ez Ey Ex
E = x + y + z
y z z x x y
| {z } | {z } | {z }
j(ky E0z kz E0y )ej(~k~rt) j(kz E0x kx E0z )ej(~k~rt) j(kx E0y ky E0x )ej(~k~rt)

Por lo tanto
~ = j [(ky E0z kz E0y )x + (kz E0x kx E0z )y + (kx E0y ky E0x )z] ej(~k~rt) = j~k E
~ E ~
| {z }
~kE
~0

~ =B
Suponiendo que el campo magntico tambin es de la forma B ~ 0 ej(~k~rt) y aplicando la
Ley de Faraday
~ ~ ~
~ E
~ = B = j~k E
~ = j B~ = B~ = kE
t
~ a ambos lados
Aplicando producto punto con E
~ ~ ~ ~
~ B
E ~ =E~ k E = ~k E E = 0

~ B
Por lo que se deduce finalmente que E ~ ~k.
172 CAPTULO 16. LEYES DE MAXWELL Y ONDAS ELECTROMAGNTICAS

Solucin 16.3 P X

a) Se debe recordar que la forma general del campo elctrico de una onda electromagntica es en
este caso:
~ = E0 nej(~k~rt)
E

Donde ~r = (x, y, z) y ~k = k k indica la direccin de propagacin de la onda y n, la polarizacin


del campo elctrico. En este caso la direccin de propagacin de la onda es k (unitario) y est
determinado por
1 1 y + z
k = (0, 3, 3) = (0, 1, 1) =
2
3 +3 2 2 2
Por otro lado se sabe que el campo elctrico est polarizado en el plano xy por lo que

1
n = p (, , 0)
2 + 2

Ahora usando el hecho que la direccin de propagacin es siempre perpendicular a la polarizacin


de la onda se cumple que

k n = 0 = (0, 1, 1) (, , 0) = 0 = = 0

Por ende
1
n = (, 0, 0) = (1, 0, 0) = x
2
El campo elctrico es entonces
k
~ = E0 xej( 2 (y+z)t)
E

El valor de k se determina volviendo a la ecuacin de onda

1 2E~ ~
2E ~
2E 1 2E~ k2 k2 2
~
2 E = + = 0 = + 2 = 0 = k =
c2 t2 y 2 z 2 c2 t2 2 2 c c

~ puede ser determinado como


Finalmente el campo magntico B
k
~ j( (y+z)t)
~ k E (y + z) E0 xe 2 E0 j( k (y+z)t)
B= = = (y z)e 2
c c 2 c 2

En resumen como = 100rad/s :

~ = E
y+z
~ = E0 xej100( c2 t) j100( y+z
t)
E B 0 (y z)e c 2
c 2

b) El vector de Poynting Promedio est dado por



~ 1 ~ ~ 1 j( y+z
t) E0 t)
j( y+z = E
2
0 E02
h|S|i = |EB | = E xe c 2 (y z)e c 2 |(y+z)| =
20 c 2
0
20 c 2 0 0 c
II. SOLUCIONES 173

Solucin 16.6 P X

~
a) Para determinar el vector de Poynting es necesario determinar el valor del campo magntico B.
Luego para el campo E~1 se tiene que

x y z



~1 =
~ E

x y z = E10 k sin(kx t)z


0 E10 cos(kx t) 0

y dado que por Maxwell

B~1
~ 1 (x, t) = E10 k cos(kx t)z = E10 cos(kx t)z
= k sin(kx t)z = B
t c
~ 2 , por lo tanto por principio de superposicin se tiene
El clculo es anlogo para determinar B
que
E~ = [E10 cos(kx t) + E20 cos(kx t + )]y

B~ = 1 [E10 cos(kx t) + E20 cos(kx t + )]z


c
Donde finalmente el vector de Poiynting vale

~ ~
~ = E B = 1 [E10 cos(kx t) + E20 cos(kx t + )]2 x
S
0 c0

b) La intensidad de la onda est dada por

T
1 1
I = hSi = [E10 cos(kx t) + E20 cos(kx t + )]2 dt
T c0
0

Antes de comenzar a trabajar la expresin debe usarse el hecho que

T
1 1
cos2 (kx t + ) =
T 2
0

Por lo tanto

T T
1 2
I= E10 cos (kx t + )dt + E20 cos2 (kx t)dt
2 2
c0 T
0 0

T

+2E10 E20 cos (kx t + ) cos (kx t)dt


0

2 2 T
1 E10 E20 2E10 E20
= + + cos (kx t + ) cos (kx t)dt
c0 2 2 T
0
174 CAPTULO 16. LEYES DE MAXWELL Y ONDAS ELECTROMAGNTICAS

Para la ltima integral se tiene que

T T
1 1
cos (kx t + ) cos (kx t)dt = [cos (kx t) cos sin (kx t) sin ] cos (kx t)dt
T T
0 0
T T
cos 2 sin
= cos (kx t)dt + cos (kx t) sin (kx t)dt
T T
0 0
T
cos sin
= + sin (2(kx t))dt
2 2T
0
cos
=
2
Finalmente la intensidad vale
 2 2

1 E10 E20
I= + + E10 E20 cos
c0 2 2

c) Repitiendo nuevamente los clculos de parte anterior se tiene que el campo magntico asociado
aE~ 1 se mantiene, mientras tanto el campo asociado a E ~ 2 vale

x y z



~ E
~2 =
x y z = E20 k sin(kx + t)z


0 E cos(kx + t + ) 0
20

Nuevamente por Maxwell,

B~2
~ 2 (x, t) = E20 cos(kx + t + )z
= E20 k sin(kx + t + )z = B
t c
donde se tiene que
~ = [E10 cos(kx t) + E20 cos(kx + t + )]y
E

~ = 1 [E10 cos(kx t) E20 cos(kx + t + )]z


B
c
lo que implica que en este caso

~ = 1 [E10
S 2
cos2 (kx t) E20
2
cos2 (kx + t + )]x
c0

Por lo que la intensidad de la onda vale

T  2 2

1 1 2 2 2 2 1 E10 E20
I= [E cos (kx t) E20 cos (kx + t + )]dt =
T c0 10 c0 2 2
0
Parte V

Campos Electromagnticos en
Medios Materiales

175
Campo Elctrico en Medios Materiales
17
I. Problemas Propuestos

Problema 17.1 X S
Considere una esfera conductora de radio R1 , cargada
con Q. La cual est rodeada de un manto dielctrico de e
permitividad  y radio R2 (con respecto al centro de la
esfera), el resto del espacio est vaco. Determine:
R1
a) El campo elctrico en todo el espacio.
Q R2
b) Las densidades de carga libre e inducidas por la
polarizacin en las interfaces (cambios de medio).

c) La diferencia de potencial entre la esfera conduc-


tora e infinito. Aumenta o disminuye est tensin
debido producto a la presencia del dielctrico?.

Problema 17.2 X S
Una esfera conductora de radio R flota hasta la mitad
en un medio dielctrico de permitividad elctrica 1 . La
regin sobre la esfera es un gas de permitividad elctrica
2 . La esfera est cargada con una carga igual a Q.
a) Calcule el campo elctrico en todo el espacio. e2

b) Calcule la densidad de carga libre superficial en R


la esfera conductora, y la densidad superficial de
carga de polarizacin de ambos medios dielctri-
cos en la interfaz con la esfera. e1
c) Si el medio en el cual flota la esfera es agua (1
790 ) y el gas que hay sobre ella es aire (2 0 ).
Obtenga la relacin (en mdulo) entre la carga
total de polarizacin en la interfaz con la esfera
y la carga libre de la esfera, qu signo tiene la
carga neta sobre la esfera? .

177
178 CAPTULO 17. CAMPO ELCTRICO EN MEDIOS MATERIALES

b
a
Problema 17.3 X S
Dielctrico 1
En un condensador cilndrico de radios a y b y de largo
L ( a, b) se han puesto dos dielctricos tal como se
muestra la Figura. Si el condensador posee carga +Q y
Q en sus superficies y los dielctricos poseen permi-
e1
tividades 1 y 2 (1 < 2 ), calcule la fuerza necesaria Condensador L
que hay que aplicar al sistema para que el dielctrico de e2
permitividad 1 permanezca una distancia 12 L dentro del
condensador. Considere que ambos dielctricos son muy
largos y se extienden ms all del largo del condensador.
Dielctrico 2

Problema 17.4 X S
Se tiene dos placas conductoras paralelas muy grandes,
separadas una distancia d. El espacio entre las placas x
est lleno con un material dielctrico de permitividad
(x), tal que (x = 0) = 1 . Se encuentra que el poten-
cial electrosttico entre las placas satisface la ecuacin
diferencial
d2 V dV
+k =0
dx2 dx e(x)
donde k es una constante positiva.
a) El potencial electrosttico V (x) dentro de las pla- + V0
cas.
x=0 x=d
b) La permitividad (x).

c) La densidad de carga libre que reside en la super-


ficie conductora a potencial V0 .

Problema 17.5 X
El espacio entre dos disco discos paralelos conductores R
de radio R y separados una distancia d  R es puesto
un dielctrico no uniforme, de modo que su permitividad
vara linealmente en funcin de la distancia del centro
d e(r)
de los discos
r
(r) = 1 + (2 1 )
R
Calcule la capacitancia que forma este sistema.
I. PROBLEMAS PROPUESTOS 179

Problema 17.6 X S
Un dispositivo de uso frecuente en muchos artefactos
elctricos de uso diario son los condensadores variables.
Este tipo de condensador posee una geometra que es
fcil de modificar mecnicamente, con lo cual se puede
ajustar su capacidad a gusto. Existen muchos diseos de
este tipo de dispositivos, un ejemplo simple e idealizado
es el que se muestra en la Figura. El cual consiste en
dos placas paralelas y cuadradas de largo a y separadas
una distancia b. Entremedio de las cuales se introdu-
ce parcialmente un dielctrico de permitividad , el cual a
penetra una distancia x de uno de los bordes del con-
densador. Suponga que dispone de una batera que da b e

+
V0
un voltaje V0 , y desea almacenar una carga Q en el
condensador. Para ajustarlo, calcule x

a) La capacidad del condensador para un x dado,


despreciando los efectos de borde (i.e. b  a, x) .

b) La penetracin x del dielctrico para el conden-


sador tenga una carga Q (dado el voltaje de la
batera V0 ).

c) Cul es la fuerza que siente el condensador en


esa posicin?.

d) Cul es el rango de carga Q que puede ser al-


macenado en el condensador?.

X S
r(r)
Problema 17.7
Una esfera dielctrica de radio b y permitividad de carga
 contienen en un interior una densidad de carga libre

0 r 0 < r < b
(r) =
0 r>b b
a) Calcule las densidades de carga de polarizacin en
volumen p y superficial p en las zonas donde e
existan.

b) Calcule la energa electrosttica del sistema de


cargas.
180 CAPTULO 17. CAMPO ELCTRICO EN MEDIOS MATERIALES

s
Problema 17.8 X e1 e2
Suponga una zona en el espacio separada por dos sec-
tores: al izquierda un sector con permitividad 1 y a
la derecha 2 . Un campo elctrico E ~ 1 (con sus corres-
pondientes vectores D ~ 1 y P~1 ) incide sobre la zona de q1 q
2
izquierda, emergiendo un campo E ~ 2 , a la derecha. Los
ngulos de incidencia son los que se muestran en la fi-
gura. En la interfaz, hay una densidad de carga libre . ~E1 ~P1 ~D1 ~E2 ~P2 ~D2
Determine P~2 en funcin de las permitividades elctri-
cas, el ngulo de incidencia 1 y la densidad de carga
.

Problema 17.9 X
d1 d2
Un condensador de placas paralelas y rea A tiene una
lmina dielctrica de espesor d2 y permitividad elctrica
 adosada a una de sus placas. Un espacio vaco de an-
cho d1 separa al dielctrico de la otra placa conductora.
La superficie del dielctrico en contacto con el vaco es-
e0 e
t cargada con una densidad superficial de carga libre
uniforme 0 . Esta carga no puede moverse. Las placas
estn conectadas a travs de una resistencia R, tal como
se indica en la figura. Desprecie efectos de borde.
a) Cuando ambas placas de condensador estn en
reposo, calcule la densidad de carga sobre cada s0
conductora.

b) Considere que la placa de izquierda se acerca al


R
dielctrico con una velocidad constante v0 . Calcu-
le la diferencia de potencial que genera entre los
extremos de la resistencia en funcin de la distan-
cia d1 y el resto de los datos del problema.

Problema 17.10 X
a
Un largo y slido dielctrico de forma cilndrica de radio
a es est permanentemente polarizado tal que su vector
de polarizacin est dado por la expresin P~ = 21 P0 rr.
a) Encuentre las densidades de cargas de polariza- w
cin.

b) El campo elctrico en todo el espacio.

c) Si el cilindro empieza a rotar con velocidad angular


(con respecto a su eje) , determine el campo
magntico en eje del cilindro.
I. PROBLEMAS PROPUESTOS 181

Problema 17.11 X S
Considere dos condensadores de placas cuadradas de la-
do 2a separadas una distancia 2d. Dentro de cada con-
densador existen dos medios con constantes dielctricas
y conductividades, 1 , g1 y 2 , g2 . Los medios llenan la
mitad del volumen de cada condensador, pero de una +

disposicin distinta en cada uno (ver figura). Despre- V0


ciando todos los efectos de borde:
a) Para cada condensador determine el vector den-
~ vector campo elctrico E
sidad de corriente J, ~ y d e1 , g 1
~
vector desplazamiento D dentro de l. e1 , g 1 e2 , g2 2d
d e2 , g2

b) Para el condensador de la derecha, determine las a a


densidades de polarizacin y carga libre donde co-
rrespondan.

c) Determine la capacitancia equivalente que forman


ambos condensadores.

d) Determine la corriente que sale por la fuente y la


resistencia equivalente del sistema.

a
2a
Problema 17.12 X
3a
Dos condensadores cilndricos de radio interior a y exte- e1
rior 3a, y largo L, han sido llenados con dos materiales e2
dielctricos 1 y 2 de distinta forma (ver figura). Si los
condensadores son conectados de la forma que se indica
en la figura, determine la capacidad equivalente entre
los puntos A y B.
a
e1 e2
3a
B
182 CAPTULO 17. CAMPO ELCTRICO EN MEDIOS MATERIALES

II. Soluciones

Solucin 17.1 P X

a) Dada la simetra esfrica del problema, es posible usar ley de Gauss de la siguiente forma
rR

D ~ = Qlibre = 0 = D
~ dS ~ =0

~ = 0 (es un conductor)
Por lo tanto E
R1 r R2

~ = Qlibre = D(r) 4r2 = Q = D(r)
~ dS ~ Q
D = r
4r2
Por lo tanto
~ = 1D
E ~ = Q r
 4r2
r R2

~ = Qlibre = D(r) 4r2 = Q = D(r)
~ dS ~ Q
D = r
4r2
Por lo tanto
~ = 1D
E ~ = Q r
0 40 r2
b) Las densidades de polarizacin estn ubicadas en
( 0 ) Q ( 0 ) Q
pol (R1 ) = P~ (R1 ) (r) = 2 r (r) =
 4R1  4R12
( 0 ) Q ( 0 ) Q
pol (R2 ) = P~ (R2 ) r = 2 r r =
 4R2  4R22
Mientras que la densidad de carga libre es
~ 1 ) r = Q r r = Q
libre (R1 ) = D(R
4R12 4R12
c) La diferencia de potencial entre el infinito y la esfera es
R2  
~ =
~ dl Q Q Q 1 1 Q
V1 (R1 ) = E dr + = +
4r2 40 r 2 4 R1 R2 40 R2
R1 R1 R2

En ausencia de dielctrico se tendra que la tensin V2 es


 
Q Q 1 1 Q
V2 (R1 ) = = +
40 R1 40 R1 R2 40 R2
Ntese que el ltimo trmino para ambos potenciales es igual, por lo que el trmino que
indica cual es mayor es el primer trmino de la suma, ahora dado que  > 0 , se tiene que
   
1 1 Q 1 1 Q 1 1
> = > = V2 (R1 ) > V1 (R1 )
0  40 R1 R2 4 R1 R2
El potencial en la esfera es mayor en ausencia del dielctrico.
II. SOLUCIONES 183

Solucin 17.2 P X

a) Para r < R se tiene E ~ = 0 ya que es un conductor. Para r > R es necesario usar ley de Gauss
de la siguiente forma

~ = Qlibre = D1 (r) 2r2 + D2 (r) 2r2 = Q
~ dS
D

Adicionalmente, dado que por simetra el campo elctrico es completamente tangencial a la


~1 = E
interfase de los medios, se tiene que E ~2 = E
~ . Luego D1 (r) = 1 E(r) y D2 (r) = 2 E(r),
por lo que se puede despejar la expresin del campo elctrico para r > R como

~ Q
E(r) = r
2r2 (1 + 2 )

b) La densidad de carga libre depender del hemisferio de la esfera, de modo que para el hemisferio
inferior
~ 1 (r) n 1 Q
l1 = D = 2
r r
r=R 2R (1 + 2 )
y de la misma forma para el hemisferio superior
2 Q
~ 2 (r) n
l2 = D = r r
r=R 2
2R (1 + 2 )

En cuanto a las cargas de polarizacin, hay que calcular previamente el vector polarizacin en
cada medio
~ = 0 E
D ~ + P~ = E~ = P~ = ( 0 )E~

De modo que
(1 0 )Q
P~1 = (1 0 )E
~ = r
2r2 (1 + 2 )
Entonces,
(1 0 )Q (1 0 )Q

P 1 = P~1 (r) n = 2
r (r) =
r=R 2R (1 + 2 ) 2R2 (1 + 2 )

De forma anloga
(2 0 )Q
P 2 =
2R2 (1 + 2 )
Notar el hecho que la normal en el caso de la densidad de carga libre apunta desde el conductor
al dielctrico y en el caso de la densidad de carga de polarizacin apunta en el sentido inverso.

c) La carga de polarizacin total est dada por



(1 + 2 20 )Q QP 1 + 2 20 39
2
QP = 2R P 1 + 2R P 2 2
= = =
(1 + 2 ) Q 1 + 2 40

Luego la carga neta vale


Q
Qneta = Q + QP = >0
40
.
184 CAPTULO 17. CAMPO ELCTRICO EN MEDIOS MATERIALES

Solucin 17.3 P X

Para iniciar el problema se necesita saber la capacitancia de un condensador cilndrico que posee un
medio con constante i en un interior, de radios a y b y de largo H. Para ello se usa la ley de Gauss
para el espacio entre los cilindros (a < r < b).

"
~ ~ = Q = D(r)2rH = Q = E
~ = Q
D(r) dS r
2i rH
Luego
b  
Q Q b
V = V (a) V (b) = = ln
2i rH 2i H a
a

Por lo que
Q 2i H
C= = 
V ln ab

z
e1 z

e2 L z

Figura 17.1: Dielctricos dentro del Condensador Cilndrico

Para lo que sigue del anlisis se debe considerar la Figura 17.1. Se ha puesto el dielctrico 1 una distan-
cia z dentro del condensador. La capacitancia de este sistema es equivalente a la de dos condensadores
en paralelo, luego la capacitancia total del sistema es igual a su suma, es decir:

21 z 22 (L z) 2
CT = C1 + C2 = b
+ b
 =  ((1 2 )z + 2 L)
ln a ln a ln ab

Por lo tanto la energa de ese condensador est dada por

1 Q2
U=
2 CT (z)
II. SOLUCIONES 185

Luego, considerando que la carga es constante, la fuerza que ejerce el sistema es


 
~ 1 2 1 1 Q2 CT (z)
Fsist = U = Q z = z
2 z CT (z) 2 CT (z)2 z
Reduciendo la expresin a
 
1 2 b 1 2
F~sist = Q ln z
4 a ((1 2 )z + 2 L)2
Ntese lo siguiente, como 1 < 2 la fuerza que efecta el sistema apunta en el sentido de negativo
de z, por lo que el dielctrico 2 intenta expulsar al dielctrico 1 del condensador. En efecto, este
resultado es consecuencia de que el sistema busque su mnima energa, la se obtiene con el condensador
completamente lleno del dielctrico con constante 2 . Por lo tanto, para poder introducir el dielctrico
1 una distancia z = 12 L dentro del condensador se debe aplicar una fuerza del mismo valor en el
sentido contrario a la que se siente en ese punto, es decir, la fuerza buscada vale
 
~ ~ Q2 b 2 1
Fext = Fsist 1 = 2
ln z
z= 2 L L a (1 + 2 )2

Solucin 17.4 P X

a) Se debe encontrar el valor de V (x), sabiendo que el potencial cumple la siguiente condicin

d2 V (x) dV (x)
2
+k =0
dx dx
Con k > 0 y 0 x d. Para ello se asume que la solucin de la EDO es V (x) = Aex + B
con A, B y constantes no nulas por determinar. En efecto, reemplazando la solucin en la
ecuacin anterior
d2 V (x) dV (x)
2
+k = A2 ex + kAex = Aex ( + k) = 0
dx dx
De la ecuacin anterior se obtiene
= k
Luego, la solucin se transforma en V (x) = Aekx + B y las otras constantes pueden ser
despejadas con las condiciones de borde

V (0) = 0 = A + B = 0 = A = B

V (d) = V0 = Aekd + B = V0
Despejando A y B de las ecuaciones anteriores

V0
A=
ekd 1
V0
B = kd
e 1
Finalmente la solucin es
V0 (ekx 1)
V (x) =
ekd 1
186 CAPTULO 17. CAMPO ELCTRICO EN MEDIOS MATERIALES

b) Dado que se conoce el potencial, es posible conocer el campo elctrico dentro de las placas
 kx 1)

~ = V = d V 0 (e V0 kekx
E x = x
dx ekd 1 ekd 1

Asumiendo que el material es lineal, istropo y no homogneo se tiene que


kx
D ~ = (x) V0 ke
~ = (x)E x
ekd 1
Dada la ausencia de carga libre dentro de las placas

~ = 0 = d ((x)ekx ) = 0 = (x)ekx = C
~ D

dx
Usando la condicin de borde (x = 0) = 1 = C = 1 por lo tanto

(x) = 1 ekx

~
c) La densidad de carga libre en la placa a potencial V0 dada por l = D n , de modo que
x=d

V0 kekx kV0
l = ekx x (x) = kd
ekd 1 e 1

Solucin 17.6 P X

a) La capacitancia de un condensador de placas paralelas es

A
C=
d
donde  es la permitividad del medio que lleva dentro el condensador, A es el rea de las placas
y d es la separacin entre ellas. Si un dielctrico se pone parcialmente una distancia x en el
condensador, se forman dos condensadores en paralelo, de capacitancia

ax 0 a(a x)
C1 = , C2 =
b b
Por lo que la capacitancia total (condensadores en paralelo) es
a
CT = C1 + C2 = (x + (a x)0 )
d

b) Por otro lado, se tiene que la definicin capacitancia es

Q
C=
V
Por lo que si el condensador est conectado a una batera de voltaje V0 y tiene una carga Q
se cumple que
 
a Q 1 Q b
CT = (x + (a x)0 ) = = x = a
b V0  0 V0 a
II. SOLUCIONES 187

d) Dada la dependencia Q = Q(x) se tiene que la carga vara en el rango x [0, a], si se usa la
parte anterior, se obtiene que
a
Q(x) = (( 0 )x 0 a)
d
lo cual es una funcin lineal, y como  0 se tiene la funcin Q(x) alcanza su mnimo en
x = 0 y su mximo en x = a. Por lo que
0 a2 a2
Q(x = 0) = Q(x) = Q(x = a)
d d
Ntese que fsicamente esto significa que el condensador alcanza su mnima carga vaco y su
mxima carga lleno de dielctrico.

Solucin 17.7 P X

a) Calcular las densidades de carga de polarizacin en el espacio. Para resolver este problema es
necesario dividir en dos reas, un radio menor y mayor al radio de esfera b.

r > b se tiene que hay vaco por lo tanto no existe el vector polarizacin.
r b, de acuerdo a la ley de Gauss

~ ~
D dS = 0 rdV

r4
D(r) 4r2 = 40
4
2
~ = 0 r r
D
4
Por lo tanto
2
~ = 0 r r
E
4
Utilizando P~ = D
~ 0 E
~ se obtiene
( 0 )0 r2
P~ = r
4
Luego, la densidad de carga de superficial polarizacin es
p = P~ n| r=b
( 0 )0 b2
=
4
y la densidad volumtrica polarizacin es
p = P~
1
= (Pr r2 )
r2 r  
1 ( 0 )0 r4
= 2
r r 4
1 ( 0 )0 r3
= 2
r 
( 0 )0 r
=

188 CAPTULO 17. CAMPO ELCTRICO EN MEDIOS MATERIALES

~ y
b) Calcular la energa electrosttica del sistema de cargas. Para esto es necesario encontrar D
~
E para un r > b.

D ~ =
~ dS 0 rdV

b4
D(r) 4r2 = 40
4
4
~ = o b r
D
4r 2

Entonces,
4
~ = o b r
E
40 r2
Ya con esto es posible encontrar la energa del sistema en todo el espacio

1 ~ EdV
~
U= D
2
Todo el Espacio
2
1
= D(r) E(r)r2 sin()ddd
2
0 0 0
b 2 2
1 1
= D(r) E(r)r2 sin()ddd + D(r) E(r)r2 sin()ddd
2 2
0 0 0 b 0 0
b 2 2
1 0 r2 0 r2 2 1 0 b4 0 b4 2
= r sin()ddd + r sin()ddd
2 4 4 2 4r2 40 r2
0 0 0 b 0 0
b  2  
0 r2 0 b4
= 2  r2 dr + 2 0 r2 dr
4 40 r2
0 b

b
20 0 2 b8 r2
= 2 r 6
dr + dr
162 1620 r4
0 b
 
20 b7 1 1
= +
8 7 0

Solucin 17.11 P X

a) Como sistema de referencia no daremos z creciente hacia abajo. Para el condensador de la


izquierda:
J1n = J2n = J~1 = J~2 = J~

(vector densidad de corriente constante en ambos medios). Suponiendo estado estacionario


dentro del condensador
~ J~ = 0 = dJ = 0 = J~ = Az
dz
II. SOLUCIONES 189

donde A es una constante por determinar. Por lo tanto el campo elctrico en cada medio est
dado por
E~ 1 = A z , E
~ 2 = A z
g1 g2
La constante A puede ser determinada como

d 2d
A A Ad Ad V0 g1 g2
V (z = 2d) V (z = 0) = V0 = dz dz = = A =
g1 g2 g1 g2 d(g1 + g2 )
0 d

Por lo tanto
V0 g1 g2
J~ = z
d(g1 + g2 )

~1 = V 0 g2 ~2 = V 0 g1
E z , E z
d(g1 + g2 ) d(g1 + g2 )

~1 = 1 V0 g2 ~ 2 = 2 V0 g1 z
D z , D
d(g1 + g2 ) d(g1 + g2 )
Para el condensador de la derecha:
~1 = E
E1t = E2t = E ~2 = E
~

(campo elctrico constante en ambos medios). Luego, dado que no hay densidades volumtricas
de carga del condensador

~ = 0 = dE = 0 = E
~ E ~ = C z
dz
donde C es una constante por determinar. La constante C puede ser determinada como

2d
V0
V (z = 2d) V (z = 0) = V0 = Cdz = C =
2d
0

Por lo tanto
~ = V0 z
E
2d
g1 V 0 g2 V0
J~1 = z , J~2 = z
2d 2d
~ 1 = 1 V0 z , D
D ~ 2 = 2 V0 z
2d 2d
b) Para el condensador de la derecha, se tienen las siguientes densidades polarizacin

V0 V0
P 1 (z = 0) = (1 0 ) z z = (1 0 )
2d 2d
V0 V0
P 1 (z = 2d) = (1 0 ) z z = (1 0 )
2d 2d
V0 V0
P 2 (z = 0) = (2 0 ) z z = (2 0 )
2d 2d
V0 V0
P 2 (z = 2d) = 2 z z = (2 0 )
2d 2d
190 CAPTULO 17. CAMPO ELCTRICO EN MEDIOS MATERIALES

Mientras que las de carga libre son


V0 V0
L1 (z = 0) = 1 z z = 1
2d 2d
V0 V0
L1 (z = 2d) = 1z z = 1
2d 2d
V0 V0
L2 (z = 0) = 2 z z = 2
2d 2d
V0 V0
L2 (z = 2d) = 2 z z = 2
2d 2d
No hay densidades volumtricas de ningn tipo dentro del condensador.

c) En este caso hay que notar que el capacitor de la izquierda puede ser dividido en dos capactores
en serie, mientras que el capacitor de la derecha puede ser dividido en dos en paralelo. Usando
el resultado conocido de la capacitancia de un condensador de placas paralelas C = A L , se tiene
que:

1 1 2a2 2 2a2 4a2 1 2 1 a2 2 a2


CT = d d
+ + = + +
1 4a2
+ 2 4a2 | 2d {z 2d } d(1 + 2 ) d d
| {z } Capacitancia de la derecha
Capacitancia de la izquierda

d) La corriente total que sale de la fuente puede ser determinada integrando los vector densidad
de corriente que pasa por el condensador de la izquierda (J)~ y los dos vectores densidad de
corriente que pasan por el condensador de la derecha (J1 y J~2 ). Por lo tanto:
~

2a 2a 2a a 2a 2a
V0 g1 g2 g1 V0 g2 V0 V0 g1 g2 g1 V0 g2 V 0
I= dxdy+ dxdy+ dxdy = 4a2 +2a2 +2a2
d(g1 + g2 ) 2d 2d d(g1 + g2 ) 2d 2d
0 0 0 0 0 a
 
V0 a2 g1 g2 4 1 1
I= + +
d g1 + g2 g2 g1
Por lo tanto la resistencia equivalente es
V0 d
R= =  
I 4
a2 g1 g2 g1 +g2 + 1
+ 1
g2 g1
Campo Magntico en Medios Materiales
18
I. Problemas Propuestos

Problema 18.1 X
3a
4a
Se tiene un dispositivo de simetra cilndrica que consta 5a
de un conductor cilndrico macizo de radio 3a rodea-
do por un conductor cilndrico hueco de radio interior
4a y radio exterior 5a por el cilindro central pasa una J0 z
densidad de corriente uniforme J~ = J0 z y por el cilin-
dro hueco exterior circula una intensidad de corriente de
igual magnitud pero de signo opuesto. El medio entre
ambos conductores tiene permeabilidad > 0 . Calcule
el campo magntico en todas partes.

Problema 18.2 X
Considere una bobina toroidal de seccin rectangular de
N espiras, por cada una de las cuales circula una co-
rriente I. El radio interior de la bobina es a y el exterior N
es b y la altura es h. El ncleo de esta bobina es de un q
material inhomogneo en tal forma que su permeabili-
(q ) h
dad magntica depende tan solo del ngulo polar y
satisface a
0 b
= 1 + k cos2

~ en el interior de
Determine la intensidad magntica H
la bobina.
Problema 18.3 X S
6b
Un material ferromagntico tiene la forma que se indica
en la figura, donde la seccin transversal es un cuadrado I
de lado de a = 2 cm. El resto de las dimensiones se
muestran en la figura con b = 2, 5 cm. El enrollado tiene N 4b
N = 500 vueltas, la corriente que circula es I = 0,3 A
y = 25000 . Calcule el flujo neto en la rama central y
en la rama del lado derecho. Desprecie cualquier prdida
3b
de flujo fuera del material ferromagntico.

191
192 CAPTULO 18. CAMPO MAGNTICO EN MEDIOS MATERIALES

Problema 18.4 X S
I
Un solenoide infinito de radio a que tiene m espiras
por unidad de largo, lleva corriente I. La permeabilidad
magntica del material al interior del solenoide es

1 si 0 <
=
2 si < 2 a
1
Encuentre el campo magntico, la intensidad magntica 2
en todo el espacio, la magnetizacin de los materiales y 1 2
las densidades de corriente.
Vista Frontal Corte Transversal

Problema 18.5 X
Considere un toroide de seccin transversal circular A
y de radio medio R como se muestra en la Figura. El
toroide est compuesto por tres medios de permeabili-
dades , 1 y 2 (ver Figura). Un cable con corriente I
atraviesa el toroide por su centro por eje perpendicular
al toroide. Para efectos de calculo, puede considerar que 1
.
q1 2
a) Calcule H ~ y B,~ para cada material si las permea- q2
R
bilidades de los materiales son lineales, uniformes
e isotrpicas.
!
b) Si las permeabilidades de los materiales son linea- I
les uniformes e isotrpicas existiran densidades
de corriente de magnetizacin?. Si existieran, cal-
clelas.

c) Cmo cambia (explique) H,~ B ~ y las corrientes


de magnetizacin si 1 = r o 1 = ?.

Problema 18.6 X
R
El sistema de la Figura representa un conductor ciln- D
drico de resistividad = s el cual est sometido a
una diferencia de potencial V0 . Este conductor posee
adems caractersticas magnticas, con una permeabi-
r
lidad relativa (r) = 0 1 R , tal como se muestra
en la Figura. Suponiendo que la corriente se distribu-
+

ye en forma homognea al interior del conductor y que


V0 , rs
D  L  R, se pide
L
~ y B~ en el interior y exterior
a) Estimar campos H z
(pero en las cercanas) del conductor.
~.
b) Estimar la magnetizacin M

c) Determinar las densidades de corrientes de mag-


netizacin y libre en el conductor.
I. PROBLEMAS PROPUESTOS 193

Problema 18.7 X S
Considere N vueltas de un alambre conductor se han en-
q
rollado sobre un toroide de hierro dulce de seccin trans-
versal S y permeabilidad . El radio medio del toroide es r
R y posee un entrehierro de longitud ` como se muestra
R
en la Figura. La corriente I que circula por el alambre es
`
suficientemente grande de manera que la magnetizacin
M~ adopta su valor de saturacin M ~ s = Ms en el hierro
N
dulce.
a) Encuentre los valores de H~ yB ~ en todo el toroide.

b) Cuando la corriente I se interrumpe, la magnitud


S
de la magnetizacin decrece a su valor remanen- I
te M~ r = Mr en el hierro dulce. Encuentre los
~ yB
valores de H ~ en esta situacin.

Problema 18.8 X
g
Considere un toroide de permeabilidad de seccin cua-
a
drada a2 y de largo 8a. El toroide cuenta con dos en-
I0
trehierros de una longitud muy pequea g tal como y se
muestra en la Figura. En uno de sus extremos el toroide
posee un enrollado de N vueltas, el cual est conectado N 6a
una fuente de corriente I0 . Determine
a) El flujo magntico y la autoinductancia del siste-
ma. a

b) Si el lado izquierdo con forma es U es fijo, la a 6a a


fuerza que siente el material de la derecha.

Problema 18.9 X
Considere una lnea de transmisin coaxial llena de un
material con permeabilidad magntica no lineal, con un
conductor interno slido de radio a y un conductor ex-
Material Magntico
terno muy delgado, de radio interior b segn se muestra
en la figura. Se sabe que en el conductor interno circu-
la una corriente I0 hacia afuera de la hoja, y vuelve en
direccin opuesta por el conductor externo. En ambos
conductores la corriente se reparte en forma homognea, a
y ambos se pueden suponer muy largos. Si la curva de
magnetizacin del material se puede aproximar como
b
1,6H
B=
1000 + H

a) Campo magntica en todo el espacio.

b) Vector de magnetizacin en el medio material.


194 CAPTULO 18. CAMPO MAGNTICO EN MEDIOS MATERIALES

Problema 18.10 X
Un circuito magntico es un toroide de material magn- R2
ticamente lineal de permeabilidad , de seccin circular
de radio R1 , siendo el radio de la circunferencia media
del toro R2 (R2  R1 ). El toroide tiene un entrehierro N
de longitud e, que est ocupado por un disco conductor
de radio a (a > R1 ) y espesor e colocado simtricamen-
te en el entrehierro. La permeabilidad del disco es 0 y I(t)
su conductividad g. El circuito magntico est alimenta-
do por un arrollamiento de N espiras por las que circula
e
una corriente I(t) = kt donde k es una constante.
a) Calcular B~ yH ~ en las dos regiones del toroide.
R1
b) Encontrar la potencia disipada en el disco conduc- a
g
tor.

Disco Conductor

X a
Problema 18.11

a) Un cubo de lado a est hecho de un material fe- ~ a


M
rromagntico el cual est permanentemente mag-
netizado, con una magnetizacin igual
a
x y
~ = M0 x z
M
a
(a)
Encuentre las densidades de corriente de magne-
tizacin presentes en el cubo. z
R
b) Se tiene un cilindro infinitamente largo, de radio
R con su eje coincidente con el eje z y con una
magnetizacin constante M ~ = M0 x perpendicu-
lar a su eje. Determine las corrientes de magne-
tizacin (volumtricas y superficiales) y el campo
magntico en el eje del cilindro.

(b)
II. SOLUCIONES 195

II. Soluciones

Solucin 18.3 P X

Para resolver este problema se usar la tcnica de reluctancias. En primera instancia, es necesario
definir reluctancia como
l
R=
A
donde l es el largo, la permeabilidad y A la seccin transversal del medio por donde pasa un
respectivo flujo. La idea de este mtodo es emular un circuito simple con resistencias, en el cual la
fuente de tensin es el equivalente al enrollado con corriente (V N I), las corrientes es equivalente
al los flujos magnticos (I ) y las resistencias a las reluctancias (R R).

Para aplicar este mtodo, es necesario identificar las reluctancias en el circuito magntico, para ello
se verifica en que ramales del circuito pasa el mismo flujo.

6b

I 1 2

N 4b

3b
Figura 18.1: Determinacin de Reluctancias.

En la Figura 18.1, es posible identificar los caminos por los cuales circula el mismo flujo. Cada uno
de esos caminos 1 , 2 y 3 tienen asociadas los siguientes valores de reluctancias:

10b 4b 10b
R1 = R2 = R3 =
A A A

Una vez determinado los valores de las reluctancias, se procede a plantear el circuito anlogo.

1
R1

2 3

NI R2 R3

Figura 18.2: Circuito Equivalente.


196 CAPTULO 18. CAMPO MAGNTICO EN MEDIOS MATERIALES

Las incgnitas en este circuito son los flujos 1 , 2 y 3 como es mostrado en la Figura 18.2.
Planteando la ley de Kirchoff de Nodos se obtiene que

1 = 2 + 3

Mientras si se plantea la ley de Kirchoff de Voltajes

N I = R1 1 + R2 2

R2 = R3 3
Usando las tres ecuaciones anteriores es posible despejar los valores pedidos, de modo que
N I0 A
2 =
18b
N I0 A
3 =
45b

Solucin 18.4 P X

Dada la geometra del problema, el campo magntico provocado por el solenoide debe ser paralelo
al eje del cilindro. Recurdese en el solenoide ideal e infinitamente largo se hace el supuesto que el
campo magntico es nulo fuera del mismo.

a
z

I
y

L ~B1 ~B2

~1
H ~2
H

1 2

Figura 18.3: Clculo de Campo Magntico Usando Ley de mpere.

En la Figura 18.3 se muestra el rectngulo con el cual puede ser usada la Ley de Ampre de la siguente
forma
H ~ = Ilibre = H1 L = mLI = H
~ 1 dl ~ 1 = mI z
II. SOLUCIONES 197

~ 2 , por lo que tambin se puede concluir


Anlogamente, es posible hacer el mismo anlisis para H
~ 2 = mI z
H
~2 H
El resultado anterior es coherente con la condicin de borde n (H ~ 1) = K
~ libre ya que en la
~ ~
interfase de los medios no hay corrientes libres circulando por ende H1 = H2 . Por otro lado, los
campos magnticos estn dados por
~ 1 = 1 mI z
B ~ 2 = 2 mI z
B

A partir de lo anterior, se procede a determinar las magnetizaciones de cada medio, las cuales cumplen
~ = 0 ( H
B ~ +M ~ ), por lo tanto
   
~ 1 ~ 2
M1 = 1 mI z M2 = 1 mI z
0 0

Ahora, para encontrar las densidades de corrientes presentes se usan las definiciones K~M = M
~ n y
~ ~
JM = M . Hay que tener precaucin debido a que los medios tienen ms de una normal.

r
1 ~1
K y
y
y x
~2 2
K

r
Figura 18.4: Clculo de Corrientes de Magnetizacin.

Siguiendo la Figura 18.4, se pueden determinar las corrientes superficiales en la divisin son
    
~ 1 1
K1 = 1 mI z (y) = 1 mI x
0 0
    
~2 = 2 2
K 1 mI z y = 1 mI x
0 0
Mientras que en los mantos se tiene que
    
~1 = 1 1
K 1 mI z r = 1 mI
0 0
    
~ 2 2
K2 = 1 mI z r = 1 mI
0 0
~ es constante J~ =
Finalmente, dado que M ~ M
~ = 0.
198 CAPTULO 18. CAMPO MAGNTICO EN MEDIOS MATERIALES

Solucin 18.7 P X

a) En primera instancia, debido al solenoide enrolado en el toroide, existe un campo magntico


confinado dentro del material magntico y tambin en el entrehierro. A priori, se supone que
existen los campos H ~1 y B~ 1 en el hierro dulce y los campos H
~0 y B
~ 0 en el entrehierro. Adi-
cionalmente, en este tipo de problema se suele suponer que el campo magntico es ms bien
homogneo dentro del toroide, por lo cual es una buena aproximacin es tomar el camino medio
del mismo para calcular H ~ yB ~ en todos los puntos.
Entonces, usando Ley de Ampre al camino medio de radio R y usando el supuesto que H ~1 =
~
H1 y H0 = H0 se obtiene que

H ~ = N I = H1 (2R `) + H0 ` = N I
~ dl

Dado que el hierro dulce alcanz su mxima magnetizacin, se puede afirmar que
~ 1 = 0 (H
B ~1 + M
~ s)

El paso clave del problema, es usar la siguiente condicin de borde


~0 B
(B ~ 1 ) n = 0 = B0n B1n = 0 = B0n = B1n

Ntese que es posible aplicar esta condicin ya que el campo magntico es perpendicular a las
~ debe ser continuo
interfases del problema (hierro-entrehierro y entrehierro-hierro), por ende B
~ ~
en esos puntos. Adicionalmente, como H0 = 0 B0 se obtiene que

~1 = N I Ms `
H1 (2R `) + H1 d + Ms d = N I = H
2R
En consecuencia,
 
~ 0 = N I Ms ` + M
H ~s ~0 = 1
~1 = B
B
N I Ms ` ~s
+ M
2R 0 2R

~ s M
b) Usando el resultado obtenido en la parte anterior, se tiene ahora que I = 0 y M ~ r . Por
lo que los valores de los campos son

~ 1 = Mr `
H
2R

~0 = M
H ~ r Mr `
2R
 
~ ~ 1 ~ Mr `
B1 = B0 = Mr
0 2R
Parte VI

Respuestas

199
Respuestas
19
Captulo 1

Problema Respuestas

~ =
a) E q
(1 r)
r
40 r2

~ = 0 para ambos
~ dl
b) Si es una circunferencia de radio R sobre el plano xy entonces E
P 1.1 campos elctricos (experimental y coulombiano).
!
c) Para una esfera de R centrada en la carga, E ~ dS~ = q (1 R). Para el caso de un
! 0
campo tradicional por Coulomb: E ~ dS
~= q.
0

P 1.2 Q= e
3

~ )=0
a) E(P
3
P ~ = q 2 1
+ y + 12 z

1.3 b) E 40 a2 3
2
2
x
3
~ total =
c) E q 2 2
y
0 a2 3


P 1.4 ~
E(0) = q
40 a2
(2 3x 2y), V (0) = 0

P 1.5 r= a

2

P 1.6 U= e2 ln(2)
20 a

201
202 CAPTULO 19. RESPUESTAS

Captulo 2

Problema Respuestas

P
 
2.1 ~ total =
E 7 3 z
20 5

p
P qK + (qK )2 + 4
2.2 cos eq = con K =
, (q ) =
20 lmg 2
2

s  
P 1 qQ
2.3 = k+
m 40 R3

 
q ~ q
a) V (C) = + b) E(C) = y
P 2.4 40 R 40 40 R2 20 R
c) q = 2R


L L
~ = a z 2
z+ 2
a) E 3   3 z

20 

L 2 L 2
P 2 2
 
2.5 z
+ a2 2
z+ 2
+ a2
 
a 1 1
b) V (O) V (O0 ) =
0 a L2 + a2

P ~ ~
2.6 a) E(A) = (y x) b) E(B) = x
40 a 40 a

P ~ = x  
2.7 E cos 2 sin
20 R 8 8

 
1 1


40 xL
x
x x>L
~
 
a) E(x) =
4 1
x1 x x < 0

0 xL

P 0xL

2.8  
2 2
b) F~ = ln (L+d)
x
40 d(2L+d)
 2
  
(L+d) 2 2
~ 1 Q2
c) ln d(2L+d) ln 1 + Ld2 Ld2 entonces F 40 d2
x

P ~ = z ~ = ( R2 + (d + a)2 R2 + d2 )z
p p
2.9 a) E z b) F
20 R2 + z 2 20


P 2.10 ~
E(0) = z
30

h
P 2.11 V = ln (1 + 2)
20

P ~ a2 (a z) 3Q0 a
2.12 a) E(z) = b) W =
20 (a + z) 40
203

Problema Respuestas

P 2.13 ~ = h (1 cos )z
E
20

P 6GM cos2
2.14 ~g = z
h3 (1 + cos )

h i
~ 1 )| = 0
p
a) |E(P 20
L ( h2 + R2 (L + h)2 + R2 )
P 2.15
~ 2 )| = 0 R 2 L
b) |E(P
20 h 4h2 +L2

a) = 0aE0
P 2.16
b) 1 = 2 = 0 E0
204 CAPTULO 19. RESPUESTAS

Captulo 3

Problema Respuestas

80 R3
a) Q = 15
0
(
15R2 0
(5R2 r 3r3 )r rR
~
b) E(r) = 20 R3
P 3.1 150 r 2
r r>R
20 R2 0 (r 2 R2 ) (7R2 3r 2 )
(
150
150 R2 4
rR
c) V (r) = 20 R3
150 r
r>R



0 0 < r < R1
2 (r 2 R12
)

r R 1 r R2


20 r


2 2
P

2 (R2 R1 )
3.2 ~
E(r) = 20 r
r R 2 r R3
2 2
2 (R2 R1 )+4 (r 2 R32
)

r R3 r R4



20 r
2 2 2 2
(R R
2 2 1 4 4 3 ) r
)+ (R R
20 r
r R4

~ = z +
a) E D
z
20 20

P 3.4 ~ = + z +
b) E 20
(D2d)
20
z
~ = + z
c) E D
z
20 20

nr
30 r
0<r<a
P

~ a3
3.3 E= 3
r (n + 1) r 2 r arb
0
3 3

30 r 2 b (n + 1)a r rb

P q qd
3.5 y(x) = x2 (trayectoria), tan = (ngulo).
20 mvx2 m0 vx2

P 3.6 E = Q
240

P 3.7 ~ =
E
~
0 d
30

~ = x0 x
a) E
P 3.8 0
b) () = 2x0 cos

4E0 r
a) (r) = a2
E0 a2 E0 a2
P 3.9
~ > a) =
b) E(r 0 r 2
r, V (r > a) = 0 r
 3

E0
c) V (r a) = 30
4a ar 2

(
2hr 3
3a
rR
a) Q(r) = 2hR3
3a
r>R
r2
(
r rR
P 3.10 ~
b) E(r) = 3a0
R3
3a0 r
r r>R
3
(
r
9a rR
c) V (r) = R3
0  r
 1

3a0
ln R
+ 3
r>R
205

Problema Respuestas


(
r r<R
~
a) E(r) = 20 r  El campo elctrico es discontinuo en r = R.
R
20 r
+ 0 r
r r>R
P 3.11 (

ln R

20 r
rR
b) V (r) = 1
 R
 El potencial elctrico es siempre continuo.
0 2
+ R ln r
r>R

P 3.12 ~
b) E(z) =
k0
(1 ekz )z

P
 
~ T (h) = sin (2) cos (2) qh q ln(2) sin 2

3.13 a) E 2h0
x + 20
2h0
y b) W = 40
+ 20 tan
cos 2
206 CAPTULO 19. RESPUESTAS

Captulo 4

Problema Respuestas

P 4.1 ~ = U z = 1 Q2 z b)
a) F z 2 0 A
W = Q
4C
2
c) V = Q
3C


a) = 2R


~ 20 r
rr rR
P b) E(r) =
4.2 0r r>R
r
 
2 ln R rR
c) V (r) = 0
0 r>R

P 4.3 b= 3a
entonces U =
2 3
270 E0 a
4 128

P 4.4 q= QR2
R1 +R2
, Umin = Q2
80 (R1 +R2 )
, V1 (R1 ) = V2 (R2 ) = Q
40 (R1 +R2 )

a) qint = q, qext = q, la densidad de carga interior es mayor en los puntos ms cercanos a


la carga. La densidad exterior es homognea.
b) No cambia.
P 4.5 c) Toda la carga negativa q interior se acumula en el punto de contacto, luego al contacto,
ambas se descargan y slo queda la carga q positiva en la superficie exterior.
d) La carga q 0 provoca que la densidad de carga en la esfera exterior cambie. La distribucin
de carga al interior se mantiene igual.

P 4.6 VB VA = 2V0 ln b
a


P 5 5
n h   io
e2 R2 R1
4.7 U= 80
1
5R1
+ 1
3 R3 )1
(R2
R26 R11 1
R2
R23 (R22 R12 ) + 5
1



0 0<r<a V0 0<r<a
V (11+1)


1 1V0 1 2 r ar<b 0 1 c 1 b+ 1 r ar<b



~ ( c b + a )r (c b a)
a) E(r) = V (r) = V0
0 br<c 11+1 c br<c

V0

( c b a)
rc
rr

V
( 1c 1b + a1 )r2

1 1 1 0
rc
( c b + a )r
a = a2 1
0 V0
, = b2 10 V0
y c = c2 1 0 V0
P 4.8 ( c 1b + a1 ) b ( c 1b + a1 ) ( c 1b + a1 )
V0 1
1b

0 0<r<a 11+1 0<r<a

V0

( c b a ) a

1 1 + 1 r 2 r ar<b
V0 1 1

b ar<b

~
b) E(r) = (c b a) V (r) = ( 1c 1b + a1 ) r


0 b r < c

0 br<c
0 rc

0 rc
0 V0 0 V0
a = a2 ( 1 1 1 , b = b2 y c = 0
c b
+a ) ( 1c 1b + a1 )

P 4.9 El potencial al interior de la burbuja es V = Q


40 R
, independiente si es o no esfrica.

a) Qfinal
1 = a
a+b
(Q1 + Q2 ), Qfinal
2 = b
a+b
(Q1 + Q2 ), como b > a entonces Qfinal
2 > Qfinal
1
P 4.10 1
b) C = 1 1 +11
db da b a

P 4.11 C= 20 L
ln(R1 R3 /R2 R4 )
207

Problema Respuestas

Q Q
a) a = 0, b = 2bL , c = 2cL , d = 0
Q c

20 L ln b  r < b
P 4.12 b) V (r) = Q
ln rc br<c
20 L
0 r>c
Q c

c) V (c) V (b) = 20 L ln b

Q
a) = l
Q Q
b) 1 = 2R1 L
; 2 = 2R2 L

~ < R0 ) = E(R
c) E(r ~ 1 < r < R2 ) = 0; E(R
~ 0 < r < R1 ) = Q
~ > R2 )
= E(r
2L0
Q
d) 10 = 2R1 L
; 20 =0
P 4.13 ~ > R2 ) = 0, en el resto de las zonas el campo permanece igual.
e) E(r
Q
f) V = 20 L
ln( R
R0
1
)
20 L
g) C = R
ln( R1 )
0

Q2
h) U = 40 L
ln( R
R0
1
)
i) C 0 = C

Q Q
80 a
+ 40 a3
(a2 r2 ) 0<r<a
P

Q
4.14 (r) = 80 a
a r < 2a
Q
r 2a


40 r

  
0 2a
1 exp z z0 > a


0



~ 0) =
a) E(z   
0 2a


1 exp z z0 < a


0

0
 z z0 > a


0
P 4.15
~ 0) =
b) E(z
0 z z0 < a



0
V0 a z a






c) V (z) = V0 00 (z a) z > a




V0 + 00 (z + a) z < a



0 r < R2
0 (r2 R22 ) r


~ 2r0
R2 r R3
a) E(r) = 2
0 ( R3 2
R2 )

2r0
r R3 r R4
P


4.16 0 r > R4

2 2
0 (R3 R2 )
(r = R1 ) = 0, (r = R4 ) = 2R4
, (r = R5 ) = 0
 2 2    
0 R3 R2
b) V (R4 ) V (R1 ) = 20 2
+ R22 ln R 3
R2
R4
+ (R32 R22 ) ln R 3
208 CAPTULO 19. RESPUESTAS

Captulo 5

Problema Respuestas

ln(| sin kx|)


b) z(x) = + C con C R.
P 5.1 k
c) = 20 V0 k cos kx

0 a2
P a) V = 0
5.2
b) V = 12 [V].

a
b < x < a

0
(x+ b)
P

20 2

5.3 V (x) = x + a(2b a) a < x < a
a
0 (x b) a<x

( 2
30 V0 (ka+1) a2 )
V0 V0 (ka+1)(r

P a2
ra 2a2
ra
5.4 (r) = (r) =
k2 0 Vr0 a ek(ar) r>a V0 a k(ar)
r
e r>a

P 5.5 C= 0 a
d
ln d+a
d


P 5.6 CT = 40 R1 R2
R2 R1
+ 40 1 2
2 1

qh
a) (x, y) = 3
2(x2 +y 2 +h2 ) 2

P 5.7 b) Qdisco = qh
q, d = h 3
d2 +h2

q2
c) W = 320 h

 
x (x) = 2bR2 1
3 + 1
3
((xa)2 +b2 ) 2 ((x+a)2 +b2 ) 2
P 5.8  
y (y) = 2aR2 1
3 + 1
3
(a2 +(yb)2 ) 2 (a2 +(y+b)2 ) 2

q P8 (1)i1
V (x, y, z) = 4 i=1 ||(x,y,z)~
ri ||
, donde: ~r1 = (d cot( 8 ), d, 0),~r2 = (d, d cot( 8 ), 0),
P 5.9
0
~r3 = (d, d cot( 8 ), 0), ~r4 = (d cot( 8 ), d, 0), ~r5 = (d cot( 8 ), d, 0), ~r6 =

(d, d cot( 8 ), 0), ~r7 = (d, d cot( 8 ), 0), ~r8 = (d cot( 8 ), d, 0) y x y > 0.

 
2
R Rd q
P 5.10 () = 3 , qind = R q
d
4(R2 +d2 2Rd cos ) 2

P
 
~ = q 00 q0 R2
5.11 F q
40 d2
+ (dd0 )2
x donde q 0 = qR
d
, q 00 = Q + qR
d
, d0 = d

qq 0 qq 00
 
1 R2
, L = d2 + `2 2d` cos y L0 =
p
a) |F | = 0
+ L
40(L L ) 2 L 2
P 5.12 r
q Rd
b) =
(d `)2 R2 40 m`
209

Problema Respuestas
 
r 2 +h2 2rh sin R2
a) V (r, ) = 40
ln r 2 +(Rd)2 2r(Rd) sin
donde h = Rd
2

P b) () = 2R(R(d (2Rd)
2 +d2 2dR sin )
5.13
c) inducido =
2
~ = (Rd)
d) F y
20 d2

sinh( n y)
P 5.14 V (x, y) = 2V0
P n(cos(n)1)
(n2 4)
sin n
x
 a
n=1,n6=2 a sinh( n
a )
b

P
    
V0 P (1+(1)n+1 nx
 ny
 n(ax) n(by)
5.15 V (x, y) = n=1 n senh(nb/a) sen a
senh a
+ sen a
senh a

P 5.16 V (x, y) =
P 8aE0
n=0 (2n+1)2 2 cosh(b) sin(x) sinh(y), = (2n+1)
2a
210 CAPTULO 19. RESPUESTAS

Captulo 6

Problema Respuestas

a) Vdip (0, L, 0) = 0
P 6.1 ~ =
b) F 2qa0 b3
30 L3
z
c) W = 0

~ = 4R2 y, Vdip (x, 0, z) = 0


a) p
R2
P 6.2 ~ dip (x, 0, z) =
b) E 3 y
0 (x2 +z 2 ) 2

c) V (x, 0, z) = 0

a) ~r = (x, y, z)
 
~ = 0 R 3 r Rz
~ ~
r +Rz
Fuera de las dos esferas: E 30 r Rz||3
||~
r +Rz||3
||~

P
 
~ = 0 R3
6.3 Dentro de la esfera positiva: E 30
(~r Rz) ||~r+Rz|| 3 (~
r + Rz)
 
~ = 0 R3
Dentro de la esfera negativa: E 30 r Rz||3
||~
(~r + Rz) (~r + Rz) .

b) El campo elctrico decae como 1/r3 .

P
q
pE0
6.4 = I

 
px 1 1
V (x, y, z) = 3 3 ,
P 6.5
40
(x2 +y 2 +(zz0 )2 ) 2 (x2 +y 2 +(z+z0 )2 ) 2
3pz0 x
(x, y) = 5
2) 2
20 (x2 +y 2 +z0

L2
a) Q = 2

P ~ )= Q
6.6 b) E(P x Campo elctrico producido por una carga puntual de carga Q
40 `2
Q
c) V (P ) = 40 `
Potencial elctrico producido por una carga puntual de carga Q.
211

Captulo 7

Problema Respuestas

g1 +3g2
a) R =
P 7.1
24ag1 g2
3V0 0 (g1 g2 )
b) = 2a (g1 +3g2 )

P 7.2 C
f (r) = r 2 , g(r) = 2C.

a) J~ = V0 g0
L ln 2
y, ~ =
E V0
y
L(1+ L ) ln 2
y
P 7.3
V 2g
b) P = a2 L ln 2e
0 0

2 2 ln 1 + 3L

P 7.4 R= ln(z2 /z1 )


gk

P 7.5 I= 4kV 2
b )2 ,
ln( a
= V
b)
r 2 ln( a

P 7.6 R=
gd ln(1+ d )
r

LV0
a) I = ln(a/b)
(g1 + (2 )g2 )
ln(a/b)
P b) R1 = Lg1
7.7 ln(a/b)
R2 = L(2)g 2

RT = L(g1ln(a/b)
+(2)g2 )

2V
a) x2
+ 1 V
= 0, V (S) V (S 0 ) = E0 a ln(2)
P 7.8
x+a x
2
ln(2)
b) R = g0 a
, Q = 0 E20 a

P
 
g 1 a3
7.9 J~ r = a, t = 0
g
= 0
e 3r2

P 7.11 R= 1
(1
2g a1
+ 1
a2
2
L
)

a) J~ = r(g1
V0 g1 g2
ln(c/b)+g2 ln(b/a))
r,
V0 g2
r(g1 ln(c/b)+g2 ln(b/a)) r
a<r<b
~ =
E
V0 g1
r b<r<c


r(g1 ln(c/b)+g2 ln(b/a))

P 7.12 b) (r = a) = 2LV0 g2
(g1 ln(c/b)+g2 ln(b/a))
, 2LV0 g1
(r = c) = (g1 ln(c/b)+g 2 ln(b/a))
2LV0 g1 g2
c) I = g1 ln(c/b)+g2 ln(b/a)
 
1 ln(c/b) ln(b/a)
d) R = 2L g2
+ g1

V02 Lg1 g2
 
1 ln(c/b) ln(b/a)
e) RL = 2L g2
+ g1
,P = 2(g1 ln(c/b)+g2 ln(b/a))
212 CAPTULO 19. RESPUESTAS

Captulo 8

Problema Respuestas

P 8.1 R1 = R0

3

a) R = 0 , P = 900 [W ]
P 8.2
b) R = 20 , P = 45 [W ]

a) Req = 35 R
2V0 V0 V0
b) Las corrientes son i1 = 5R
, i2 = 5R
, i3 = 5R
P 8.3 (Nota: Las corrientes son simtricas, i1 es la corriente por la resistencia a la izquierda R, i2
es la corriente que pasa por las tres resistencias en serie e i3 es la corriente que pasa
por la resistencia central de la figura).

P 8.4 R = 600

P 8.5 RT = 56 R

P 8.6 |U | = Q0 C2
2C1 (C1 +C2 )

CV
a) Q(0) = 2
V
b) I(0) =
P 8.7
4R
V t/2RC
c) I(t) = 4R
e
CV 2
d) U (0) = 8

t
a) Q(t) = V0 Ce RC
P 8.8 t

~ = V0 Ce
RC
b) E z donde z apunta desde la placa cargada positiva a la negativa.
b2 0


P 8.9 R1 = R, R2 = 43 R, R3 = 11
15
R, R = ( 3 1)R.
213

Captulo 9

Problema Respuestas

P 9.1 ~ = 0 I k donde k est saliendo de la hoja de papel.


B 8R

 
P 9.2 ~ =
B 0 I
4b
1 a
z
a2 +b2

2
P ~ =
B IL
 2 0 q 2 x
9.3 2 L4 +x2 L +x2
2

P 9.4 ~ = D) = 0 I (x + y)
B(z 4D

P 9.5 Susana tiene la razn.

P 9.6 ~ =
B 0 I
4 ln 2
z


 
P 9.7 ~ =
B 0
4
(3 ~
2 4)Rz Propuesto: B(z) = 0
2
2z

2
+R2
2|z| z
R2 +z 2

~ = 0 I (sin A + sin C )z (z entrando a la hoja de papel).


a) B
P 9.8
4r
~ = 0 nI tan sin z
 
b) B a n n

P 9.9 ~ )=
B(P nI z
2
(cos 2 cos 1 )
214 CAPTULO 19. RESPUESTAS

Captulo 10

Problema Respuestas

P 10.1 = , d = 2 mv
eB
0
cos

P 10.2 t= m
qB

a) x(t) = 0,
qB0 z
y(t) = m
,
q
z(t) = m
(E 0 cos (t) B0 y)
b) x(t) = 0,
q 2 B0 E0 qB0
y(t) = qB (cos (t) cos m
t),
m2 ( m0 2 )
qE0
P z(t) = ( qB0
sin ( qB0
t) sin(t))
10.3 m(
qB0
m
2 ) m m

c) x(t) = 0,
q 2 E0 B0
y(t) = qB ( sin (t) m
qB0
sin qB0
m
t),
m2 ( m0 2 )
q 2 E0 qB0
z(t) = qB (cos (t) cos m
t)
m( m0 2 )
Si = qB0 /m la partcula entra en resonancia y la amplitud de su movimiento se
vuelve infinita.

qLB
a) |vcritico | = m
b) Si la velocidad es mayor que ese vcritico , la carga escapa de la zona de campo, sino la carga
P se devuelve siguiendo una trayectoria de semicircunferencia.
10.4 m
v
qB B
v0 < vcritico
c) h =
v arcsin qLB
m
qB B v0 m
v0 > vcritico

a) v = 6 107 [m/s].
P 1
10.5 b) |B| = [T]. El campo magntico debe apuntar hacia dentro de la hoja.
600
c) El electrn choca con una de las placas.

P 10.6 I2 = 2mgd(a+d)
0 I1 ab
, la corriente I2 debe tener sentido horario.

P
q
eB0 s 2

10.7 |~v | = m
R2 + 2

P La velocidad debe ser igual a v = 10 0 = c (velocidad de la luz). El resultado no es vlido


10.8
ya que para velocidad de esa magnitud no es posible usar la fsica tradicional.

2 2
a) L(r) = L0 + eB r a 2
q
P 10.9 b) v(b) = 2eV0
m
q
2eV0
c) Bc = e(b2mb
2 a2 ) m
215

Captulo 11

Problema Respuestas

3I0 I0
a) = 2a3
, = (c2 b2 )

0 I0 r 2

r<a
P 2a3


11.1 0 I0

a<r<b

~ =
b) B 2r
0 I0 c2 r 2


2r c2 b2
b<r<c
0 r>c

d
a) x
P 11.2
3
2
~
b) dF
dz
0I y
d2
y

P 11.3 ~ 1 ) = 0 I (x + y), B(P


B(P d
~ 2 ) = 0. (x creciendo hacia la derecha e y hacia arriba)


0 ln( ab )z r<a
P

~ = 0 (r 2 a2 )
11.4 B 2r
+ 0 ln( rb )z a<r<b
0 (b2 a2 )

r>b

2r

 0
 
0 I


2(bx)
2a
0I
ln xa x
k a<xbR
 0
 
I(xb) I
x
0
2a0
ln xa k bR<xb

2
~ =
a) B  2R 0
 
P 11.5




2(bx)

0 I
2a0 I
ln xa

x

k b<xb+R
0
I I
x

0
2a0
ln xa k x>b+R
2(xb)
0
 
~
F
b) dy = 2a
0 II
ln bab
x

~2
P 11.6
dF
dS
= 0 K1 K2 2 sin z donde z apunta desde plano 1 al 2 . Si (0, ), la fuerza es
atractiva, si (, 2) la fuerza es repulsiva, y para = 0, no existe fuerza.

P 11.7
~
B(z) = 02J0 k ~b, el vector ~b es que parte del centro del cilindro hasta el centro de la
circunferencia de radio a.

P 11.8 J= 3K
R

P 11.9
~
dF
= 0 I( R +
3 J1 w
)y
dl 3s 2

N1
a) I2 = N I1
P 11.10
2
~
b) dF
dS
= 0 (N1 I1 )2 r
216 CAPTULO 19. RESPUESTAS

Captulo 12

Problema Respuestas

0 r<R
P

12.1 ~
A(r) =
2
0I r

ln z rR

R

 1

0 IN 0 I (R2 +(pN )2 ) 2 +pN
a) Bz (0) = 1 , Az (0) = 4
ln 1
(R2 +p2 N 2 ) 2 (R2 +(pN )2 ) 2 pN

P 12.2
  2  12
b) f (R, N, p) = 1 + NRp
R 1
c) L
=
10 2

  q 
~= 0 L R2 ~= 0 2L

a) A 4
I ln R
1+ 1+ L2
z si L  R entonces A 4
I ln R
z
P 12.3 h i
~ 0 x2 +(dy) 2
b) A 4
I ln x2 +y 2
z

~ 0 0 R2 ~
a) B(0) = z A(0) =0
P 12.4
3
40 R5
b) m
~ = 15
z

P 12.5 ~ =
B 0 J0 r
2

P 12.6 = M g tan
2LB
y la direccin es horaria segn la figura.

P 12.7 I= Mg
5BR

q
N I0
P a) 0 = MR
12.8
b) ( = 0) = 0 0

P e2 B q R
12.9 |~ | = 40 m
2

0 m2
a) W = 0
,
P 12.10
2a3
30 m2
b) En A: ~ = 8a3
0
k, En C: ~ = 0. k sale de la hoja de papel.

P 12.11 k= 0 Iia2
R
217

Captulo 13

Problema Respuestas

~ = 0 mI0 sen(t)z
a) B
P 13.1 ~ t) = r0 mI0 cos(t)
b) E(r, 2
2
~ t) = b
c) E(r, 0 mI0 cos(t)

2r

 2

a) v(t) = v0 exp (aB)
mR
t
P 13.2
h 2 2
b) v(t) = v0 1 BmRa
1 exp t
i
, Q(t) = Bav0
1 exp t

, donde 1
= 1
+
R RC
2 2
B a
mR

P 13.3 v(t) = BLbt2


y (t) = B 2 L2 bt2
si t [0, T ].
2m 2m

Ra2 B
P a) f = I
13.4
b) Es independiente de B(t), solamente importa el valor inicial y final.

P 13.5 B= 2QR
N D 2

 2

a) v(t) = v0 exp (BmR
0 b)
t xmax = v0 mR
(B0 b)2
P 13.6 
(B0 b)2

b) v(t) = BV00b 1 e mR t

P 13.7 P = gh(a2 0 I0 cos t)2


, Prop. I = gha
2
0 nI0 cos t
8 4

P 13.8 vT = mRg
B 2 a2
, el sentido de la corriente en la figura es antihorario.

a) Antihorario (considerando normal positiva +z)


2
b) I = Ca z
P 13.9
R
~r = 0, F
c) F ~z = a2 CI z
mgR
d) vT = (Ca2 )2

P 13.10 |V | = 0 vI
2
ln 1 + d
a

, si a  d entonces |V | 0 vId
2a

P 13.11 || = B0 R2
2
218 CAPTULO 19. RESPUESTAS

Captulo 14

Problema Respuestas

P 14.1 M= 0 h tan

0 (ab)2
P 14.2 M= 3
2(d2 +a2 ) 2

P 14.3 M = 20 a

a) M = 0 nb2 sin
P 14.4
b) Vmax = 0 nb2 I0 sin

|| 0 d ln(4/3)
a) M = I
= 2
P 14.5 b) Antihorario en la figura.
~tot = 0 I 0 d ln 3 dI y

c) F 6 2r 4 dt

0 I02 b

a) um = ln
P 14.6 4 a
0 b

b) l = 2
ln a

P 14.7 a) L = 0 N 2 h
ln b

2 a

P 14.8 Um = 0 (0 t)2 R4 H
Ue = 0 (0 )2 R6 H
2 16

a)  = a2 B0 0 sin(0 t)
a2 B0 R
P 14.9 b) I(t) = 2 L2 (R sin 0 t
R2 +0
0 L(cos 0 t e L t ))
2 a4 B0
2
c) (t) = R2 +02 L2 (R sin 0 t 0 L cos 0 t) sin 0 t


P v(t) = v0 cos aB t, i(t) = mv 0
sin aB t. El largo del carrito debe ser menor a v0 mL
14.10 mL mL mL aB
para que logre entrar.

0 I1 0 kabI1
a) = 2
a ln z+b
z
, = 2
P 14.11 b) V0 +  = I2 R + L dI2
dt
R
c) I2 (t) = V0
R
+ 
R
(1 e L t )

a) I3 = 0 (la corriente no puede aumentar en forma abrupta en una inductancia), I1 = I2 =


V0
P 14.12
R1 +R2
V0
b) I1 = I3 = R1
, I2 = 0 (la inductancia acta como un cortocircuito en un tiempo muy
largo).
219

Problema Respuestas

a) I(0) = 4 A. e I() = 8 A.
P 14.13
b) I(t) = 8 4e300t A.


3R 2
0 R
a) L = 20
, C= 3
P 14.14
q
b) Q(t) = Aet cos( 0 t + ) con = R
2L
, 0 = R 2
( 2L 1 2
) + ( LC ) ,A= Q0
cos
y tan =
0
220 CAPTULO 19. RESPUESTAS

Captulo 15

Problema Respuestas

1
b) = 2 arctan RC
P 15.1 c) Cambian los dos resultados anteriores, ya que ocurre una divisin de voltaje previa que
cambia el mdulo y el desfase de la diferencia de potencial.

a) C = 281n[F ]
b) R = (225)ej1,11
P 15.2 c) I(t) = 0,89 cos(10000t 1,11) [A]
d) hP i = 39,6 [W]
R L C
e) Imax = 0,89 [A] , Vmax = 89 [V], Vmax = 89 [V], Vmax = 100,7 [V]

2
LC1 1
a) Z = R + j (C1 +C 2
2 LC1 C2 )

b) 0 = 1
LC1
r  2
V0 2 LC1 1
P c) I(t) = |Z|
cos(t ) donde |Z| = R2 + (C1 +C2 2 LC1 C2 )
15.3
 2

LC1 1
= arctan R(C1 +C2 2 LC1 C2 )

V02 R
d) hP i = 
2 LC1 1
2
2R2 +2
(C1 +C2 2 LC1 C2 )


P 15.4 Imax = 7 2 > 7 [A] . El enchufe no es capaz de trabajar con esa corriente.

1
a) Zeq = ( R1 +jX + 1
R2 jXC
)1 , donde XL = L y XC = 1
C
, |I1 (t)| = V0
2 + 2 L2
L R1
|I2 (t)| = 2 V0 1 2
P 15.5 R2 +( C )
r
R2 CL (R1 +R2 )2 +(0 L 1C )2
1
b) 0 = LC R12 CL y |I| = V0 L )2 +( LR 1 R )2
(R1 R2 + C
0
2 0 2 C 1
0

r
V2 Rampolleta P R2
rms-chile ampolleta
P 15.6 Rampolleta = 240, L = P (2fchile )2
= 1,2[H] (una inductancia muy
grande!). Es mejor un inductor ya que no consume potencia.

a) Contiene un inductor y un condensador.


P 15.7 b) C = 0,5 F y L = 1H.

c) S, y la frecuencia es 0 = 2 rad/s .

a) R = 100, C 105 F, V (t) = 23 sin(100t +


2
), P = 72,25 W.
L
R C
P b) 0 = 1 , |Z| =
15.8
q
LC L
( C ) +R ( 2 L2 21 2
2 2 )
C

Vrms 2 1
c) Imax = R
, esto ocurre en resonancia con = 0 = LC
y VLC ( = 0 ) = 0.

P 15.9 La potencia consumida es de 616 W, por lo que bastara un transformador de 800 W.

|VEF |2 RC |VEF |2 1
a) Caso (I): |VAB |2
= , Caso (II): |VAB |2
=
1+(RC)2 1+(RC)2
P 15.10
b) El Sonido (I) viene del filtro pasa alto (Caso I) y el Sonido (II) viene del filtro pasa bajos
(Caso II).
221

Captulo 16

Problema Respuestas

a) =
5
myT = 2
3
109 s.
b) La onda se propaga en x.
P 16.2 ~ =
c) B E0
c
cos[(10 m1 ) x + (3 109 s1 ) t]y
2
~ = E0 cos2 [(10 m1 ) x + (3 109 s1 ) t]x
d) S 0 c

y+z
~ = E0 xej100( c2 t)
a) E
P ~ = E j100( y+z
t)
16.3 B 0 (y
c 2
z)e c 2

2
~ =
b) h|S|i
E0
0 c

P 16.4 ~ = aB0 sin


E
x
a

sin (kz t) y

a) hSi = 1
c0
(C12 + C22 )z
~
b) E(z = 0, t) = C(x cos t + y sin t). El campo elctrico rota en plano xy pasado un
P 16.5 periodo.
~
c) E(z, t = 0) = C(x cos kz + y sin kz). El campo elctrico rota a medida que se avanza de
una longitud de onda a otra.

~=
a) S 1
cos(kx t) + E20 cos(kx t + )]2 x
[E10
c0
h 2
E2
i
P 16.6
E
b) I = c10 210 + 220 + E10 E20 cos
h 2
E2
i
E
c) I = c10 210 220

a) b = ac
b) E = cB0 , la onda se propaga en la direccin x
P 16.7 ~ = cB02
c) hSi 0
x
2
B0
d) P = 0

~ = E0 xy
a) E
cos( (x + y) t)
2 c 2

P 16.8 ~ = E0 z cos(
B c

c 2
(x + y) t)
2
~ =
b) h|S|i
E0
20 c

k z j(k zt+ )
~ =
b) B E0 k0
2e 0 e 0 4
P 16.9

2
~ =
c) h|S|i
E0 k0 2k0 z
e
20
222 CAPTULO 19. RESPUESTAS

Captulo 17

Problema Respuestas

0 r < R1




~ = Q
a) E 4r 2
r R1 < r < R2



P
Q
r r > R2

17.1 40 r 2

b) pol (R1 ) = (



0) Q
2,
4R1
pol (R2 ) = (

0) Q Q
2 , libre (R1 ) = 4R2
4R2 1
 
Q 1 1 Q
c) V1 (R1 ) = 4 R1
R2 + 40 R2 , el potencial en la esfera es mayor en ausencia del
dielctrico.


0 r<R
~ =
a) E
Q
r r>R

2r 2 (1 +2 )
P 17.2 (1 0 )Q (2 0 )Q
b) l1 = 2(1+
1Q
2 )R
2 , l2 =
2 Q
2(1 +2 )R2
, P 1 = 2R 2 ( + ) , P 2 = 2R2 ( + )
1 2 1 2

QP Q
c) Q = 39
40
, Qneta = 40

P 17.3 ~ext =
F Q2
L2
ln b
a
 2 1
(1 +2 )2
z

V0 (ekx 1)
a) V (x) = ekd 1
P 17.4 b) (x) = 1 ekx
kV0
c) l = ekd 1

P 17.5 C= R2
(1 + 22 )
3d

a
a) CT = d
(x + (a x)0 )
 
1 Q b
b) x = 0
a
V0 a
P 17.6
~ =
c) F (0 )aV0 2
x, donde x crece de derecha a izquierda.
2d
2
0 a a2
d) d
Q(x) d

2
a) p (r = b) = (40 )0 b
, p (r < b) = (0 )0 r
P 17.7 2 b7
 
1
b) U = 80 7
+ 10

P 17.8 ~2 = (2 0 )((


P 2
1
E
2 1
sin 1 )x + E1 cos 1 y)

a) 1 = d200d+d
2 0
, 2 = d1+d
0 d1 

P 17.9
1 2 0
A0 0 Rv0 t
b) V = (d2 0 +(d1 v0 t))2
223

Problema Respuestas

a) p = P0 , p = P20 a
P0 r
20 r r < a
P 17.10 ~ =
b) E
0 r>a

~
c) B(z) =0

a) Condensador izquierda:
J~ = d(g
V0 g1 g2
1 +g2 )
~ 1 = V0 g2 z, E
z, E d(g1 +g2 )
~ 2 = V0 g1 z, D
d(g1 +g2 )
~ 1 = 1 V0 g2 z, D
d(g1 +g2 )
~2 = 2 V0 g1
d(g1 +g2 )
z
Condensador derecha:
E~ = V0 z, J~1 = g1 V0 z, J~2 = g2 V0 z, D
~ 1 = 1 V0 z, D
~ 2 = 2 V0 z
2d 2d 2d 2d 2d
b) Densidades superficiales de polarizacin:
P 1 (z = 0) = (1 0 ) V2d0 , P 1 (z = 2d) = (1 0 ) V2d0 , P 2 (z = 0) = (2 0 ) V2d0 ,
P 17.11 P 2 (z = 2d) = (2 0 ) V2d0
Densidades superficiales de carga libre:
L1 (z = 0) = 1 V2d0 , L1 (z = 2d) = 1 V2d0 , L2 (z = 0) = 2 V2d0 , L2 (z = 2d) = 2 V2d0
No existen densidades volumtricas de carga de ningn tipo.
4a2 1 2 1 a2 2 a2
c) CT = d(1 +2 )
+ d
+ d
d
d) R = 
4

a2 g1 g2 g +g + g1 + g1
1 2 2 1

L
P 17.12 C= ln(2)
+
ln(3)ln(2) ln(3)
+  +
21 22 1 2
224 CAPTULO 19. RESPUESTAS

Captulo 18

Problema Respuestas

0 rJ0

2
r < 3a




9a2 J0

P 3a < r < 4a

18.1 ~ =
B 2r


0 J0
(25a2 r2 )




2r
4a < r < 5a
0 r > 5a

P 18.2 ~ =
H 1+k2 cos2 N I
2+k r

P 18.3 central = N I0 A
18b
y derecho = N I0 A
45b

 
 1 1 mI z si 0 <
P ~ = mI z, B
~ = 1 mI z si 0 < ~ = 0
18.4 H ,M  
2 mI z si < 2 2 1 mI z si < 2
0

~ =
a) B 1 2 I
~1 =
H 2 I
~2 =
H 1 I
~3 = 0
H
P 18.5
R(1 2 +2 1 ) R(1 2 +2 1 ) R(1 2 +2 1 )

~ 1| =
b) |K 1 0 2 I ~ 2| =
|K 2 0 1 I
J~ = 0
0 R(1 2 +2 1 ) 0 R(1 2 +2 1 )


rV0 r rV0

2s L
r<R 1
R 2s L
r<R
~ =
a) H ~ =
B
V0 R2

r>R

2
0 V0 R

2
r>R
P

2s Lr s Lr
18.6
2
~ (r < R) =
b) M r V0
, ~ (r > R) = 0
M
2s RL

c) J~libre = Vs0L z, J~M = 23rV 0 ~ M = R2 V0 z


z, K
s LR 2s RL

 
~1 =
a) H N IMs `
, ~0 =
H +M ~ s, B
~1 = B
N IMs ` ~ 0 = 1 N IMs ` + M
~s .
2R 2R 0 2R
P 18.7  
~ 1 = Mr ` , H
b) H ~0 = M
~ r Mr ` , B
~1 = B ~0 = 1 M ~ r Mr ` .
2R 2R 0 2R

N I0 0 a2 N 2 0 a2
a) = , L=
P 18.8
32a0 +2g 32a0 +2g

~| = N 2 I02 2 0 a2
b) |F (32a0 +2g)2
la fuerza es atractiva.

~ =
a) B 1,6I0
, ~ =
H I0

2000r+I0 2r
P 18.9  
~ =
b) M 1,6I0
I0

0 (2000r+I0 ) 2r


NI
~ 1| =
a) |H NI
, ~ 2| =
|H 0
|B| = N I
2R2 +( 1)e 2R2 +( 1)e 2R2 +( 1)e
P 18.10 0 0 0

ge
 h R12 i
b) P = 2
B
t 4
+ R14 ln a
R1

a) K ~ 2 = M0 x x, K
~ 1 = M0 x x, K ~ 3 = M0 x. En las otras caras no hay densidades superficiales.
a a
P 18.11 J~ = 0 y
M
a

b) J~ = 0, K
~ = M0 z sin , B
~ = 0 M0
2R
x
Bibliografa

[1] Barger, V., and Olsson, M. G. Classical electricity and magnetism: a contemporary
perspective. Allyn & Bacon, 1987.

[2] Cordero, P. Electromagnetismo. Editorial Universitaria, 2015.

[3] Dourmashkin, P., Liao, S.-B., and Winston Belcher, J. Introduction to electricity
and magnetism. Pearson, 2013.

[4] Griffiths, D. J., and College, R. Introduction to electrodynamics, vol. 3. Prentice Hall
Upper Saddle River, NJ, 1999.

[5] Lim, Y.-K. Problems and solutions on quantum mechanics, vol. 6. World Scientific, 1998.

[6] Nayfeh, M. H., and Brussel, M. K. Electricity and magnetism. Courier Dover Publications,
2015.

[7] Nez Cubero, F., Lpez Prez, E., et al. 100 problemas de electromagnetismo. Alianza
Editorial, 1997.

[8] Pollack, G. L., and Stump, D. R. Electromagnetism. Addison-Wesley, 2002.

[9] Purcell, E. M. Electricidad y magnetismo, vol. 2. Revert, 1988.

[10] Rodrguez, V. L. Problemas resueltos de Electromagnetismo. Editorial Universitaria Ramon


Areces, 2003.

[11] Serway, R. A., Jewett, J. W., Pedroza, M. d. C. R., and Lpez, E. F. Fsica para
ciencias e ingeniera, 7 ed., vol. 2. Cengage Learning Editores, 2015.

225

Das könnte Ihnen auch gefallen